NPTE : Grays Anatomy Review (UE + LE)

Réussis tes devoirs et examens dès maintenant avec Quizwiz!

65 A 48-year-old female piano player visited the outpatient clinic with numbness and tingling in her left hand. A diagnosis was made of nerve compression in the carpal tunnel, and the patient underwent an endoscopic nerve release. Two weeks postoperatively the patient complained of a profound weakness in the thumb, with loss of thumb opposition. The sensation to the hand, however, was unaffected. Which of the following nerves was injured during the operation? A. The first common digital branch of the median nerve B. The second common digital branch of the median nerve C. Recurrent branch of median nerve D. Deep branch of the ulnar nerve E. Anterior interosseus nerve

65 C. The recurrent branch of the median nerve innervates the thenar muscles (opponens pollicis, abductor pollicis brevis, and flexor pollicis brevis) and is not responsible for any cutaneous innervation. Damage to the palmar cutaneous branches of the median nerve or to the ulnar nerve would not cause weakness of opposition of the thumb for they are principally sensory in function. The deep branch of the ulnar nerve supplies the hypothenar muscles, adductor and abductor muscles of digits 2-5, and does not innervate the abductor pollicis brevis. GAS 817; N 463; McM 159

21 A 36-year-old man is brought to the emergency department because of a deep knife wound on the medial side of his distal forearm. He is unable to hold a piece of paper between his fingers and has sensory loss on the medial side of his hand and little finger. Which nerve is most likely injured? A. Axillary B. Median C. Musculocutaneous D. Radial E. Ulnar

E. The ulnar nerve innervates the palmar interossei, which adduct the fingers. This is the movement that would maintain the paper between the fingers. The axillary nerve does not innervate muscles of the hand. The median nerve supplies the first and second lumbricals, the opponens pollicis, abductor pollicis brevis, and the flexor pollicis brevis muscles. None of these muscles would affect the ability to hold a piece of paper between the fingers. The musculocutaneous and radial nerves do not supply muscles of the hand.

100 Physical examination reveals weakness of medial deviation of the wrist (adduction), loss of sensation on the medial side of the hand, and clawing of the fingers. Where is the most likely place of injury? A. Compression of a nerve passing between the humeral and ulnar heads of origin of flexor carpi ulnaris B. Compression of a nerve passing at Guyon's canal between the pisiform bone and flexor retinaculum C. Compression of a nerve passing through the carpal tunnel D. Compression of a nerve passing between the ulnar and radial heads of origin of flexor digitorum superficialis E. Compression of a nerve passing deep to brachioradialis muscle

100 A. The ulnar nerve enters the forearm by passing between the two heads of the flexor carpi ulnaris and descends between and innervates the flexor carpi ulnaris (for medial wrist deviation) and flexor digitorum profundus (medial half) muscles. Injuring the ulnar nerve results in claw hand. It enters the hand superficial to the flexor retinaculum and lateral to the pisiform bone, where it is vulnerable to damage. The ulnar nerve also enters Guyon's canal, but damage to it here would not present with the aforementioned symptoms. The median nerve enters the carpal tunnel and the radial nerve passes deep to the brachioradialis. GAS 777, 784; N 464; McM 145

103 A 10-year-old boy suffered a dog bite that entered the common flexor synovial sheath of his forearm. He was admitted to the hospital, where the wound was cleaned and dressed and he was treated further with rabies antiserum. Two days later the boy had an elevated temperature and his palm and one digit were obviously swollen, causing him to cry with pain. Into which of the digits could the infection spread most easily, following the anatomy of the typical common flexor sheath? A. First B. Second C. Third D. Fourth E. Fifth

103 E. The common flexor sheath encloses the long flexor tendons of the fingers in the carpal tunnel and proximal palm. This sheath is usually continuous with the flexor sheath of the little (fifth) finger, which continues within the palm, having no connection with sheaths of the other digits, which do not extend into the digits. GAS 800-802; N 448; McM 158

109 A 69-year-old man has numbness and pain in the middle three digits of his right hand at night. He retired 9 years ago after working as a carpenter for 30 years. He has atrophy of the thenar eminence (see Fig. 6-3). Which of the following conditions will be the most likely cause of this atrophy? A. Compression of the median nerve in the carpal tunnel B. Formation of the osteophytes that compress the ulnar nerve at the ulnar condyle C. Hypertrophy of the triceps brachii muscle compressing the brachial plexus D. Osteoarthritis of the cervical spine E. Repeated trauma to the ulnar nerve

109 A. The median nerve supplies sensory innervation to the thumb, index, and middle fingers as well as to the lateral half of the ring finger. The median nerve also provides motor innervation to muscles of the thenar eminence. Compression of the median nerve in the carpal tunnel explains these deficits in conjunction with normal functioning of the flexor compartment of the forearm. The ulnar nerve is not implicated in these symptoms. Compression of the brachial plexus could not be attributed to pressure from hypertrophy of the triceps brachii muscle, it is located distal to the plexus. In addition, symptoms would include several upper limb deficits rather than the focal symptoms described in this instance. Osteoarthritis of the cervical spine would also lead to increasing complexity of symptoms. GAS 745, 817; N 463; McM 159

149 A 22-year-old football player suffered a wrist injury after falling on his outstretched hand. When the anatomical snuffbox is exposed in surgery, an artery is visualized crossing the fractured bone that provides a floor for this space. Which of the following arteries was most likely visualized? A. Ulnar B. Radial C. Anterior interosseous D. Posterior interosseous E. Deep palmar arch

149 B. The radial artery enters the anatomical snuffbox as it passes to the posterior aspect of the hand to pass between the two heads of the 1st dorsal interosseous muscle. The ulnar artery continues anteriorly and enters the hand on the palmar surface. The anterior and posterior interosseous arteries are found anteriorly and posteriorly, respectively, on the interosseous membrane, which is located between the radius and ulna. The deep palmar arch is an anastomosis on the palmar surface of the hand that is formed by the radial artery and the deep branch of the ulna artery and lies on the anterior surface of the hand. GAS 782, 800, 810-815; N 454; McM 161

151 A 28-year-old telephone company worker falls off a street pole during a telephone line repair and lands directly on his right shoulder. Plain radiographs reveal a vertical fracture through the entire length of the floor of the intertubercular sulcus of the right humerus. The muscle that is most likely affected by the fracture is innervated by a nerve that is composed of which of the following nerve roots? A. C3 and C4 B. C6 to C8 C. C4 and C5 D. C2 to C4 E. C5 to C7

151 B. The muscle that attaches into the intertubercular sulcus of the humerus is the latissimus dorsi. Nerve supply is via the thoracodorsal nerve, which is a branch of the posterior cord and is made up of roots C6-8. Nerves C2, C3 and C4 are not part of the brachial plexus but of the cervical plexus and will supply the "strap" muscles. Nerves C4 and C5 are the main contributions to the phrenic nerve, and C5 does not contribute to the formation of the thoracodorsal nerve. GAS 728; N 416; McM 115

72 Fine motor function in the right hand of a 14-yearold girl with scoliosis since birth appeared to be quite reduced, including opposition of the thumb, abduction and adduction of the digits, and interphalangeal joint extension. Radiography confirmed that her severe scoliosis was causing marked elevation of the right first rib. Long flexor muscles of the hand and long extensors of the wrist appear to be functioning within normal limits. There is notable anesthesia of the skin on the medial side of the forearm; otherwise, sensory function in the limb is intact. Which of the following neural structures is most likely impaired? A. Median nerve B. Middle trunk of the brachial plexus C. Radial nerve D. Lower trunk of the brachial plexus E. T1 ventral ramus

72 E. Scoliosis (severe lateral curvature of the spine) in the patient is causing compression or stretching of the T1 spinal nerve ramus by the first rib as the nerve ascends to join C8 and form the lower trunk of the brachial plexus. T1 provides sensation for the medial side of the forearm, via the medial antebrachial cutaneous nerve from the medial cord of the brachial plexus. T1 is the principal source of motor supply to all of the intrinsic muscles in the palm. Its dysfunction affects all fine motor movements of the digits. Long flexors of the fingers are intact; therefore, the median nerve and ulnar nerve are not injured. The extensors of the wrist are functional; therefore, the radial nerve is not paralyzed. The only sensory disturbance is that of the T1 dermatome. GAS 695-700, 744-745; N 161; McM 94

75 A 43-year-old female tennis player visits the outpatient clinic with pain over the right lateral epicondyle of her elbow. Physical examination reveals that the patient has lateral epicondylitis. Which of the following tests should be performed during physical examination to confirm the diagnosis? A. Nerve conduction studies B. Evaluation of pain experienced during flexion and extension of the elbow joint C. Observing the presence of pain when the wrist is extended against resistance D. Observing the presence of numbness and tingling in the ring and little fingers when the wrist is flexed against resistance E. Evaluation of pain felt over the styloid process of radius during brachioradialis contraction

75 C. The common extensor tendon originates from the lateral epicondyle, and inflammation of this tendon is lateral epicondylitis, nicknamed "tennis elbow" because the tendon is often irritated during the backhand stroke in tennis. Because the extensors of the wrist originate as part of the common extensor tendon, extension of the wrist will exacerbate the pain of lateral epicondylitis. GAS 768, 785; N 427; McM 152

79 A 68-year-old woman fell when she missed the last step from her motor home. Radiographic examination at the local medical care center reveals a fracture of the distal radius. The distal fragment of the radius is angled forward. What name is commonly applied to this type of injury? A. Colles' fracture B. Scaphoid fracture C. Bennett's fracture D. Smith's fracture E. Boxer's fracture

79 D. Colles' fracture is a fracture of the distal radius with the distal fragment displaced dorsally. Smith's fracture involves the distal fragment displaced in a volar direction. Smith's fracture is sometimes referred to as a reverse Colles' fracture. GAS 771-774; N 440; McM 153

85 A 43-year-old man is admitted to the hospital, having suffered a whiplash injury when his compact automobile was struck from behind by a sports utility vehicle. MRI examination reveals some herniation of a disc in the cervical region. Physical examination reveals that the patient has lost elbow extension; there is absence of his triceps reflex and loss of extension of the metacarpophalangeal joints on the ipsilateral side. Which of the following spinal nerves is most likely affected? A. C5 B. C6 C. C7 D. C8 E. T1

85 C. C7 is the main spinal nerve that contributes to the radial nerve and innervates the triceps brachii. Absence of the triceps reflex is usually indicative of a C7 radiculopathy or injury. GAS 745-746, 756; N 416; McM 144

86 A 29-year-old patient has a dislocated elbow in which the ulna and medial part of the distal humerus have become separated. What classification of joint is normally formed between these two bones? A. Trochoid B. Ginglymus C. Enarthrodial D. Synarthrosis E. Sellar

86 B. Ginglymus joint is the correct technical term to describe a hinge joint. It allows motion in one axis (flexion and extension in the case of the humeroulnar joint) and is therefore a uniaxial joint. The other types of joints listed allow motion in more than one axis. GAS 764; N 442; McM 120

88 A 55-year-old female choreographer had been treated in the emergency department after she fell from the stage into the orchestra pit. Radiographs revealed fracture of the styloid process of the ulna. Disruption of the triangular fibrocartilage complex is suspected. With which of the following bones does the ulna normally articulate at the wrist? A. Triquetrum B. Hamate C. Radius and lunate D. Radius E. Pisiform and triquetrum

88 D. Normally the distal part of the ulna articulates only with the radius at the distal radioulnar joint at the wrist, a joint that participates in pronation and supination. The head of the ulna does not articulate with any of the carpal bones; instead, it is separated from the triquetrum and lunate bones by the triangular fibrocartilage complex between it and the radius. The pisiform articulates with the triquetrum. The carpal articulation of the radius is primarily that of the scaphoid (the old name is navicular) bone. GAS 764-765; N 439; McM 123

91 A 45-year-old man visits the outpatient clinic after a digit of his left hand was injured when a door was slammed on his hand. A superficial cut on his middle finger has been sutured, but functional deficits are observed in the finger. The proximal interphalangeal joint is pulled into constant flexion, whereas the distal interphalangeal joint is held in a position of hyperextension. What is the most likely diagnosis? A. Mallet finger B. Boutonnière deformity C. Dupuytren's contracture D. Swan-neck deformity E. Silver fork wrist deformity

91 B. In boutonnière deformity, the central portion of the extensor tendon expansion is torn over the proximal interphalangeal (PIP) joint, allowing the tendon to move toward the palm, causing the tendon to act as a flexor of the PIP joint. This causes the distal interphalangeal (DIP) joint to be hyperextended. The tear in the extensor tendon is said to resemble a buttonhole (boutonnière in French), and the head of the proximal phalanx may stick through the hole. GAS 795-796; N 451; McM 166

94 A 72-year-old man consulted his physician because he had noticed a thickening of the skin at the base of his left ring finger during the preceding 3 months. As he described it, "There appears to be some hard tissue that is pulling my little and ring fingers into my palm." On examination of the palms of both hands, localized and firm ridges are observed in the palmar skin that extend from the middle part of the palm to the base of the ring and little fingers. What is the medical term for this sign? A. Ape hand B. Dupuytren's contracture C. Claw hand D. Wrist drop E. Mallet finger

94 B. Dupuytren's contracture or deformity is a result of fibromatosis of palmar fascia, resulting in irregular thickening of the fascial attachments to the skin, which causes gradual contraction of the digits, especially digits 4 and 5. In 50% of cases, it is bilateral in occurrence. Ape hand, or flat hand, is a result of loss of the median and ulnar nerves at the elbow, with paralysis of all long flexors of the fingers and all intrinsic hand muscles. The term can also be specific for just median nerve injury and a flattened thenar eminence. Claw hand results from paralysis of interphalangeal joint extension by interossei and lumbricals, innervated primarily by the ulnar nerve. Wrist drop occurs with radial nerve paralysis and loss of the extensors carpi radialis longus and brevis. Mallet finger results from detachment of the extensor mechanism from the distal phalanx of a finger and unopposed flexion of that distal interphalangeal joint. GAS 800; N 446; McM 157

97 After an orthopedic surgeon examined the MRI of the shoulder of a 42-year-old woman he informed her that the supraspinatus tendon was injured and needed to be repaired surgically. Which of the following is true of the supraspinatus muscle? A. It inserts onto the lesser tubercle of the humerus. B. It initiates adduction of the shoulder. C. It is innervated chiefly by the C5 spinal nerve. D. It is supplied by the upper subscapular nerve. E. It originates from the lateral border of the scapula.

97 C. The supraspinatus muscle inserts on the greater tubercle of the humerus and is said to initiate abduction of the arm at the shoulder. It is supplied principally by spinal nerve C5. The subscapularis muscle is the only muscle that inserts on the lesser tubercle. The subscapularis muscle is innervated by the upper and lower subscapular nerves. The teres minor muscle takes origin from the lateral border of the scapula; the teres major muscle takes origin from the region of the inferior angle and the lateral border of the scapula. GAS 717; N 411; McM 115

A. The talocrural (tibiotalar, ankle) joint is a hinge-type synovial joint between the tibia and talus. It permits dorsiflexion and plantar flexion, and fracture of this joint would affect these movements

A 37-year-old man is admitted to the hospital after an injury to his foot while playing flag football with friends on a Saturday morning. A series of radiographs demonstrates a fracture involving the talocrural (tibiotalar, ankle) joint. Which movements are the major ones to be affected by this injury? A. Plantar flexion and dorsiflexion B. Inversion and eversion C. Plantar flexion, dorsiflexion, inversion, and eversion D. Plantar flexion and inversion E. Dorsiflexion and eversion

34 A 21-year-old female softball pitcher is examined in the emergency department after she was struck in the arm by a line drive. Plain radiographic and MRI studies show soft tissue injury to the region of the spiral groove, with trauma to the radial nerve. Which of the following muscles would be intact after this injury? A. Flexor carpi ulnaris B. Extensor indicis C. Brachioradialis D. Extensor carpi radialis longus E. Supinator

A. The flexor carpi ulnaris muscle is not innervated by the radial nerve but rather by the ulnar nerve. The brachioradialis, extensor carpi radialis longus and brevis, and supinator muscles are all innervated by the radial nerve distal to the spiral groove. GAS 777, 787; N 418; McM 149

39 The kidneys of a 32-year-old woman were failing, and she needed to be placed on dialysis. However, the search in her upper limb for a suitable vein was unexpectedly difficult. The major vein on the lateral side of the arm was too small; others were too delicate. Finally, a vein was found on the medial side of the arm that passed through the superficial and deep fascia to join veins beside the brachial artery. Which of the following veins was this? A. Basilic B. Lateral cubital C. Cephalic D. Medial cubital E. Medial antebrachial

A. The basilic vein can be used for dialysis, especially when the cephalic vein is judged to be too small, as in this case. The basilic vein can be elevated from its position as it passes through the fascia on the medial side of the arm (brachium). The cephalic vein passes more laterally up the limb. The lateral cubital vein is a tributary to the cephalic vein, and the medial cubital vein joins the basilic vein, both of which are rather superficial in position. The medial antebrachial vein courses up the midline of the forearm (antebrachium) ventrally. GAS 700, 759, 769-770; N 401; McM 148

14 A 45-year-old man is admitted to the hospital after accidentally walking through a plate glass door in a bar while intoxicated. Physical examination shows multiple lacerations to the upper limb, with inability to flex the distal interphalangeal joints of the fourth and fifth digits. Which of the following muscles is most likely affected? A. Flexor digitorum profundus B. Flexor digitorum superficialis C. Lumbricals D. Flexor digitorum profundus and flexor digitorum superficialis E. Interossei

A. The flexor digitorum profundus muscle is dually innervated by the ulnar nerve to the medial two fingers and the median nerve for the long and index fingers. Because of the superficial course of the ulnar nerve, it is vulnerable to laceration. Such an injury would result in an inability to flex the distal interphalangeal joints of the fourth and fifth digits because the flexor digitorum profundus muscle is the only muscle that flexes this joint. The flexor digitorum superficialis muscle is innervated by the median nerve only, and the course of this nerve runs too deep to be usually affected by lacerations. The lumbricals function to flex the MP joints and assist in extending the IP joints. The interossei adduct and abduct the fingers.

2 A 27-year-old man was admitted to the emergency department after an automobile collision in which he suffered a fracture of the lateral border of the scapula. Six weeks after the accident, physical examination reveals weakness in medial rotation and adduction of the humerus. Which nerve was most likely injured? A. Lower subscapular B. Axillary C. Radial D. Spinal accessory E. Ulnar

A. The lower subscapular nerve arises from the cervical spinal nerves 5 and 6. It innervates the subscapularis and teres major muscles. The subscapularis and teres major muscles are both responsible for adducting and medially rotating the arm. A lesion of this nerve would result in weakness in these motions. The axillary nerve also arises from cervical spinal nerves 5 and 6 and innervates the deltoid and teres minor muscles. The deltoid muscle is large and covers the entire surface of the shoulder, and contributes to arm movement in any plane. The teres minor muscle is a lateral rotator and a member of the rotator cuff group of muscles. The radial nerve arises from the posterior cord of the brachial plexus. It is the largest branch, and it innervates the triceps brachii and anconeus muscles in the arm. The spinal accessory nerve is cranial nerve XI, and innervates the trapezius muscle, which elevates and depresses the scapula. The ulnar nerve arises from the medial cord of the brachial plexus and runs down the medial aspect of the arm. It innervates muscles of the forearm and hand.

29 A 45-year-old man is admitted to the hospital after a car crash. Radiographic examination reveals mild disc herniations of C7, C8, and T1. The patient presents with a sensory deficit of the C8 and T1 spinal nerve dermatomes. The dorsal root ganglia of C8 and T1 would contain cell bodies of sensory fibers carried by which of the following nerves? A. Medial antebrachial cutaneous nerve B. Long thoracic nerve C. Lateral antebrachial cutaneous nerve D. Deep branch of ulnar nerve E. Anterior interosseous nerve

A. The medial antebrachial cutaneous nerve carries sensory fibers derived from the C8 and T1 levels. The lateral antebrachial cutaneous nerve is the distal continuation of the musculocutaneous nerve, carrying fibers from the C5, C6, and C7 levels. The deep branch of the ulnar nerve and the anterior interosseous nerves carry predominantly motor fibers. The sensory fibers coursing in the radial nerve are derived from the C5 to C8 levels.

50 A 22-year-old male construction worker is admitted to the hospital after he suffers a penetrating injury to his upper limb from a nail gun. Upon physical examination, the patient is unable to flex the distal interphalangeal joints of digits 4 and 5. What is the most likely cause of his injury? A. Trauma to the ulnar nerve near the trochlea B. Trauma to the ulnar nerve at the wrist C. Median nerve damage proximal to the pronator teres D. Median nerve damage at the wrist E. Trauma to spinal nerve root C8

A. The nail was fired explosively from the nail gun and then pierced the ulnar nerve near the coronoid process of the ulna and the trochlea of the humerus. Paralysis of the medial half of the flexor digitorum profundus muscle would result (among other significant deficits), with loss of flexion of the distal interphalangeal joints of digits 4 and 5. Ulnar trauma at the wrist would not affect the interphalangeal joints, although it would cause paralysis of interossei, hypothenar muscles, and so on. Median nerve damage proximal to the pronator teres would affect proximal interphalangeal joint flexion and distal interphalangeal joint flexion of digits 2 and 3 as well as thumb flexion. Median nerve injury at the wrist would cause loss of thenar muscles but not long flexors of the fingers. Trauma to spinal nerve ventral ramus C8 would affect all long finger flexors. GAS 784, 814; N 434; McM 149

40. A 29-year-old woman had sustained a deep laceration in the proximal part of the forearm. After the wound is closed, the following functional deficits are observed by the neurologist on service: the first three digits are in a position of extension and cannot be flexed; digits 4 and 5 are partially flexed at the metacarpophalangeal (MCP) joints and noticeably more flexed at the distal interphalangeal joints; sensation is absent in the lateral side of the palm and the palmar surfaces of digits 1 to 3 and half of digit 4. Which of the following nerve(s) has (have) most likely been injured? A. Median nerve B. Ulnar and median nerves C. Ulnar nerve D. Radial and ulnar nerves E. Radial nerve

A. The patient exhibits the classic "benediction attitude" of the thumb and fingers from injury to the median nerve proximally in the forearm. The thumb is somewhat extended (radial supplied abductor and extensors unopposed); digits 2 and 3 are extended (by intact interossei); digits 4 and 5 are partially flexed (by their intact flexor digitorum profundus). A lesion of the median nerve would result in weakened flexion of the proximal interphalangeal joints of all digits (flexor digitorum superficialis muscle), loss of flexion of the interphalangeal joint of the thumb, the distal interphalangeal joints of digits 2 and 3 (flexor digitorum profundus muscle), and weakened flexion of the metacarpophalangeal joints of the second and third digits (first and second lumbricals). A lesion of both the ulnar and median nerves would cause weakness or paralysis of flexion of all of the digits. A lesion of the ulnar nerve would mostly cause weakness in flexion of the DIP of the fourth and fifth digits and would affect all of the interosseous muscles and the lumbricals of the third and fourth digits. A lesion of the radial nerve would cause weakness in extension of the wrist, thumb, and metacarpophalangeal joints. GAS 784, 817; N 434; McM 149

B. The obturator nerve is responsible for innervation of the thigh adductors which form the medial compartment of the thigh. The femoral nerve innervates the anterior compartment and is responsible for extension of the knee. Common fibular (peroneal) nerve supplies the anterior and lateral compartments of the leg while the tibial nerve supplies the posterior compartments of the leg and thigh. The common fibular (peroneal) and tibial nerves are branches of the sciatic nerve.

After a revascularization procedure involving the common iliac artery, a 68-year-old man has difficulty walking. Nerve conduction studies reveal decreased activity in the nerve that innervates the adductors of the thigh. Which nerve is this? A. Femoral B. Obturator C. Common fibular (peroneal) D. Tibial E. Sciatic

32 A 15-year-old girl was brought to the emergency department with a tear of the tendons in the first dorsal compartment of the wrist from a severe bite by a pit bull dog. The injured tendons in this compartment would include which of the following muscles? A. Extensor carpi radialis longus and brevis B. Abductor pollicis longus and extensor pollicis brevis C. Extensor digitorum D. Extensor indicis proprius E. Extensor carpi ulnaris

B. The abductor pollicis longus and extensor pollicis brevis muscles are the occupants of the first dorsal compartment of the wrist. The extensor carpi radialis longus and brevis are in the second compartment. The extensor digitorum is in the third compartment, as is the extensor indicis. The extensor carpi ulnaris is located in the sixth dorsal compartment.

175 A 30-year-old intoxicated man stumbled while descending stairs and fell on his outstretched and hyperextended hand. Since the fall (2 to 3 hours ago) he has had constant pain in his wrist, but over the past 30 minutes he has developed tingling and burning pain in his hand as well. Radiographs reveal fractures of both the radial and ulnar styloid processes, as well as dislocation of a carpal bone. Which of the following abnormal sensory and motor findings are most likely to be found on examination? A. Dysesthesia (tingling in response to light touch) along the medial border of the hand and little finger and weakness in adduction of the thumb B. Dysesthesia over the palm and palmar aspect of the thumb, index, and middle fingers and weakness in thumb opposition C. Numbness along the medial border of the hand and little finger and weakness in wrist extension D. Numbness over the dorsum of the hand laterally including the dorsal aspect of the thumb, index, and middle fingers and weakness in grip strength E. Numbness over the palm and palmar aspect of the thumb, index and middle fingers and weakness in adduction of the thumb

B. The lunate is the most commonly dislocated carpal bone. It helps to form the floor of the carpal arch. When it is dislocated, it is displaced into the carpal tunnel compressing the median nerve. The patient will then present with dysesthesia over the palm and palmar aspect of the thumb, index and middle fingers, and weakness in thumb opposition. The other options are not symptoms of injury solely to the median nerve. GAS 792-795; N 439; McM 122

183 A 55-year-old woman is admitted to the emergency department after a car crash. Radiographic examination of her hand reveals a fractured carpal bone, which lies in the floor of the anatomical snuffbox. Which bone is fractured? A. Triquetrum B. Scaphoid C. Capitate D. Hamate E. Trapezoid

B. The scaphoid is the most commonly fractured carpal bone as a result of the relationship with the styloid process of the radius in the distal forearm. When a person falls as described in this question, the scaphoid gets pushed against the styloid process, usually at the narrowest ("waist") part of the scaphoid and fractures as a result of the forces transmitted through the bones. GAS 797; N 454; McM 122

46 A 47-year-old woman's right breast exhibited characteristics of peau d'orange, that is, the skin resembled an orange peel. This condition is primarily a result of which of the following? A. Shortening of the suspensory ligaments by cancer in the axillary tail (of Spence) of the breast B. Blockage of cutaneous lymphatic vessels C. Contraction of the retinacula cutis of the areola and nipple D. Invasion of the pectoralis major by the cancer E. Ipsilateral (same side) inversion of the periareolar skin from ductular cancer

B. When cutaneous lymphatics of the breast are blocked by cancer, the skin becomes edematous, except where hair follicles cause small indentations of the skin, giving an overall resemblance to orange peel. Shortening of the suspensory ligaments (of Cooper) or retinacula cutis leads to pitting of the overlying skin, pitting that is intensified if the patient raises her arm above her head. Invasion of the pectoralis major by cancer can result in fixation of the breast, seen upon elevation of the ipsilateral limb. Inversion of areolar skin with involvement of the ducts would also be due to involvement of the retinacula cutis. GAS 748; N 181; McM 179

19 A 35-year-old man has a small but painful tumor under the nail of his little finger. Which of the following nerves would have to be anesthetized for a painless removal of the tumor? A. Superficial radial B. Common palmar digital of median C. Common palmar digital of ulnar D. Deep radial E. Recurrent branch of median

C. The common palmar digital branch comes off the superficial branch of the ulnar nerve and supplies the skin of the little finger and the medial side of the ring finger. The superficial branch of the radial nerve provides cutaneous innervation to the radial (lateral) dorsum of the hand and the radial two and a half digits over the proximal phalanx. The common palmar digital branch of the median nerve innervates most of the lateral aspect of the palmar hand and the dorsal aspect of the second and third finger as well as the lateral part of the fourth digit. The deep radial nerve supplies the extensor carpi radialis brevis and supinator muscles and continues as the posterior interosseous nerve. The recurrent branch of the median nerve supplies the abductor pollicis brevis, flexor pollicis brevis, and opponens pollicis muscles.

54 Arthroscopic examination of the shoulder of a 62-year-old woman clearly demonstrated erosion of the tendon within the glenohumeral joint. What tendon was this? A. Glenohumeral B. Long head of triceps brachii C. Long head of biceps brachii D. Infraspinatus E. Coracobrachialis

C. The tendon of the long head of the biceps brachii muscle passes through the glenohumeral joint, surrounded by synovial membrane. The glenohumeral is a ligament that attaches to the glenoid labrum. The long head of the triceps brachii arises from the infraglenoid tubercle, beneath the glenoid fossa. The infraspinatus tendon passes posterior to the head of the humerus to insert on the greater tubercle. The coracobrachialis arises from the coracoid process and inserts on the humerus. GAS 707; N 417; McM 137

4 A 45-year-old man arrived at the emergency department with injuries to his left elbow after he fell in a bicycle race. Plain radiographic and magnetic resonance imaging (MRI) examinations show a fracture of the medial epicondyle and an injured ulnar nerve. Which of the following muscles will most likely be paralyzed? A. Flexor digitorum superficialis B. Biceps brachii C. Brachioradialis D. Flexor carpi ulnaris E. Supinator

D. Fracture of the medial epicondyle often causes damage to the ulnar nerve due to its position in the groove behind the epicondyle. The ulnar nerve innervates one and a half muscles in the forearm, the flexor carpi ulnaris and the medial half of the flexor digitorum profundus muscles. The nerve continues on to innervate most of the muscles in the hand. The flexor digitorum superficialis is innervated by the median nerve and the biceps brachii muscle by the musculocutaneous. The radial nerve innervates both the brachioradialis and supinator muscles.

11 A 22-year-old man is diagnosed with metastatic malignant melanoma of the skin over the xiphoid process. Which lymph nodes receive most of the lymph from this area and are therefore most likely to be involved in metastasis of the tumor? A. Deep inguinal B. Vertical group of superficial inguinal C. Horizontal group of superficial inguinal D. Axillary E. Deep and superficial inguinal

D. Lymph from the skin of the anterior chest wall primarily drains to the axillary lymph nodes.

62 A 23-year-old male basketball player is admitted to the hospital after injuring his shoulder during a game. Physical and radiographic examinations reveal total separation of the shoulder (Fig. 6-5). Which of the following structures has most likely been torn? A. Glenohumeral ligament B. Coracoacromial ligament C. Tendon of long head of biceps brachii D. Acromioclavicular ligament E. Transverse scapular ligament

D. The acromioclavicular ligament connects the clavicle to the coracoid process of the scapula. Separation of the shoulder (dislocation of the acromioclavicular [AC] joint) is associated with damage to the acromioclavicular ligament (capsule of the AC joint) and, in more severe injuries, disruption of the coracoclavicular ligaments (conoid and trapezoid portions). The glenohumeral ligament may be injured by an anterior dislocation of the humerus but is not likely to be injured by a separated shoulder. The coracoacromial ligament, transverse scapular ligament, and tendon of the long head of triceps brachii are not likely to be injured by separation of the shoulder. GAS 706; N 408; McM 136

13 A 22-year-old woman is admitted to the emergency department in an unconscious state. The nurse takes a radial pulse to determine the heart rate of the patient. This pulse is felt lateral to which tendon? A. Palmaris longus B. Flexor pollicis longus C. Flexor digitorum profundus D. Flexor carpi radialis E. Flexor digitorum superficialis

D. The location for palpation of the radial pulse is lateral to the tendon of the flexor carpi radialis, where the radial artery can be compressed against the distal radius. The radial pulse can also be felt in the anatomic snuffbox between the tendons of the extensor pollicis brevis and extensor pollicis longus muscles, where the radial artery can be compressed against the scaphoid.

177 A 29-year-old woman injures her wrist in a fall on an outstretched hand. Examination reveals pain on movement of the wrist associated with numbness and tingling on the radial side of the palm and palmar aspect of the thumb, index, and middle fingers. A radiograph of the wrist reveals, anterior dislocation of a carpal bone. Which dislocated carpal bone is compressing which structure? A. Pisiform compressing ulnar nerve B. Hook of hamate compressing ulnar artery C. Scaphoid compressing radial artery D. Lunate compressing median nerve E. Trapezoid bone compressing superficial radial nerve

D. The lunate is compressing the median nerve. The pisiform compressing the ulnar nerve is incorrect as the ulnar nerve innervates the skin on the medial one and a half digits. Hook of hamate compressing the ulnar artery is also incorrect. The hook of the hamate forms part of Guyon's canal; compression of the ulnar artery will not produce the deficits described because of the collateral circulation and anastomoses that exist with the radial artery. The scaphoid compressing the radial artery is also incorrect because there is collateral circulation from the palmar arches to compensate for radial artery occlusion. The trapezoid is not compressing the superficial radial nerve because the superficial branch of the radial nerve supplies the radial side and ball of the thumb and radial side of the index finger via its lateral and medial branches. GAS 792, 817; N 452; McM 168

1 A 45-year-old woman is being examined as a candidate for cosmetic breast surgery. The surgeon notes that both of her breasts sag considerably. Which structure(s) has most likely become stretched to result in this condition? A. Scarpa's fascia B. Pectoralis major muscle C. Pectoralis minor muscle D. Suspensory (Cooper's) ligaments E. Serratus anterior muscle

D. The suspensory ligaments of the breast, also known as Cooper's ligaments, are fibrous bands that run from the dermis of the skin to the deep layer of superficial fascia and are primary supports for the breasts against gravity. Ptosis of the breast is usually due to the stretching of these ligaments and can be repaired with plastic surgery. Scarpa's fascia is the deep membranous layer of superficial fascia of the anterior abdominal wall. The pectoralis major and pectoralis minor are muscles that move the upper limb and lie deep to the breast but do not provide any direct support structure to the breast. The serratus anterior muscle is involved in the movements of the scapula.

31 A 45-year-old man had fallen on his outstretched hand, resulting in Smith's fracture of the distal end of the radius. The fractured bone displaced a carpal bone in the palmar direction, resulting in nerve compression within the carpal tunnel. Which of the following carpal bones will most likely be dislocated? A. Scaphoid B. Trapezium C. Capitate D. Hamate E. Lunate

E. In Smith's fracture, the distal fragment of the radius deviates palmarward, often displacing the lunate bone. The other listed bones are unlikely to be displaced in a palmar direction by Smith's fracture.

174 A 24-year-old man sustained multiple injuries including complex fractures in the right wrist as the result of a motor vehicle collision. After his injuries were stabilized and surgical repairs planned, neurological testing reveals decreased sensation along the medial border of the hand, as well as the little and ring fingers, and decreased strength of thumb adduction and finger adduction/abduction but with intact flexion of the distal interphalangeal joints of the ring and little fingers. If the nerve injury is not repaired, which of the following will become apparent in the affected hand over the next few weeks to months? A. Flattening of the thenar eminence B. Wrist drop C. Radial deviation at the wrist D. Ulnar deviation at the metacarpophalangeal joints E. Prominent metacarpal bones with "guttering" between adjacent metacarpals

E. Loss of ulnar nerve innervations eventually leads to atrophy of the interossei muscles, which presents as guttering between the metacarpals. Since the median nerve is intact there is no thenar atrophy. Similarly, if the wrist extensors, which are supplied by radial nerve, are intact, then no wrist drop is observed. Radial deviation is not seen due to action of the extensor carpi ulnaris supplied by the radial nerve. GAS 814-816; N 464; McM 145

53 A 54-year-old male cotton farmer visits the outpatient clinic because of a penetrating injury to his forearm from a baling hook. After the limb is anesthetized, the site of the wound is opened and flushed thoroughly to remove all debris. The patient is not able to oppose the tip of the thumb to the tip of the index finger, as in making the OK sign. He is able to touch the tips of the ring and little fingers to the pad of his thumb. What nerve has most likely been injured? A. Median B. Posterior interosseous C. Radial D. Recurrent median E. Anterior interosseous

E. The anterior interosseous nerve is a branch of the median nerve that supplies the flexor pollicis longus, the lateral half of the flexor digitorum profundus, and the pronator quadratus muscles. If it is injured, flexion of the interphalangeal joint of the thumb will be compromised. The median nerve gives rise to the anterior interosseous nerve but is not a direct enough answer as injury to it would result in more widespread effects. The posterior interosseous nerve supplies extensors in the forearm, not flexors. The radial nerve gives rise to the posterior interosseous nerve and is not associated with the anterior interosseous nerve; therefore, it would not have any effect on the flexors of the forearm. The recurrent median nerve is also a branch of the median nerve but supplies the thenar eminence muscles, and its injury would result in problems with opposable motion of the thumb (GAS Fig. 7-87). GAS 784; N 463; McM 151

18 After a forceps delivery of an infant boy, the baby presents with his left upper limb adducted, internally rotated, and flexed at the wrist. The startle reflex is not seen on the ipsilateral side. Which part of the brachial plexus was most likely injured during this difficult delivery? A. Lateral cord B. Medial cord C. Ventral rami of the lower trunk D. Ventral ramus of the middle trunk E. Ventral rami of the upper trunk

E. The injury being described is also known as Erb-Duchenne paralysis or "waiter's tip hand" and is relatively common in children after a difficult delivery. This usually results from an injury to the upper trunk of the brachial plexus, presenting with loss of abduction, flexion, and lateral rotation of the arm. The superior trunk of the brachial plexus consists of spinal nerve ventral rami C5-6.

A: Only the femoral nerve innervates the quadriceps femoris. The sciatic nerve, specifically the tibial part, supplies the posterior thigh muscles not the anterior compartment. The obturator nerve supplies the medial compartment and the saphenous nerve does not supply any thigh muscles.

The quadriceps femoris muscle group is formed by the rectus femoris, vastus lateralis, vastus medialis, and vastus intermedius. Which of the following nerves innervates this group of muscles? A. Sciatic nerve B. Femoral nerve C. Obturator nerve D. Saphenous nerve E. Tibial nerve

A. The superior gluteal nerve innervates the gluteus medius, gluteus minimus, and tensor fasciae latae muscles. The tensor fasciae latae arises from the iliac crest, inserts into the iliotibial tract of the lateral aspect of the thigh, and assists in flexion of the hip. The rectus femoris is innervated by the femoral nerve; it flexes the hip and extends the knee, thus acting upon two major joints. It arises in part from the anterior inferior iliac spine and the rim of the acetabulum and inserts into the quadriceps tendon. The gluteus maximus is supplied by the inferior gluteal nerve. The piriformis and quadratus femoris are both short lateral rotators of the hip and are supplied by branches of the sacral plexus.

Three years following a 62-year-old's hip replacement, the man's CT scans indicated that two of his larger hip muscles had been replaced by adipose tissue. The opinion is offered that his superior gluteal nerve could have been injured during the replacement procedure, and the muscles supplied by that nerve had atrophied and been replaced by fat. Which of the following muscles receives its innervation from the superior gluteal nerve? A. Tensor fasciae latae B. Rectus femoris C. Gluteus maximus D. Piriformis E. Quadratus femoris

67 A 41-year-old woman is scheduled for a latissimus dorsi muscle flap to cosmetically augment the site of her absent left breast after mastectomy. Part of the latissimus dorsi muscle is advanced to the anterior thoracic wall, based upon arterial supply provided in part by the artery that passes through the triangular space of the axilla. Which artery forms the vascular base of this flap? A. Circumflex scapular artery B. Dorsal scapular artery C. Transverse cervical artery D. Lateral thoracic artery E. Thoracoacromial artery

67 A. The circumflex scapular artery passes through the triangular space after arising from the subscapular artery. It provides superficial branches to the overlying latissimus dorsi, whereas its deep portion passes into the infraspinous fossa to anastomose with the suprascapular artery. The dorsal scapular artery passes between the ventral rami of the brachial plexus and then deep to the medial border of the scapula. The transverse cervical artery arises from the thyrocervical trunk at the root of the neck and can provide origin for a dorsal scapular branch. The lateral thoracic and thoracoacromial arteries are branches of the second part of the axillary artery and provide no supply to the latissimus dorsi. GAS 721; N 414; McM 141

81 A 31-year-old female figure skater is examined in the emergency department following an injury that forced her to withdraw from competition. When her male partner missed catching her properly from an overhead position, he grasped her powerfully, but awkwardly, by the forearm. Clinical examination demonstrated a positive Ochsner test, inability to flex the distal interphalangeal joint of the index finger on clasping the hands. In addition, she is unable to flex the terminal phalanx of the thumb and has loss of sensation over the thenar half of the hand. What is the most likely nature of her injury? A. Median nerve injured within the cubital fossa B. Anterior interosseous nerve injury at the pronator teres C. Radial nerve injury at its entrance into the posterior forearm compartment D. Median nerve injury at the proximal skin crease of the wrist E. Ulnar nerve trauma halfway along the forearm

81 A. Because the median nerve is injured within the cubital fossa, the long flexors are paralyzed, including the flexor pollicis longus muscle. The flexor pollicis longus would not be paralyzed if the median nerve were injured at the wrist. Lateral palm sensory loss confirms median nerve injury. If only the anterior interosseous nerve were damaged, there would be no cutaneous sensory deficit. The radial nerve supplies wrist extensors, long thumb abductor, and metacarpophalangeal joint extensors. The ulnar nerve does not supply sensation to the lateral palm. GAS 768, 804; N 434; McM 149

92 A 67-year-old housepainter visits the outpatient clinic complaining that his hands are getting progressively worse, becoming more and more painful and losing their function. On physical examination of the hands, there is flexion of the metacarpophalangeal joints, extension of the proximal interphalangeal joints, and slight flexion of the distal interphalangeal joints. What is the most likely diagnosis? A. Mallet finger B. Boutonnière deformity C. Dupuytren's contracture D. Swan-neck deformity E. Silver fork wrist deformity

92 D. Swan-neck deformity involves slight flexion of the metacarpophalangeal (MCP) joints, hyperextension of the proximal interphalangeal (PIP) joints, and slight flexion of the distal interphalangeal (DIP) joints. This condition results most often from shortening of the tendons of intrinsic muscles, as in rheumatoid arthritis. When asked to straighten the injured finger, the patient is unable to do so and the curvature of the finger somewhat resembles the neck of a swan. GAS 795-796; N 451; McM 159

C. The fibularis (peroneus) brevis arises from the fibula and inserts upon the tuberosity at the base of the fifth metatarsal bone. Its attachment is often involved in an inversion fracture of the foot. This common fracture can often be overlooked when it is combined with an inversion sprain of the ankle. The fibularis (peroneus) longus arises from the fibula passes under the lateral malleolus, and then turns medially into the plantar surface of the foot, where it inserts upon the medial cuneiform and first metatarsal bones. The tibialis posterior arises from the tibia in the posterior compartment of the leg; it passes under the medial malleolus and inserts upon the navicular and metatarsal bones. The extensor digitorum brevis arises dorsally from the calcaneus and inserts upon the proximal phalanges of the lateral toes. The adductor hallucis arises from the lateral metatarsals and transverse tarsal ligament and inserts upon the proximal phalanx and lateral sesamoid bone of the big toe.

An 18-year-old professional tennis player fell when she leaped for an overhead shot and landed with her foot inverted. Radiologic examination in the hospital revealed an avulsion fracture of the tuberosity of the fifth metatarsal. Part of the tuberosity is pulled off, producing pain and edema. Which of the following muscles is pulling on the fractured fragment? A. Fibularis (peroneus) longus B. Tibialis posterior C. Fibularis (peroneus) brevis D. Extensor digitorum brevis E. Adductor hallucis

A. The perineal cutaneous branch of the posterior femoral cutaneous nerve provides a significant portion of the cutaneous innervation of the perineum in some individuals and can require separate anesthetic blockade in childbirth or perineal surgery, if other types of anesthesia are not used. The inferior cluneal branches of the posterior femoral cutaneous nerve supply the lower part of the gluteal skin. The lateral cutaneous branch of the iliohypogastric nerve provides sensation for the anterior superior aspect of the gluteal area. The inferior gluteal nerve innervates the gluteus maximus muscle. The middle cluneal nerves arise from the dorsal primary rami of S1 to S3 and supply skin over the middle of the gluteal region.

The baby was quite large, and the pelvis of the mother-to-be was somewhat narrow, causing her considerable difficulty and pain during the delivery. At her specific request, it was decided to inject local anesthetic into the perineum. The genitofemoral and ilioinguinal nerves were infiltrated anteriorly, and a deep injection was made medial to the ischial tuberosity to anesthetize the pudendal nerve, which supplies much of the perineum in most cases. A few minutes later, it became very obvious to those in attendance that the injection had not been effective enough in the central and posterior parts of the perineum. A separate injection was therefore inserted lateral to the ischial tuberosity. What other nerve(s) can provide much of the sensory supply to the perineum in some individuals? A. Posterior femoral cutaneous B. Inferior cluneal nerves C. Iliohypogastric nerve D. Inferior gluteal nerve E. Middle cluneal nerves

C. The tensor fasciae latae (which is innervated by the superior gluteal nerve) and the iliotibial tract are dense, wide aponeurosis that receives the insertion of the tensor fasciae latae and about 75% of the gluteus maximus. The gluteus maximus is the only one of the muscles listed that is supplied by the inferior gluteal nerve; in fact, it is the only muscle innervated by the inferior gluteal nerve. Gluteus medius and minimus insert on the greater trochanter and are innervated by the superior gluteal nerve. The rectus femoris, supplied by the femoral nerve, inserts via the quadriceps tendon on the patella and tibial tuberosity.

The neurosurgeon had removed a portion of the dense tissue (dura mater) covering the brain of the patient when she removed the tumor that had invaded the skull. To replace this important tissue covering of the brain, she took a band of the aponeurotic tissue of the lateral aspect of the thigh, covering the vastus lateralis muscle. What muscle, supplied by the inferior gluteal nerve, inserts into this band of dense tissue as part of its insertion? A. Gluteus medius B. Gluteus minimus C. Gluteus maximus D. Tensor fasciae latae E. Rectus femoris

101 A 22-year-old pregnant woman was admitted urgently to the hospital after her baby had begun to appear at the introitus. The baby had presented in the breech position, and it had been necessary to exert considerable traction to complete the delivery. The newborn is shown in Fig. 6-7. Which of the following structures was most likely injured by the trauma of childbirth? A. Radial nerve B. Upper trunk of the brachial plexus C. Lower trunk of the brachial plexus D. Median, ulnar, and radial nerves E. Upper and lower trunks of the brachial plexus

101 B. During a breech delivery as described here, downward traction is applied to the shoulders and upper limbs as the baby is forcibly extracted from the birth canal. This exerts traction on the upper cord of the brachial plexus, often causing a traction injury from which the baby can often recover. If the ventral rami of C5 and C6 are avulsed from the spinal cord, the injury is permanent. GAS 738, 747; N 416; McM 31

110 A 54-year-old woman presents with pain in her right wrist that resulted when she fell forcefully on her outstretched hand. Radiographic studies indicate an anterior dislocation of a carpal bone of the proximal row (see Fig. 6-4). Which of the following bones is most commonly dislocated? A. Capitate B. Lunate C. Scaphoid D. Pisiform E. Triquetrum

110 B. The lunate bone is the most commonly dislocated carpal bone. Displacement is almost always anteriorly. Dislocation of the lunate bone can precipitate the signs associated typically with carpal tunnel syndrome. GAS 793-795; N 443; McM 122

104 While sharpening his knife, a 23-year-old male soldier accidentally punctured the ventral side of the fifth digit at the base of the distal phalanx. The wound became infected, and within a few days the infection had spread into the palm, within the sheath of the flexor digitorum profundus tendons. If the infection were left untreated, into which of the following spaces could it most likely spread? A. Central compartment B. Hypothenar compartment C. Midpalmar space D. Thenar compartment E. Thenar space

104 C. The infectious agent was introduced into the synovial sheath of the long tendons of the little (fifth) finger. Proximally, this sheath runs through the midpalmar space, and inflammatory processes typically rupture into this space unless aggressively treated with the appropriate antibiotics. GAS 800-801; N 448; McM 158

108 A 55-year-old male firefighter is admitted to the hospital after blunt trauma to his right axilla. Examination reveals winging of the scapula and partial paralysis of the right side of the diaphragm. Which of the following parts of the brachial plexus have been injured? A. Cords B. Divisions C. Ventral rami D. Terminal branches E. Trunks

108 C. The winged scapula results from a lesion of the long thoracic nerve, which supplies the serratus anterior muscle. This muscle is responsible for rotating the scapula upward, which occurs during abduction of the arm above the horizontal. The long thoracic nerve arises from the ventral rami of C5 to C7 of the brachial plexus. The diaphragm is supplied by the phrenic nerve, which comes from the ventral rami of C3 to C5 (mnemonic: C3, 4 and 5 keep the diaphragm alive). GAS 727; N 413; McM 129

118 A 34-year-old woman is admitted to the emergency department after a car crash. Radiographic studies show marked edema and hematoma of the arm, but there are no fractures. During physical examination the patient presents with inability to abduct her arm without first establishing lateral momentum of the limb, and inability to flex the elbow and shoulder. Which of the following portions of the brachial plexus is most likely injured? A. Superior trunk B. Middle trunk C. Inferior trunk D. Lateral cord E. Medial cord

118 A. The superior trunk of the brachial plexus includes C5 and C6, which give rise to the suprascapular nerve, which innervates the supraspinatus muscle. The supraspinatus muscle is the primary muscle involved in abduction of the arm from 0 to 15 degrees. The deltoid muscle, supplied primarily by C5, abducts the arm from 15 to 90 degrees. The middle trunk is just C7 and has nothing to do with the muscle involved in initial abduction of the arm. The inferior trunk is C8-T1 and does not supply the supraspinatus muscle; therefore, it is not the right answer. The cords are distal to the branching of the supraspinatus muscle; therefore, neither lateral cord nor medial cord is the correct answer. GAS 738, 747; N 416; McM 28

128 A 41-year-old woman is admitted to the hospital after a car crash. Radiographic examination reveals a transverse fracture of the radius proximal to the attachment of the pronator teres muscle. The proximal portion of the radius is deviated laterally. Which of the following muscles will most likely be responsible for this deviation? A. Pronator teres B. Pronator quadratus C. Brachialis D. Supinator E. Brachioradialis

128 D. The supinator muscle attaches to the radius proximally and when fractured would cause a lateral deviation. The pronator teres muscle originates on the medial epicondyle and coronoid process of the ulna and inserts onto the middle of the lateral side of the radius, pulling the radius medially below the fracture. The pronator quadratus muscle originates on the anterior surface of the distal ulna and inserts on the anterior surface of the distal radius, pulling the radius medially. The brachioradialis muscle originates on the lateral supracondylar ridge of the humerus and inserts at the base of the radial styloid process, far below the fracture. The brachialis muscle originates in the lower anterior surface of the humerus and inserts in the coronoid process and ulnar tuberosity, hence not causing an action on the radius. GAS 777, 787-788; N 426; McM 121

129 A 45-year-old woman is bitten by a dog on the lateral side of her hand. Two days later the woman develops fever and swollen lymph nodes. Which of the following group of lymphatics will most likely be involved? A. Central B. Humeral C. Pectoral D. Subscapular E. Parasternal

129 A. Lymph from the lateral side of the hand drains directly into humeral (epitrochlear) nodes then to the central (axillary) nodes. Pectoral nodes receive lymph mainly from the anterior thoracic wall, including most of the breast. Subscapular nodes receive lymph from the posterior aspect of the thoracic wall and scapular region. Parasternal nodes receive lymph from the lower medial quadrant of the breast (GAS Fig. 7-57). GAS 748; N 403; McM 364

133 A 54-year-old woman is admitted to the hospital after falling from a tree with an outstretched hand. Radiographic examination reveals a wrist dislocation. Which of the following carpal bones will most likely be involved? A. Scaphoid-lunate B. Trapezoid-trapezium C. Hamate-lunate D. Pisiform-triquetrum E. Hamate-capitate

133 A. The hallmark fracture caused by a fall on an outstretched hand is a scaphoid-lunate fracture; the scaphoid and lunate are the two wrist bones most proximal to the styloid process of the radius. All the other wrist bones are less likely to be affected by this injury. GAS 793-794, 797; N 439; McM 122

145 An emergency department physician examines a patient who fell from a motorcycle and injured his shoulder. The clinician notices a loss of the normal contour of the shoulder and an abnormal-appearing depression below the acromion. Which of the following injuries did the patient most likely sustain? A. Avulsion of the coronoid process B. Dislocated shoulder joint C. Fracture of the midshaft of the humerus D. Fracture of the surgical neck of the humerus E. Laceration of the axillary branch of the posterior cord

145 B. The glenohumeral joint is an extremely mobile joint with a wide range of motion. Anterior dislocation of the humerus is most common and usually associated with an isolated traumatic incident. When the head of the humerus is displaced anteriorly and inferiorly, flattening of the deltoid prominence occurs, leading to loss of the normal contour of the humerus. There is protrusion of the acromion, and the slope of the shoulder lateral to the acromion is depressed and has a "dented" appearance. Avulsion of the coronoid process of the ulna usually occurs with elbow hyperextension, which affects the shoulder joint. A fracture of the midshaft of the humerus damages the radial nerve. Although a fracture to the surgical neck of the humerus and a laceration to the axillary part of the posterior cord affect the axillary nerve, which innervates the deltoid muscle, there will not be any depression beneath the acromion in either case. GAS 707; N 422, 424; McM 136

156 A 16-year-old girl is brought to the emergency department after attempting suicide by cutting her wrist. The deepest part of the wound is between the tendons of the flexor carpi radialis and the flexor digitorum superficialis. This patient is most likely to have a deficit of which of the following? A. Adduction and abduction of the fingers B. Extension of the index finger C. Flexion of the ring and little finger D. Sensation over the base of the little finger E. Opposition of the thumb and slightly weakened flexion of the second and third digits

156 E. Opposition, a complex movement, begins with the thumb in the extended position and initially involves abduction and medial rotation of the first metacarpal. This is produced by the action of the opponens pollicis muscle at the carpometacarpal joint by the flexor pollicis brevis muscle and then by flexion at the metacarpophalangeal joint. The opponens pollicis and flexor pollicis brevis muscles are supplied by the recurrent branch of the median nerve (C8, T1). The median nerve is the principal nerve of the anterior compartment of the forearm and the thenar muscles of the hand. It passes through the carpal tunnel with the tendons of the flexor digitorum profundus, flexor digitorum superficialis, and flexor pollicis longus to supply the thenar muscles of the hand. Abduction and adduction of the fingers is done by the palmar and dorsal interossei muscles, which are supplied by the median nerve. The extensor indicis extends the index finger. The ring and little fingers are flexed by the medial two tendons of the flexor digitorum superficialis (supplied by the median nerve) and the medial two tendons of the flexor digitorum profundus (supplied by the ulnar nerve). GAS 817; N 452; McM 159

166 A mother tugs violently on her child's arm to pull him out of the way of an oncoming car and the child screams in pain. The child is admitted to the emergency department and radiographic examination reveals a dislocated head of the radius resulting from the radial head slipping out past which ligament? A. Anular B. Joint capsule C. Interosseous membrane D. Radial collateral E. Ulnar collateral

166 A. "Nursemaid's elbow," a condition commonly found in children below 5 years of age, is caused by a sharp pull of the child's hand. In children, the anular ligament, which holds the head of the radius in place, is lax and allows the radial head to sublux when the hand is pulled. Also the radial head is small, so the anular ligament does not have a good "grip" on the hand. The joint capsule of the radioulnar joint is not attached to the radius; rather it passes around the neck of the radius inferiorly to attach to the coronoid process of the ulna. The interosseous membrane binds the radius and ulna together and does not maintain stability of the joint. The radial collateral ligament attaches the lateral side of the head of the radius to the lateral condyle of the humerus. The ulnar collateral ligament attaches the medial side of the ulnar head to the medial condyle of the humerus. GAS 766; N 424; McM 146

171 An 18-year-old man who is a professional cyclist complains of sensory loss to the medial one and a half fingers on the dorsal aspect of his hand. The orthopedic surgeon diagnoses "Handlebar neuropathy." What other signs may be elicited during physical examination? A. Sensory loss of the medial one and a half digits on the palmar aspect of the hand B. Weakness in abduction of the thumb C. Weakness in extension of the thumb D. Thenar muscle atrophy E. Tinel's sign at the scaphoid

171 A. Handlebar or ulnar neuropathy causes sensory loss of both palmar and dorsal aspects of the medial one and a half digits. Abduction of the thumb is by the abductor pollicis longus supplied by the radial nerve and the abductor pollicis brevis supplied by the median nerve. Extensors of the thumb are supplied by the radial nerve. Median nerve palsy can result in thenar muscle atrophy. A Tinel's sign might be observed near the hamate at Guyon's canal but not laterally at the scaphoid. GAS 814-816; N 464; McM 159

22 A 19-year-old man is brought to the emergency department after dislocating his shoulder while playing soccer. Following reduction of the dislocation, he has pain over the dorsal region of the shoulder and cannot abduct the arm normally. An MRI of the shoulder shows a torn muscle. Which of the following muscles is most likely to have been damaged by this injury? A. Coracobrachialis B. Long head of the triceps brachii C. Pectoralis minor D. Supraspinatus E. Teres major

22 D. The supraspinatus muscle is one of the four rotator cuff muscles—the other three being the infraspinatus, teres minor, and subscapularis muscles. The tendon of the supraspinatus muscle is relatively avascular and is often injured when the shoulder is dislocated. This muscle initiates abduction of the arm, and damage would impair this movement. The coracobrachialis muscle, which runs from the coracoid process to the humerus, functions in adduction and flexion of the arm. The main function of the triceps brachii muscle is to extend the elbow, and damage to its long head would not affect abduction. The pectoralis minor muscle functions as an accessory respiratory muscle and to stabilize the scapula and is not involved in abduction. The teres major muscle functions to adduct and medially rotate the arm.

23 A 47-year-old female tennis professional is informed by her physician that she has a rotator cuff injury that will require surgery. Her physician explains that over the years of play, a shoulder ligament has gradually caused severe damage to the underlying muscle. To which of the following ligaments is the physician most likely referring? A. Acromioclavicular ligament B. Coracohumeral ligament C. Transverse scapular ligament D. Glenohumeral ligament E. Coracoacromial ligament

23 E. The coracoacromial ligament contributes to the coracoacromial arch, preventing superior displacement of the head of the humerus. Because this ligament is very strong, it will rarely be damaged; instead, the ligament can cause inflammation or erosion of the tendon of the supraspinatus muscle as the tendon passes back and forth under the ligament. The acromioclavicular ligament, connecting the acromion with the lateral end of the clavicle, is not in contact with the supraspinatus tendon. The coracohumeral ligament is located too far anteriorly to impinge upon the supraspinatus tendon. The glenohumeral ligament is located deep to the rotator cuff muscles and would not contribute to injury of the supraspinatus muscle. The transverse scapular ligament crosses the scapular notch and is not in contact with the supraspinatus tendon.

69 A 74-year-old woman is admitted to the emergency department after stumbling over her pet dog. Radiographic examination reveals a fracture of the upper third of the right radius, with the distal fragment of the radius and hand pronated. The proximal end of the fractured radius deviates laterally. Which of the following muscles is primarily responsible for the lateral deviation? A. Pronator teres B. Supinator C. Pronator quadratus D. Brachioradialis E. Brachialis

69 B. The fracture line of the upper third of the radius lies between the bony attachments of the supinator and the pronator teres muscles. The distal radial fragment and hand are pronated due to unopposed contraction of pronator teres and pronator quadratus muscles. The proximal fragment deviates laterally by the unopposed contraction of the supinator muscle. The brachioradialis inserts distally on the radius. The brachialis inserts on the coronoid process of the ulna and would not be involved in the lateral deviation of the radius. GAS 772-774; N 431; McM 152

66 A 19-year-old man had suffered a deep laceration to an upper limb when he stumbled and fell on a broken bottle. On examination of hand function it is observed that he is able to extend the metacarpophalangeal joints of all his fingers in the affected limb. He cannot extend the interphalangeal (IP) joints of the fourth and fifth digits, and extension of the IP joints of the second and third digits is very weak. There is no apparent sensory deficit in the hand. Which of the following nerves has most likely been injured? A. Radial nerve at the elbow B. Median nerve at the wrist C. Ulnar nerve in midforearm D. Deep branch of ulnar nerve E. Recurrent branch of the median nerve

66 D. Injury to the deep branch of the ulnar nerve results in paralysis of all interosseous muscles and the lumbrical muscles of digits 4 and 5. Extension of the metacarpophalangeal joints is intact, a function of the radial nerve. Interphalangeal extension of digits 4 and 5 is absent, due to the loss of all interosseous muscle and the lumbricals of digits 4 and 5. Some weak interphalangeal joint extension is still present in digits 2 and 3 because the lumbricals of these two fingers are innervated by the median nerve. The radial nerve and the median nerve appear to be intact in this case. If the ulnar nerve were injured in the midforearm region, there would be sensory loss in the palm and digits 4 and 5 and on the dorsum of the hand. The recurrent branch of the median nerve supplies the thenar muscles; it does not supply lumbricals. Moreover, paralysis of this nerve would have no effect on the interphalangeal joints. GAS 814; N 464; McM 159

68 A 31-year-old male hockey player fell on his elbow and is admitted to the emergency department. Radiographic examination reveals a fracture of the surgical neck of the humerus, producing an elevation and adduction of the distal fragment. Which of the following muscles would most likely cause the adduction of the distal fragment? A. Brachialis B. Teres minor C. Pectoralis major D. Supraspinatus E. Pectoralis minor

68 C. The surgical neck of the humerus is a typical site of fractures. The fracture line lies above the insertions of the pectoralis major, teres major, and latissimus dorsi muscles. The supraspinatus muscle abducts the proximal fragment, whereas the distal fragment is elevated and adducted. The elevation results from contraction of the deltoid, biceps brachii, and coracobrachialis muscles. The adduction is due to the action of pectoralis major, teres major, and latissimus dorsi. GAS 705; N 413; McM 140

70 A 12-year-old boy lacerated the palmar surface of the wrist while playing with a sharp knife. The cut ends of a tendon could be seen within the wound in the exact midline of the wrist. Which tendon lies in this position in most people? A. Palmaris longus B. Flexor carpi radialis C. Abductor pollicis longus D. Flexor carpi ulnaris E. Flexor pollicis longus

70 A. The palmaris longus passes along the midline of the flexor surface of the forearm. The flexor carpi radialis is seen in the lateral portion of the forearm superficially, passing over the trapezium to insert at the base of the second metacarpal. The abductor pollicis longus tendon is laterally located in the wrist, where it helps form the lateral border of the anatomic snuffbox. The flexor carpi ulnaris tendon can be seen and palpated on the medial side of the wrist ventrally. The flexor pollicis longus tendon passes deep through the carpal tunnel. GAS 777; N 432; McM 158

71 A 22-year-old male medical student was seen in the emergency department with a complaint of pain in his right hand. He confessed that he had hit a vending machine in the hospital when he did not receive his soft drink after inserting money twice. The medial side of the dorsum of the hand was quite swollen, and one of his knuckles could not be seen when he "made a fist." The physician made a diagnosis of a "boxer's fracture." What was the nature of the impatient student's injury? A. Fracture of the styloid process of the ulna B. Fracture of the neck of the fifth metacarpal C. Colles' fracture of the radius D. Smith's fracture of the radius E. Bennett's fracture of the thumb

71 B. The student had broken the neck of the fifth metacarpal when hitting the machine with his fist. This is the more common type of "boxer's fracture." Neither a fracture of the ulnar styloid nor a Colles' fracture nor Smith's fracture of the distal radius would present with the absence of a knuckle as observed here. Bennett's fracture involves dislocation of the carpometacarpal joint of the thumb. Indications are that the injury is on the medial side of the hand, not the wrist, nor the lateral side of the hand or wrist. GAS 793-794; N 440; McM 167

80 It was reported by the sports media that the outstanding 27-year-old shortstop for a New York team would miss a number of baseball games. He was hit on a fingertip while attempting to catch a ball barehanded. A tendon had been torn. The team doctor commented that the ballplayer could not straighten the last joint of the long finger of his right hand, and the finger would require surgery. From what injury did the ballplayer suffer? A. Claw hand deformity B. Boutonnière deformity C. Swan-neck deformity D. Dupuytren's contracture E. Mallet finger

80 E. The extensor tendons of the fingers insert distally on the distal phalanx of each digit. If the tendon is avulsed, or the proximal part of the distal phalanx is detached, the distal interphalangeal joint (DIP) is pulled into total flexion by the unopposed flexor digitorum profundus muscle. This result gives the digit the appearance of a mallet. In boutonnière deformity, the central portion of the extensor tendon expansion is torn over the proximal interphalangeal (PIP) joint, allowing the tendon to move palmarward, causing the tendon to act as a flexor of the PIP joint. This causes the DIP joint to be hyperextended. Swanneck deformity involves slight flexion of the metacarpophalangeal (MCP) joints, hyperextension of PIP joints, and slight flexion of DIP joints. This condition results most often from shortening of the tendons of intrinsic muscles, as in rheumatoid arthritis. Dupuytren's contracture results from connective tissue disorder in the palm, usually causing irreversible flexion of digits 4 and 5. Claw hand occurs with lesions to the median and ulnar nerves at the wrist. In this clinical problem all intrinsic muscles are paralyzed, including the extensors of the interphalangeal joints. The MCP joint extensors, supplied by the radial nerve, and the long flexors of the fingers, supplied more proximally in the forearm by the median and ulnar nerves, are intact and are unopposed, pulling the fingers into the "claw" appearance. GAS 787-790, 802; N 451; McM 153

84 A 54-year-old woman was found unconscious on the floor, apparently after a fall. She was admitted to the hospital, and during physical examination it was observed that she had unilateral absence of her brachioradialis reflex. Which spinal nerve is primarily responsible for this reflex in the majority of cases? A. C5 B. C6 C. C7 D. C8 E. T1

84 B. The C6 spinal nerve is primarily responsible for the brachioradialis reflex. C5 and C6 are both involved in the biceps brachii reflex; C5 for motor, C6 for the sensory part of the reflex arc; C7 is the key spinal nerve in the triceps reflex. GAS 755; N 432; McM 143

90 An 18-year-old man suffered a significant laceration through the skin and underlying tissues at the distal crease of the wrist. The medical student rotating through the emergency department suspected (correctly) that the ulnar nerve was cut completely through at this location. Which of the following would most likely occur? A. The patient could not touch the tip of the thumb to the tips of the other digits B. There would be loss of sensation on the dorsum of the medial side of the hand C. The patient would be unable to flex the interphalangeal joints D. There would be decreased ability to extend the interphalangeal joints E. There would be no serious functional problem at all to the patient

90 D. The interossei are the most important muscles in extension of the interphalangeal (IP) joints because of the manner of their insertion into the extensor expansion of the fingers, which passes dorsal to the transverse axes of these joints. The lumbrical muscles assist in IP extension, in addition to flexing the metacarpophalangeal joints. Ulnar nerve injury at the wrist results in paralysis of all the interossei and the medial two lumbricals. Extensors of the metacarpophalangeal (MCP) joints are innervated by the deep radial nerve. Unopposed extension of the MCP joints causes them to be held in extension whereas unopposed long flexors of the fingers (supplied by median and ulnar nerves proximally in the forearm) cause them to be flexed into the "claw" position. The lumbricals of digits two and three are still intact because they are supplied by the median nerve, so clawing is not seen as much on these digits. Loss of opposition would result from median or recurrent nerve paralysis. If the ulnar nerve is cut at the wrist, its dorsal cutaneous branch to the dorsum of the hand is unaffected. GAS 808-809; N 451; McM 159

A. The sartorius is indicated by the arrow in Figure 5-3. This muscle forms the roof of the subsartorial canal (Hunter's canal), with the adductor longus and vastus medialis forming other muscular borders. The femoral artery and vein, the saphenous nerve, the nerve to the vastus medialis, and the medial cutaneous nerve of the thigh all pass into this canal. The femoral artery leaves the canal by passing through the hiatus of the adductor magnus. The saphenous nerve emerges from the canal and from beneath the sartorius on the medial side of the lower limb proximally, thereafter providing sensory branches to the medial side of the lower limb and foot. Dr. Hunter mobilized the sartorius, thereby exposing the femoral artery (which continues as the popliteal artery beyond the adductor hiatus), which could be clamped while an aneurysmal popliteal artery was treated surgically.

A popliteal arterial aneurysm can be very fragile, bursting with great loss of blood and the potential loss of the leg if it is not dealt with safely and effectively. In the 18th century, Dr. John Hunter (1728-1793) discovered that if a primary artery of the thigh is temporarily compressed, blood flow in the popliteal artery can be reduced long enough to treat the aneurysm in the popliteal fossa surgically, with safety. What structure is indicated in Fig. 5-3 that is related to his surgical procedure? A. Sartorius B. Femoral vein C. Femoral artery D. Gracilis E. Adductor brevis

6 An 18-year-old man is brought to the emergency department after an injury while playing rugby. Imaging reveals a transverse fracture of the humerus about 1 inch proximal to the epicondyles. Which nerve is most frequently injured by the jagged edges of the broken bone at this location? A. Axillary B. Median C. Musculocutaneous D. Radial E. Ulnar

B. A supracondylar fracture often results in injury to the median nerve. The course of the median nerve is anterolateral, and at the elbow it lies medial to the brachial artery on the brachialis muscle. The axillary nerve passes posteriorly through the quadrangular space, accompanied by the posterior circumflex humeral artery, and winds around the surgical neck of the humerus. Injury to the surgical neck may damage the axillary nerve. The musculocutaneous nerve pierces the coracobrachialis muscle and descends between the biceps brachii and brachialis muscle. It continues into the forearm as the lateral antebrachial cutaneous nerve. The ulnar nerve descends behind the medial epicondyle in its groove and is easily injured and produces "funny bone" symptoms.

58 A 22-year-old male football player suffered a wrist injury while falling with force on his outstretched hand. When the anatomic snuffbox is exposed in surgery, which artery is visualized crossing the fractured bone that provides a floor for this space? A. Ulnar B. Radial C. Anterior interosseous D. Posterior interosseous E. Deep palmar arch

B. As the radial artery passes from the ventral surface of the wrist to the dorsum, it crosses through the anatomic snuffbox, passing over the scaphoid bone. The ulnar artery at the wrist is located on the medial side of the wrist, passing from beneath the flexor carpi ulnaris to reach Guyon's canal between the pisiform bone and the flexor retinaculum. Guyon's canal is adjacent to but not in communication with the carpal tunnel. The anterior interosseous and posterior interosseous arteries arise from the common interosseous branch of the ulnar artery and pass proximal to distal in the forearm between the radius and ulna, in the flexor and extensor compartments, respectively. The deep palmar branch of the ulnar artery passes between the two heads of the adductor pollicis to anastomose with the radial artery in the palm (GAS Fig. 7-86). GAS 810-814; N 454; McM 166

64 A 43-year-old woman visits the outpatient clinic with a neurologic problem. Diagnostically, she cannot hold a piece of paper between her thumb and the lateral side of her index finger without flexing the distal joint of her thumb. This is a positive Froment's sign, which is consistent with ulnar neuropathy. Weakness of which specific muscle causes this sign to appear? A. Flexor pollicis longus B. Adductor pollicis C. Flexor digiti minimi D. Flexor carpi radialis E. Extensor indicis

B. Froment's sign is positive for ulnar nerve palsy. More specifically it tests the action of the adductor pollicis muscle. The patient is asked to hold a sheet of paper between the thumb and a flat palm. The flexor pollicis longus is innervated by the anterior interosseous branch of the median nerve. The flexor digiti minimi is innervated by the deep branch of the ulnar nerve and would not be used to hold a sheet of paper between the thumb and palm. The flexor carpi radialis is innervated by the median nerve, and the extensor indicis is innervated by the radial nerve (Fig. 6-14). GAS 814-816, 826; N 464; McM 157

182 A 25-year-old woman experiences numbness and tingling in her right arm and hand while carrying a piece of luggage. Physical examination showed no motor or sensory deficits in the upper limb. When asked to abduct her upper limb to 90 degrees and to maintain this position while repeatedly closing and opening her hands, the symptoms are reproduced along the medial border of the limb, from the axilla to the hand. Which nerve structure(s) is/are most likely compressed? A. Ulnar nerve at the medial epicondyle B. Radial nerve at the neck of the radius C. Median nerve in the carpal tunnel D. Inferior trunk of the brachial plexus E. Divisions of the brachial plexus

D. Compression on the inferior trunk of the brachial plexus compresses nerves C8 and T1. These nerves contribute to the medial cutaneous nerve of the arm (C8, T1) and the medial cutaneous nerve of the forearm (C8, T1). They also contribute to the median, medial pectoral, ulnar, and radial nerves. This patient has thoracic outlet syndrome, which causes compression of the inferior trunk of the brachial plexus usually by the presence of a cervical rib. Compression of the ulnar nerve at the medial epicondyle, radial nerve at the neck of the radius, or median nerve in the carpal tunnel would cause motor deficits not present in this patient. GAS 738, 747; N 416; McM 129

B. This type of injury can result in the "unhappy triad" (of O'Donoghue) injury, with damage to the medial collateral ligament (MCL), anterior cruciate ligament (ACL), and medial meniscus. A blow to the lateral side of the knee stretches and tears the MCL, which is attached to the medial meniscus. The ACL is tensed during knee extension and can tear subsequent to the rupture of the MCL. The remaining answer choices describe structures on the lateral surface of the knee, which are not usually injured by this type of trauma.

During a football game a 21-year-old wide receiver was illegally blocked by a linebacker, who threw himself against the posterolateral aspect of the runner's left knee. As he lay on the ground, the wide receiver grasped his knee in obvious pain. Which of the following structures is frequently subject to injury from this type of force against the knee? A. Fibular collateral ligament B. Anterior cruciate ligament C. Lateral meniscus and posterior cruciate ligament D. Fibular collateral and posterior cruciate ligament E. All the ligaments of the knee will be affected

B. The rough bony projection at the junction of the inferior end of the body of the ischium and its ramus is the large ischial tuberosity. Much of the body's weight rests on these tuberosities when sitting, and it provides the proximal, tendinous attachment of the posterior thigh muscles (hamstring muscles and adductor magnus). The hamstring muscles are associated with hip extension and knee flexion. The adduction of the hip joint will be affected slightly because the adductor magnus is affected, although the rest of the adductor muscles are intact.

During a gymnastic session, a 24-year-old woman suddenly developed pain and swelling on the right buttock. This happened following a forceful thigh movement. There is severe weakness of right hip extension and knee flexion. Adduction of the thigh is also slightly weak. An avulsion fracture of the ischial tuberosity is found on a radiograph. Which of the following group of muscles has most likely involved in this process? A. Adductor brevis, adductor longus, adductor magnus, pectineus, and gracilis B. Biceps femoris, semimembranosus, semitendinosus, and adductor magnus C. Iliacus and psoas major D. Gluteus medius and gluteus minimus E. Gluteus maximus and adductor magnus F. Iliacus, psoas major, rectus femoris and sartorius

A. The Achilles tendon inserts upon the calcaneus bone. This tendon represents a combination of the tendons of gastrocnemius and soleus muscles. The tendon of the plantaris can insert with this tendon.

In an accident during cleanup of an old residential area of the city, the Achilles tendon of a 32-year-old worker was cut through by the blade of a brush cutter. The patient is admitted to the hospital and a laceration of the Achilles tendon is diagnosed. Which of the following bones serves as an insertion for the Achilles tendon? A. Calcaneus B. Fibula C. Cuboid D. Talus E. Navicular

D. The tibialis posterior tendon is the most anterior of the structures that pass under the laciniate ligament (flexor retinaculum) on the medial side of the ankle to enter the sole of the foot. Increases of pressure within the tissues of the plantar aspect of the foot, usually due to increased fluid from hemorrhage, inflammatory processes, or infections, cause tarsal tunnel syndrome, comparable to carpal tunnel syndrome of the hand. The plantar aponeurosis and other fibrous and osseous tissues of the plantar surface cause this area to be relatively nondistensible; therefore, it takes little increase of fluid content to result in pressures adequate to restrict venous drainage and, thereafter, arterial inflow to the region. Fasciotomy of the medial skin and fascia of the foot and the posterior compartment of the leg can be required to reduce the pressure and allow healing to take place. The structures that pass beneath the flexor retinaculum are, from anterior to posterior: Tendon of tibialis posterior; tendon of flexor Digitorum longus; posterior tibial Vessels and Nerve; tendon of flexor Hallucis longus. (This is the basis of the mnemonic: "Tom, Dick, and a Very Nervous Harry.") Neither the plantaris tendon nor the tibialis anterior tendon pass through this canal.

The swollen and painful left foot of a 23-year-old female long distance runner is examined in the university orthopedic clinic. She states that she stepped on an unseen sharp object while running through the park several days earlier. Emergency surgery is ordered to deal with her tarsal tunnel syndrome. The tarsal tunnel is occupied normally by tendons, vessels, and nerves that pass beneath a very strong band of tissue (flexor retinaculum) on the medial side of the ankle. What is the most anterior of the structures that pass through this tunnel? A. Flexor hallucis longus tendon B. Plantaris tendon C. Tibialis anterior tendon D. Tibialis posterior tendon E. Tibial nerve

C. The deep fibular (peroneal) nerve supplies the fibularis (peroneus) tertius muscle. Although its name might lead one to think that this muscle is in the lateral compartment with the other two fibularis (peroneus) muscles, it is in the anterior (extensor) compartment of the leg. It is named for its origin from the fibula. It inserts upon the dorsum of the base of the fifth (or fourth) metatarsal bone and assists in extension and eversion of the foot. The sural nerve is a cutaneous nerve, formed by contributions from the tibial and common fibular (peroneal) nerves; it supplies the posterior lateral leg and the lateral side of the foot. The lateral plantar nerve is a branch of the tibial nerve; it innervates the quadratus plantae, muscles of the little toe, the adductor hallucis, lumbricals 2 to 4, and all of the interossei. It is sensory to the lateral side of the sole and the lateral three and a half digits. The superficial fibular (peroneal) nerve supplies the fibularis (peroneus) longus and brevis and innervates the skin on most of the dorsum of the foot. The tibial nerve supplies the calf muscles and divides into the medial and lateral plantar nerves.

Young parents were concerned that their 14-month-old daughter had not yet begun walking. Their pediatrician reassured them, saying that one of the muscles of the leg, the fibularis (peroneus) tertius, had to complete its central neurologic development before the child could lift the outer corner of the foot and walk without stumbling over her toes. What is the most common nerve supply of this muscle? A. Sural B. Lateral plantar C. Deep fibular (peroneal) D. Superficial fibular (peroneal) E. Tibial

102 A 17-year-old female student of martial arts entered the emergency department with a complaint of pain in her hand. Patient history reveals that she had been breaking concrete blocks with her hand. Examination reveals that the patient has weak abduction and adduction of her fingers but has no difficulty in flexing them. The patient also has decreased sensation over the palmar surfaces of the fourth and fifth digits. Which of the following best describes the nature of her injury? A. Compression of the median nerve in the carpal tunnel B. Fracture of the triquetrum, with injury to the dorsal ulnar nerve C. Dislocation of a bone in the proximal row of the carpus D. Fracture of the shaft of the fifth metacarpal E. Injury of the ulnar nerve in Guyon's canal

102 E. Striking the concrete blocks with the medial side of her hand has injured the ulnar nerve in Guyon's canal. This is the triangular tunnel formed by the pisiform bone medially, the flexor retinaculum dorsally, and the deep fascia of the wrist ventrally. This injury would result in loss of sensation to the medial palm and the palmar surface of the medial one and a half digits and motor loss of the hypothenar muscles, the interossei, and the medial two lumbricals. The median nerve is not involved because the thenar muscles and lateral palmar sensations are intact. The dorsal ulnar nerve arises proximal to the wrist, thus it would not be lost. Carpal bone dislocation is unlikely. If the lunate bone were dislocated, it would not cause compression of the ulnar nerve at the wrist. There is no indication of fifth metacarpal fracture, the so-called boxer's fracture. GAS 784, 814; N 449; McM 159

105 A 36-year-old man is admitted to the emergency department with a dull ache in his shoulder and axilla (Fig. 6-8). During physical examination the pain worsens by activity, and, conversely, rest and elevation relieve the pain. History reveals that the patient was hospitalized 2 days ago and a central venous line was used. What is the most likely diagnosis? A. Axillary-subclavian vein thrombosis B. Compression of C5 to C8 spinal nerve C. Disc herniation of C4 to C8 D. Impingement syndrome E. Injury to radial, ulnar, and median nerves

105 A. Axillary-subclavian vein thrombosis is becoming much more common in recent years because of the extensive use of catheters in cancer patients and other chronic medical conditions. Effort-induced thrombosis is seen with strenuous use of the dominant arm with hyperabduction and external rotation of the arm or backward and downward rotation of the shoulder as in playing cricket, volleyball, or baseball, or chopping wood. Because the symptoms of subclavian stenosis are fairly dramatic, most patients present promptly, usually within 24 hours. They complain of a dull ache in the shoulder and axilla, the pain worsened by activity. Conversely, rest and elevation often relieve the pain. Patients with catheter-associated axillary-subclavian deep vein thrombosis report similar symptoms at the arm or shoulder on the side with the indwelling catheter. GAS 759; N 420; McM 206

106 A 22-year-old woman had suffered a severe knife wound to the upper lateral portion of her pectoral region, with entry of the knife at the deltopectoral groove. Pressure applied to the wound had prevented further profuse bleeding. In the emergency department, vascular clamps were applied to the axillary artery, proximal and distal to the site of injury, which had occurred between the second and third parts of the axillary artery. The vascular surgeon knew there was time to repair the wound of the artery because of the rich collateral pathway provided by the anastomoses between which of the following arteries? A. Transverse cervical and suprascapular B. Posterior circumflex humeral and profunda brachii C. Suprascapular and circumflex scapular D. Supreme (superior) thoracic and thoracoacromial E. Lateral thoracic and suprascapular

106 C. The injury is at the second part of the axillary artery. The suprascapular artery is a branch of the thyrocervical trunk off the subclavian artery, proximal to the axillary artery. The subscapular artery is the major branch of the third part of the axillary artery, giving off the thoracodorsal and the circumflex scapular arteries. In this case blood would be flowing from the circumflex scapular artery in a retrograde direction into the axillary artery, supplying blood distal to the injury. GAS 733-735; N 420; McM 134

107 In a penetrating wound to the forearm of a 24-year-old man, the median nerve is injured at the entrance of the nerve into the forearm. Which of the following would most likely be apparent when the patient's hand is relaxed? A. The MCP and IP joints of the second and third digits of the hand will be in a condition of extension. B. The third and fourth digits will be held in a slightly flexed position. C. The thumb will be flexed and slightly abducted. D. The first, second, and third digits will be held in a slightly flexed position. E. The MCP and IP joints of the second and third digits of the hand will be in a condition of flexion.

107 A. This proximal injury to the median nerve would paralyze all of the long flexors of the digits, except for the muscle that flexes the distal interphalangeal joints of digits 4 and 5, thereby swinging the "balance of power" to the muscles that extend the digits, all of which are innervated by the radial nerve. The intrinsic hand muscles can aid in flexion of the metacarpophalangeal joints, and are innervated by the ulnar nerve. However, they are of insufficient size to compensate for the extensor forces exerted on fingers. GAS 742-746; N 463; McM 148

111 A 32-year-old man who is an expert target shooter reports pain in his right upper limb and slight tingling and numbness of all digits of the ipsilateral hand. However, the tingling and numbness of the fourth and fifth digits is the most severe. The man states that the problem usually occurs when he is firing his gun with his hand overhead. Radiographic studies reveal the presence of a cervical rib and accessory scalene musculature. Which of the following structures is most likely being compressed? A. Axillary artery B. Upper trunk of brachial plexus C. Subclavian artery D. Lower trunk of brachial plexus E. Brachiocephalic artery and lower trunk of brachial plexus

111 D. A cervical rib (usually found at C7) may cause thoracic outlet syndrome, which is a condition characterized by weak muscle tone in the hand and loss of radial pulse when the upper limb is abducted above the shoulder. The mechanism of injury with the gun being fired overhead suggests a lower trunk injury to the brachial plexus. The axillary artery supplies the shoulder muscles, and there is no loss of function to these muscles. The upper trunk of the brachial plexus also supplies innervation to the shoulder muscles, which are unaffected based on the patient's presenting abnormalities. The subclavian artery is located anterior to the brachial plexus until the plexus separates into cords as it passes under the clavicle. The brachiocephalic artery and lower trunk of the brachial plexus is only partially correct; the brachiocephalic artery is not directly associated with the brachial plexus due to its location at the midline of the body behind the sternum. GAS 150; N 183, 416; McM 129

112 A 23-year-old woman arrives at the emergency department with a swollen, painful forearm. An MRI examination reveals a compartment syndrome originating at the interosseous membrane between the radius and ulna. Which of the following type of joint will most likely be affected? A. Synarthrosis B. Symphysis C. Synchondrosis D. Trochoid E. Ginglymus

112 A. A synarthrosis joint is a fibrous connection that allows minimal to no movement. In this case, virtually no movement is allowed by the interosseous membrane joint between the radius and ulna. Symphysis joints are permanent fibrocartilaginous fusions between two bones; pubic symphysis is an example. Synchondrosis is a temporary joint made of cartilage that transitions to bone typically after growth completes (i.e., epiphyseal plate). Trochoid joints are pivot joints, and the humeral-radial portion of the elbow joint is an example. Ginglymus joints are hinge joints located at the interphalangeal junctions in the hand and foot (PIPs and DIPs). GAS 18-20, 774-775; N 425; McM 146

113 While working out with weights, a 28-year-old woman experiences a severe pain in her chest. The pain is referred to the anterior chest wall, radiating to the mandible and her left arm. The woman felt dizzy and after 10 minutes she collapsed and was unconscious. A physician happened to be near the woman and immediately tried to feel her radial pulse. The radial artery lies between two tendons near the wrist, which are useful landmarks. Which of the following is the correct pair of tendons? A. Flexor carpi radialis and palmaris longus B. Flexor carpi radialis and brachioradialis C. Brachioradialis and flexor pollicis longus D. Flexor pollicis longus and flexor digitorum superficialis E. Flexor pollicis longus and flexor digitorum profundus

113 B. The radial pulse is best located on the forearm (antebrachium) just proximal to the wrist joint. At this point the radial artery travels on the distal radius between the flexor carpi radialis and brachioradialis tendons. The palmaris longus tendon travels more medially to the radial artery and above the flexor retinaculum. The flexor pollicis longus tendon is a deeper structure in the antebrachium and is also located medially to the radial artery. GAS 827; N 432; McM 161

114 A 59-year-old woman is admitted to the hospital in a state of shock. During physical examination, several lacerations are noted in her forearm and her radial pulse is absent. Where is the most typical place to identify the radial artery immediately after crossing the radiocarpal joint? A. Between the two heads of the first dorsal interosseous muscle B. In the anatomic snuffbox C. Below the tendon of the flexor pollicis longus D. Between the first and second dorsal interossei muscles E. Between the first dorsal interosseous muscle and the adductor pollicis longus

114 B. The radial artery enters the palm through the anatomic snuffbox. The artery then moves on to pierce through the two heads of the first dorsal interosseous muscle and enter the deep aspect of the palm. The flexor pollicis longus tendon runs on the palmar aspect of the hand and the radial artery runs on the dorsal aspect of the hand before entering the deep aspect of the palm, and therefore the radial artery does not run below this tendon. The radial artery does not run between the first and second interosseous muscle and therefore cannot be used as a landmark to identify the artery. Finally, the artery does not run between the first dorsal interosseous muscle and the adductor pollicis longus. GAS 800, 812; N 454; McM 161

120 A 56-year-old woman is admitted to the hospital after a severe car crash. A large portion of her chest wall needed to be surgically removed and replaced with a musculo-osseous scapular graft involving the medial border of the scapula. Which of the following arteries will most likely recompensate the blood supply to the entire scapula? A. Suprascapular B. Dorsal scapular artery C. Posterior circumflex humeral artery D. Lateral thoracic E. Supreme thoracic artery

120 A. The suprascapular artery arises as a major branch of the thyrocervical trunk from the subclavian artery. It has rich anastomoses with the circumflex scapular artery and could provide essential blood supply to the scapula. The dorsal scapular artery would be lost with the graft. None of the other vessels listed is in position to provide adequate supply to the scapula. GAS 720; N 414; McM 31

115 A 69-year-old woman visits the outpatient clinic with a complaint of numbness and tingling of her hand for the past 3 months. Physical examination reveals she has numbness and pain in the lateral three digits of her right hand that are relieved by vigorous shaking of the wrist. In addition, the abductor pollicis brevis, opponens pollicis, and the first two lumbrical muscles are weakened. Sensation was decreased over the lateral palm and the volar aspect of the first three digits. Which of the following nerves is most likely compressed? A. Ulnar B. Radial C. Recurrent median D. Median E. Posterior interosseous

115 D. The median nerve provides innervation to most of the muscles in the flexor compartment of the forearm; cutaneous innervation of the second, third, and fourth digits and palmar and dorsum aspects of the hand; and innervation of four intrinsic hand muscles: first and second lumbricals, abductor pollicis brevis, opponens pollicis, and flexor pollicis brevis. The thenar compartment contains the abductor pollicis brevis, opponens pollicis, and flexor pollicis brevis muscles, and these muscles are innervated by the recurrent branch of the median nerve. The patient has weakening of the first two lumbricals and not simply the thenar muscles, so the median nerve is most likely to be compressed. Another indication that the median nerve is compressed is the vigorous shaking of the wrist. Because the median nerve traverses the carpal tunnel, carpal tunnel compression could lead to this action on part of the patient. The ulnar nerve provides innervation for part of the flexor digitorum profundus and flexor carpi ulnaris muscles. These muscles are not weakened in this patient. The radial nerve provides cutaneous supply to the dorsum of the hand and forearm as well as extensor muscles of the forearm. The posterior interosseous nerve is a branch of the radial nerve and provides innervation of the extensor muscles in the forearm. GAS 745, 784; N 463; McM 157

116 A 32-year-old man is admitted to the emergency department after a severe car crash. Radiographic examination reveals multiple fractures of his right upper limb. A surgical procedure is performed and metallic plates are attached to various bony fragments to restore the anatomy. Five months postoperatively the patient visits the outpatient clinic. Upon physical examination the patient can abduct his arm and extend the forearm, and the sensation of the forearm and hand is intact; however, hand grasp is very weak, and he cannot extend his wrist against gravity. Which of the following nerves was most likely injured during the surgical procedure? A. Posterior cord of the brachial plexus B. Radial nerve at the distal third of the humerus C. Radial and ulnar D. Radial, ulnar, and median E. Radial and musculocutaneous

116 B. The patient can extend his forearm, which suggests that the triceps brachii muscle is not weakened. Supination appears to be weak along with hand grasp and wrist drop. This would indicate that part of the radial nerve has been lost below the innervation of the triceps brachii and above the branches to the supinator and extensors in the forearm. However, sensation on the forearm and hand is intact, indicating that the superficial branch of the radial nerve is intact. The superficial branch of the radial nerve separates from the deep radial nerve at the distal third of the humerus. The posterior cord of the brachial plexus is responsible for providing innervation of the axially and radial nerves. This patient does have some radial nerve innervation and no loss of axillary nerve function. The patient does not have weakened adduction of the wrist, indicating that the ulnar nerve is not injured. If both the radial and musculocutaneous nerves are injured, supination would not be possible as the supinator and biceps brachii muscles provide supination of the forearm. GAS 761-763, 785; N 465; McM 143

117 A 52-year-old man is admitted to the emergency department after falling on wet pavement. Radiographic examination reveals fracture of the radius. An MRI study reveals a hematoma between the fractured radius and supinator muscle. Upon physical examination the patient has weakened abduction of the thumb and extension of the metacarpophalangeal joints of the fingers. Which of the following nerves is most likely affected? A. Anterior interosseous B. Posterior interosseous C. Radial nerve D. Deep branch of ulnar nerve E. Median nerve

117 B. The posterior interosseous nerve is an extension of the deep branch of the radial nerve after it emerges distal to the supinator. It is responsible for innervation of several muscles in the extensor compartment of the posterior aspect of the forearm, including extension of the metacarpophalangeal joints. The deep radial nerve courses laterally around the radius and passes between the two heads of the supinator muscle and is thus likely to be compressed by a hematoma between the fractured radius and the supinator muscle. Though the radial nerve gives rise to the posterior interosseous nerve, this answer choice is too general and would not indicate the precise injured branch of the radial nerve. Both the deep branch of the ulnar nerve and the median nerve traverse the medial and anteromedial aspect of the arm, respectively. These nerves primarily supply the flexor compartment of the arm. The anterior interosseous nerve is a branch of the median nerve and supplies the flexor digitorum profundus, flexor pollicis longus, and the pronator quadratus muscles (GAS Fig. 7-90). GAS 785, 792; N 466; McM 152

119 A 22-year-old man is admitted to the hospital after a car collision. Radiographic examination reveals an oblique fracture of his humerus. Upon physical examination the patient is unable to extend his forearm. The damaged nerve was most likely composed of fibers from which of the following spinal levels? A. C5, C6 B. C5, C6, C7 C. C5, C6, C7, C8, T1 D. C6, C7, C8, T1 E. C7, C8, T1

119 C. The radial nerve acts to extend the forearm at the elbow. This nerve is derived from all the ventral rami of the brachial plexus C5 to T1. None of the other answers includes all the ventral rami and are therefore incorrect. GAS 745-746; N 416; McM 96

136 A 54-year-old woman is found unconscious in her car. She is admitted to the hospital, and during physical examination her biceps brachii reflex is absent. What is the spinal level of the afferent component of this reflex? A. C5 B. C6 C. C7 D. C8 E. T1

136 B. The biceps brachii reflex is elicited by tapping on the tendon of the biceps near its insertion on the radius. The biceps brachii reflex involves C5 and C6 spinal nerves. C5 provides the motor component; C6 the afferent side of the reflex arc. GAS 731-732; N 417; McM 148

121 A 56-year-old woman visits the emergency department after falling on a wet pavement. Radiographic examination reveals osteoporosis and a Colles' fracture. Which of the following carpal bones are often fractured or dislocated with a Colles' fracture? A. Triquetrum and scaphoid B. Triquetrum and lunate C. Scaphoid and lunate D. Triquetrum, lunate, and scaphoid E. Triquetrum and pisiform

121 C. The scaphoid and lunate carpal bones have a direct articulation with the radius, which is fractured in a Colles' fracture; therefore, they would most likely be disrupted or fractured. The other carpal bones listed do not have direct contact with the radius and have a more distal location; therefore, they would not be as likely to be injured with a Colles' fracture. GAS 774; N 439; McM 123

122 A 3-year-old girl is admitted to the emergency department with severe pain. History taking reveals that the girl was violently lifted by her raised arm by her mother to prevent the girl from walking in front of a moving car. Which of the following is most likely the cause of the pain? A. Compression of the median nerve B. Separation of the head of the radius from its articulation with the trochlea of the humerus C. Separation of the head of the radius from its articulation with the ulna and the capitulum of the humerus D. Separation of the ulna from its articulation with the trochlea of the humerus E. Stretching of the radial nerve as it passes behind the medial epicondyle of the humerus

122 C. This type of dislocation is common in children and results when the radius is dislocated and slips out from the anular ligament, which holds it in place, articulating with the ulna and the capitulum of the humerus. In adults the anular ligament has a good "grip" at the radial neck, but in young children the radial head is not fully developed, leading to an indistinct neck. Compression of the median nerve is not likely due to its medial position in the cubital fossa. The radius does not articulate with the trochlea of the humerus; the ulna articulates at this position. The ulna is not likely to be dislocated because it is more stable than the radius, which has only the anular ligament for its support. The radial nerve does not pass behind the medial epicondyle; rather, the ulnar nerve does this, so this is not the correct answer. GAS 766-768, 775; N 424; McM 146

123 A 61-year-old man was hit by a cricket bat in the midhumeral region of his left arm. Physical examination reveals normal elbow motion; however, he could not extend his wrist or his metacarpophalangeal joints and he reported a loss of sensation on a small area of skin on the dorsum of the hand proximal to the first two digits. Radiographic examination reveals a hairline fracture of the shaft of the humerus just distal to its midpoint. Which of the following nerves is most likely injured? A. Median B. Ulnar C. Radial D. Musculocutaneous E. Axillary

123 C. Injury to the radial nerve can be caused by a blow to the midhumeral region because the nerve winds around the shaft of the humerus. The symptoms described include the loss of wrist and finger extension and a loss of sensation in an area of skin supplied by the radial nerve. GAS 763; N 465; McM 144

124 A 34-year-old man is admitted to the hospital after a car collision. Radiographic examination reveals a fracture at his wrist. Physical examination reveals paralysis of the muscles that act to extend the interphalangeal joints (Fig. 6-9). Which of the following nerves is most likely injured? A. Ulnar B. Recurrent branch of median C. Radial D. Musculocutaneous E. Anterior interosseous

124 A. The ulnar nerve innervates the dorsal and palmar interossei, which act to abduct and adduct the fingers and assist the lumbricals in their actions of flexing the metacarpophalangeal joints and extending the interphalangeal joints. The recurrent branch of the median nerve innervates the thenar muscle group that functions in the movement of the thumb. The radial and musculocutaneous nerves do not innervate any muscles in the hand. The anterior interosseous innervates the flexor pollicis longus and the pronator quadratus. GAS 808-809; N 464; McM 159

125 A 45-year-old woman is admitted to the hospital with neck pain. An MRI examination reveals a herniated disc in the cervical region. Physical examination reveals weak triceps brachii muscle. Which of the following spinal nerves is most likely injured? A. C5 B. C6 C. C7 D. C8 E. T1

125 C. The triceps brachii muscle is innervated by the radial nerve (primarily C7), which comes off C5 to T1 spinal nerves. Because the patient's only motor deficit involves the triceps brachii muscles, one can rule out C5 and C6, which supply fibers to the axillary, musculocutaneous, and upper subscapular nerves. Damage to either of these ventral rami would result in additional motor deficits of the shoulder and flexor compartment of the arm. One can also rule out C8-T1 because these ventral rami form the medial pectoral nerve and the medial brachial and antebrachial cutaneous nerves. Damage to these ventral rami would result in loss of pectoral muscle function and cutaneous sensation over the medial surface of the upper limb. GAS 745, 761; N 416; McM 96

126 A 34-year-old woman is admitted to the hospital after a car collision. Physical examination reveals a mallet finger. Which of the following conditions is expected to be present during radiographic examination? A. A lesion of the ulnar nerve at the distal flexor crease of the wrist B. A separation of the extension expansion over the middle interphalangeal joint C. Compression of the deep ulnar nerve by dislocation of the lunate bone D. Avulsion fracture of the dorsum of the distal phalanx E. Fracture of the fourth or fifth metacarpal bone

126 D. Mallet finger, also known as baseball finger, is a deformity in which the finger is permanently flexed at the distal interphalangeal joint due to avulsion of the insertion of the extensor tendon at the distal phalanx. GAS 802; N 443; McM 165

137 A 54-year-old woman is found unconscious in her bed. She is admitted to the hospital, and during physical examination she has absence of her brachioradialis reflex. The ventral ramus of which spinal nerve is responsible for this reflex? A. C5 B. C6 C. C7 D. C8 E. T1

137 B. The brachioradialis reflex is elicited by tapping the tendon of the brachioradialis muscle. The reflex involves spinal nerves C5, C6, and C7. The major contribution is from C6. GAS 785-787; N 432; McM 150

127 A 42-year-old woman is admitted to the hospital with injury to the upper (superior) trunk of the brachial plexus. The diagnosis is Erb-Duchenne palsy. Which of the following conditions is expected to be present during physical examination? A. Winged scapula B. Inability to laterally rotate the arm C. Paralysis of intrinsic muscles of the hand D. Paraesthesia in the medial aspect of the arm E. Loss of sensation in the dorsum of the hand

127 B. Injury to the superior trunk of the brachial plexus can damage nerve fibers going to the suprascapular, axillary, and musculocutaneous nerves. Damage to the suprascapular and axillary nerves causes impaired abduction and lateral rotation of the arm. Damage to the musculocutaneous nerve causes impaired flexion of the forearm. A winged scapula would be caused by damage to the long thoracic nerve. The long thoracic nerve is formed from spinal cord levels C5, C6, and C7, so the serratus anterior muscle would be weakened from the damage to C5 and C6, but the muscle would not be completely paralyzed. The intrinsic muscles of the hand are innervated by the ulnar nerve, which would most likely remain intact. Paraesthesia in the medial aspect of the arm would be caused by damage to the medial brachial cutaneous nerve (C8-T1; inferior trunk). Loss of sensation on the dorsum of the hand would be caused by damage to either the ulnar or radial nerves (C6 to T1). GAS 738, 747; N 416; McM 31

130 A 25-year-old woman is admitted to the emergency department after a car collision. Radiographic examination reveals a fracture at the spiral groove of the humerus. A cast is placed, and 3 days later she complains of severe pain over the length of her arm. During physical examination the arm appears swollen, pale, and cool. Radial pulse is absent, and any movement of the arm causes severe pain. Which of the following conditions will most likely characterize the findings of the physical examination? A. Venous thrombosis B. Thoracic outlet syndrome C. Compartment syndrome D. Raynaud's disease E. Injury of the radial nerve

130 C. Compartment syndrome is characterized by increased pressure within a confined space by a fascial compartment, which impairs blood supply, resulting in paleness and loss of pulses distal to the compartment. Venous thrombosis would not cause pain but could cause death from a pulmonary embolism if a thrombus (clot) broke free and became lodged in the pulmonary trunk. Thoracic outlet syndrome affects nerves in the brachial plexus and the subclavian artery and blood vessels between the neck and the axilla, far above the cast. Raynaud's disease affects blood flow to the limbs when they are exposed to temperature changes or stress. The fracture at the radial groove probably resulted in a radial nerve injury but would not be responsible for these symptoms. GAS 590, 763; N 432; McM 143

131 A 22-year-old woman is admitted to the hospital after falling from a tree. Radiographic examination reveals fractured pisiform and hamate bones. Which of the following nerves will most likely be injured? A. Median B. Recurrent median C. Radial D. Anterior interosseous E. Deep ulnar

131 E. The deep branch of the ulnar nerve arises at the level of the pisiform bone and passes between the pisiform and the hook of the hamate; hence the deep branch of the ulnar nerve is most likely to be injured in this patient. The median nerve enters the forearm between the humeral and ulnar heads of the pronator teres muscle then becomes superficial near the wrist. The recurrent branch of the median nerve branches off after the median nerve enters the palm through the carpal tunnel. The radial nerve divides into superficial and deep branches when it enters the cubital fossa. GAS 814; N 452; McM 162

132 A 43-year-old man visits the outpatient clinic with a painful shoulder. Physical examination reveals a painful arc syndrome due to supraspinatus tendinopathy. Which of the following conditions are expected to be present during physical examination as the patient abducts his arm? A. Painful abduction 0 to 15 degrees B. Painful abduction 0 to 140 degrees C. Painful abduction 70 to 140 degrees D. Painful abduction 15 to 140 degrees E. Painful abduction 40 to 140 degrees

132 A. The supraspinatus initiates abduction of the arm during the first 15 degrees of abduction; palpation of the tendon during this phase would result in pain from a tendinopathy of the supraspinatus. GAS 717; N 411; McM 132

134 A 62-year-old man is admitted to the emergency department after falling on wet pavement. Radiographic examination reveals a carpometacarpal fracture at the base of the thumb. What is the term applied to the described fracture? A. Colles' fracture B. Scaphoid fracture C. Bennett's fracture D. Smith's fracture E. Boxer's fracture

134 C. Bennett's fracture is a carpometacarpal fracture at the base of the thumb. Smith's fracture is also called a reverse Colles' fracture and is caused when the distal fragment of the radius angles forward. Colles' fracture is also called "silver fork deformity" because the distal fragment is displaced posteriorly. Boxer's fractures of the necks of metacarpal bones are fractures to the fingers. A scaphoid fracture would be indicated by pain in the anatomical snuffbox. GAS 793-796; N 439; McM 122

135 A 23-year-old woman is participating in a dry skislope competition. The woman is admitted to the emergency department after falling and catching her thumb in the matting. Radiographic and physical examinations reveal rupture of the ulnar collateral ligament of the metacarpophalangeal joint of the thumb. The thumb is extremely painful and an injection of lidocaine is given. What is the most likely diagnosis in this case? A. Gamekeeper's thumb B. Scaphoid fracture C. Bennett's fracture D. Smith's fracture E. Boxer's fracture

135 A. Interestingly, "gamekeeper's thumb" was a term coined to describe an injury common among Scottish gamekeepers who, it is said, killed small animals such as rabbits by breaking their necks between the ground and the gamekeeper's thumb and index finger. The resulting valgus force on the abducted metacarpophalangeal (MCP) joint caused injury to the ulnar collateral ligament. Today this injury is more commonly seen in skiers who land awkwardly with their hand braced on a ski pole, causing the valgus force on the thumb, as seen in this patient. Whereas the term "skier's thumb" is sometimes used, "gamekeeper's thumb" is still in common usage. Bennett's fracture is a fracture at the base of the metacarpal of the thumb. Scaphoid fracture occurs after a fall on an outstretched hand, involving the scaphoid and lunate bone. Colles' fracture is also called silver fork deformity because the distal fragment of the radius is displaced posteriorly. Boxer's fracture is a fracture of the necks of the second and third (and sometimes the fifth) metacarpals. Smith's fracture is also called a reverse Colles' fracture and is caused when the distal radius is fractured and the distal radial fragment is angled forward. GAS 793-796; N 442; McM 163

138 A 55-year-old woman is admitted to the emergency department after a car crash. Physical examination reveals severe pain in the flexor muscles of the forearm, fixed flexion position of the finger, and swelling, cyanosis, and anesthesia of the fingers. Which of the following is the most likely diagnosis? A. Colles' fracture B. Scaphoid fracture C. Bennett's fracture D. Volkmann's ischemic contracture E. Boxer's fracture

138 D. Volkmann's contracture is a flexion deformity of the fingers and sometimes the wrist from an ischemic necrosis of the forearm flexor muscles. Bennett's fracture is a fracture at the base of the metacarpal of the thumb. Scaphoid fracture occurs after a fall on an outstretched hand and involves the scaphoid and lunate bones. Colles' fracture is also called silver fork deformity because the distal fragment of the radius is displaced posteriorly. Boxer's fracture is a fracture of the necks of the second and third (and sometimes the fifth) metacarpals. Smith's fracture is also called a reverse Colles' fracture and is caused when the distal radius is fractured, with the radial fragment angled forward. GAS 774; N 432; McM 150

139 A 62-year-old man visits the outpatient clinic with pain after falling on his outstretched hand. Radiographic examination reveals a fracture of the pisiform bone and hematoma of the surrounding area. Which of the following nerves will most likely be affected? A. Ulnar B. Radial C. Median D. Deep ulnar E. Deep radial

139 D. The ulnar nerve enters the forearm by passing between the two heads of the flexor carpi ulnaris and descends between and innervates the flexor carpi ulnaris and flexor digitorum profundus (medial half) muscles. It enters the hand superficial to the flexor retinaculum and lateral to the pisiform bone, where it is vulnerable to damage and provides the deep ulnar branch. The deep branch of the radial nerve arises proximally in the forearm. GAS 784; N 464; McM 149

140 A 32-year-old woman visits the outpatient clinic after injuring her elbow falling from her bicycle. Physical examination reveals a "benediction attitude" of the hand with the index and long fingers extended and the ring and little fingers flexed. Which of the following is the most likely diagnosis? A. Injury to median and radial nerves B. Injury to median nerve C. Injury to radial and ulnar nerves D. Injury to ulnar nerve E. Injury to median ulnar and radial nerves

140 B. "Benediction attitude" of the hand with the index and long fingers straight and the ring and little fingers flexed is caused by an injury to the median nerve. The long flexors of the digits are supplied by the median nerve; the unopposed radial nerve and deep ulnar nerve supply the extensors of the digits 1-3, causing them to be in the extended position. Digits 4 and 5 are slightly flexed because the flexors of the proximal interphalangeal joints are supplied by the ulnar nerve. GAS 784, 417; N 463; McM 157

141 A 54-year-old man is admitted to the emergency department with severe chest pain. Electrocardiographic evaluation reveals a myocardial infarction. Due to the severity of the infarction, a coronary artery bypass surgery using a radial artery graft is proposed. Which of the following tests should be performed during physical examination before the bypass graft operation? A. Allen test B. Triceps reflex C. Tinel test D. Brachioradialis reflex E. Biceps reflex

141 A. The Allen test involves compression of the radial and ulnar arteries at the wrist with the fingers flexed tightly to move the blood out the palm. Pressure is then released on the radial and ulnar arteries successively to determine the degree of supply to the hand by either vessel and the patency of the anastomoses between them. The usefulness of the radial artery for bypass can thereby be assessed. If the the palm does not flush with blood when the radial artery is released, then the ulnar artery is not sufficient to supply the hand if the radial artery is harvested for a graft. The other tests have nothing to do with the patency of the radial artery. GAS 814; N 435; McM 160

142 A 34-year-old man visits the outpatient clinic with a painful upper limb after a fall onto a concrete floor. Physical examination reveals that the patient has weak abduction and adduction of his fingers but has no difficulty in flexing them. The patient also has decreased sensation over the palmar surface of the fourth and fifth fingers. Which of the following diagnoses is most likely? A. Compression of the median nerve in the carpal tunnel B. Injury of the radial nerve from fractured humerus in the radial tuberosity C. Compression of the median nerve as it passes between the two heads of the pronator teres D. Compression of the radial nerve from the supinator E. Injury of the ulnar nerve by a fractured pisiform

142 E. The ulnar nerve enters the hand superficial to the flexor retinaculum and lateral to the pisiform bone and innervates all the interossei via the deep branch. These muscles are responsible for adduction and abduction of the fingers. Flexion of the fingers is spared because the flexor digitorum superficialis and most of the flexor digitorum profundus are innervated by the median nerve, which is unaffected by this injury. Had the median nerve been compressed in the carpal tunnel, one would have difficulty with motion of the thumb as a result of a lack of innervation of the thenar muscles. An injury of the radial nerve in the arm results in extension deficit in the forearm and hand. GAS 814; N 452; McM 158

143 A 65-year-old man is admitted to the emergency department after falling on his outstretched hand. The patient complains of severe right shoulder pain. Upon physical examination, the patient holds his arm externally rotated and slightly abducted. There is also flattening and sensory loss over the right deltoid muscle. Which of the following is the most likely diagnosis? A. Anterior dislocation of the humerus B. Acromioclavicular joint subluxation C. Clavicular fracture D. Spiral fracture of the humeral midshaft E. Rotator cuff tear

143 A. The glenohumeral joint is an extremely mobile joint with a wide range of movement. Anterior dislocation is the most common. Anterior dislocations of the humerus usually follow injuries where abnormal force is applied to the shoulder while the arm is extended, abducted, and externally rotated. When the head of the humerus is displaced anteriorly and inferiorly, there is flattening of the deltoid prominence (due to the increased weight of the humerus pulling on the muscle), protrusion of the acromion, and anterior axillary fullness (due to the movement of the humeral head into this location). The most commonly injured nerve is the axillary nerve, which innervates the teres minor and deltoid and also provides cutaneous supply to the posterior arm and the skin overlying the deltoid muscle. Acromioclavicular joint subluxation typically results from a blow to the tip of the shoulder when the arm is at the side and slightly adducted. It produces swelling and superior displacement of the clavicle. It is not associated with specific major nerve injuries or sensory deficits. The clavicle is a commonly fractured bone typically after direct trauma. Most fractures occur in the middle third of the clavicle. There is local swelling and tenderness but rarely any neurovascular damage. A spiral humerus midshaft fracture may result from a fall on an outstretched hand. The radial nerve is commonly fractured as it runs in the radial groove. Rotator cuff tears usually occur when there is some degenerative injury to the tendons. The rotator cuff is made up of the subscapularis, supraspinatus, infraspinatus, and teres minor and tendons. GAS 707; N 422, 424; McM 136

144 A 4-year-old boy is brought to the emergency department after falling while holding hands and walking with his two parents. The boy cannot move his right upper extremity because any movement produces pain, and he holds it at his side with his elbow extended and forearm pronated. There are no visible hematomas or swelling. Which of the following structures is most likely injured in this patient? A. Anular ligament B. Biceps brachii tendon C. Interosseous membrane D. Radial collateral ligament E. Ulnar collateral ligament

144 A. The patient is experiencing radial head subluxation ("nursemaid's elbow"), the most common elbow injury in children. The injury often results from a sharp pull on the hand while the forearm is pronated and the elbow is extended. The underdevelopment of the radial head and the laxity of the anular ligament allows for the radial head to sublux (partially dislocate) from this cuff of tissue. This condition is extremely painful but can be easily treated with supination and compression of the elbow joint. Although it is uncommon for muscle tendons to rupture, the most common is the tendon of the long head of the biceps brachii. It produces a characteristic deformity when flexing the elbow: an extremely prominent bulge of unattached muscle belly called the "Popeye sign." The interosseous membrane is an expansive sheet of connective tissue that connects the radius and ulna at their midsection. It serves as an attachment site for the muscles of the forearm. The radial collateral ligament lies on the lateral side of the elbow joint reinforcing the radiohumeral joint. The ulnar collateral ligament lies on the medial side of the elbow joint reinforcing the ulnohumeral joint. GAS 764, 766; N 424; McM 146

146 A 22-year-old woman who is in training to become a phlebotomist is performing venipuncture on another student. She places the needle into the median cubital vein but is unable to withdraw blood. She quickly realizes that she passed the needle completely through the vein. Which of the following structures located deep to the median cubital vein has acted as a barrier and has prevented her from puncturing an artery? A. Flexor retinaculum B. Pronator teres muscle C. Bicipital aponeurosis D. Brachioradialis muscle E. Biceps brachii tendon

146 C. The median cubital vein is a superficial vein that lies on the biceps brachii aponeurosis. The biceps brachii aponeurosis, also known as lacertus fibrosus, is a flat sheet of connective tissue that fans out from the medial side of the biceps brachii tendon to blend with the deep fascia of the biceps brachii muscle. It reinforces the cubital fossa and protects the brachial artery, which runs beneath it. GAS 768-769; N 403; McM 148

147 A 21-year-old painter sustains a laceration on the anterior surface of his left wrist just distal to the skin fold crease. When he arrives at the emergency department, the physician extends the patient's wrist to determine the depth of the laceration and observes a broad, glistening white structure deep to the superficial fascia. The patient has no numbness or tingling of any of the fingers and is able to discriminate sharp/dull sensation in all of the fingers and palm of the hand. There is no loss of motion in any of the fingers or the hand, and grip strength is normal. Which structure is the physician most likely observing? A. Flexor retinaculum B. Flexor carpi ulnaris tendon C. Palmar skin D. Flexor digitorum superficialis tendons E. Flexor digitorum profundus tendons

147 A. The flexor retinaculum is a thick connective tissue ligament that spans the space between the medial and lateral sides of the base of the carpal tunnel. It protects and stabilizes the tendons that run beneath it. Damage to the flexor carpi ulnaris tendon, flexor digitorum superficialis tendons, and flexor digitorum profundus tendons result in functional losses in the hand. The palmar skin is loose connective tissue and does not have a shiny, glistening appearance (GAS Fig. 7-103). GAS 798; N 449; McM 158

148 While using a wood-carving gouge, a 34-year-old woman lacerates the proximal aspect of her palm from the base of the thumb across to the pisiform bone. Neurological examination reveals pronounced weakness in opposition of the thumb, with intact sensation in the hand. Which of the following injuries best accounts for her findings? A. Injury of the median nerve in the carpal tunnel B. Injury of the superficial palmar branch of the median nerve C. Injury of the recurrent and superficial branch of the median nerve D. Injury of the recurrent median nerve at the wrist E. Injury of the radial and ulnar nerves

148 D. The recurrent branch of the median nerve usually originates from the lateral side of the median nerve at the distal margin of the flexor retinaculum. It innervates the three thenar muscles: the opponens pollicis, flexor pollicis brevis, and abductor pollicis brevis muscles. Injury of the median nerve in the carpal tunnel, as well as injury of the recurrent and superficial branch of the median nerve, causes both sensory and motor deficits. Injury of the superficial palmar branch of the median nerve results in loss of sensation only of the palm. Injury to the radial and ulnar nerves results in a greater number of sensory and motor deficits in the distribution of theses nerves (GAS Fig. 7-103). GAS 817; N 463; McM 157

150 A 36-year-old man is brought to the emergency department because of a deep knife wound on the medial side of his distal forearm. He is unable to hold a piece of paper between his fingers and has lost sensation from the fifth digit and the medial side of the fourth digit. Which of the following nerves is most likely injured? A. Axillary B. Median C. Musculocutaneous D. Radial E. Ulnar

150 E. The ulnar nerve is responsible for cutaneous innervation to the medial one and a half digits and motor innervation to most of the intrinsic muscles of the hand including the interossei. The interossei muscles are responsible for adduction of the digits, which is the action that would be used to hold a piece of paper between the fingers. The median nerve supplies cutaneous innervation to the lateral three and a half fingers and the thenar eminence and lateral two lumbricals. These muscles function to oppose the thumb and flex the MP joints, respectively. The musculocutaneous nerve is responsible for innervation of the anterior compartment of the arm, and muscular nerve fibers of this nerve would not be damaged by a wound in the distal forearm. The radial nerve supplies the dorsum of the hand, with sensation and extension function of the forearm muscles, and damage will not lead to this array of symptoms (GAS Fig. 7-109). GAS 814, 815; N 464; McM 159

152 A 21-year-old woman who is an athlete dislocated her glenohumeral joint while playing soccer and the shoulder was reduced in the emergency department. However, after 1 week the physician noted that the woman had lost strength when she attempted internal rotation of her arm at the shoulder. This finding was most likely caused by a tear in which of the following muscles? A. Infraspinatus B. Pectoralis minor C. Subscapularis D. Supraspinatus E. Teres minor

152 C. Anterior dislocation of the humerus may damage the nerves located in the axilla or cause tears in the rotator cuff muscles. Internal rotation is the primary function of subscapularis muscle; with this being the only action impaired it is the most likely damaged muscle, probably as a result of injury to the upper and/or lower subscapular nerves that innervate this muscle. The infraspinatus and trees minor muscles are external rotators, and the supraspinatus muscle is the abductor of the arm from 0 to 15 degrees. The pectoralis major is a flexor, adductor, and medial rotator and would not likely be damaged during a shoulder dislocation. GAS 712; N 411; McM 136

153 A 29-year-old man presents with difficulty with fine motor control in his hand. A few weeks ago he fell from a ladder; as he was falling he reached out and grabbed a limb of a tree. Examination reveals a deficit in his ability to abduct and adduct his digits and inability to oppose his thumb on his right hand. Which of the following was most likely injured? A. Lower trunk of the brachial plexus B. Median nerve C. Musculocutaneous nerve D. Ulnar nerve E. Upper trunk of the brachial plexus

153 A. The nerve responsible for innervation of the interosseus muscles that are weakened in this patient is the deep branch of the ulnar nerve. Innervation of the muscles responsible for opposition of the thumb is via the recurrent branch of the median nerve. Both of these nerves are formed by the C8 and T1 ventral rami, which combine to form the inferior trunk of the brachial plexus. Damage to either the median or ulnar nerves would not produce both of these symptoms. Median nerve damage would involve all of the flexors of the wrist except the flexor carpi ulnaris and most digits except for the interphalangeal joints of the 4th and 5th fingers. It will also result in loss of function of the thumb entirely. Ulnar nerve damage will result in weakness of the medial half of flexor digitorum profundus (4th and 5th interphalangeal joint flexion), as well as the intrinsic muscles of the hand except for the lateral two lumbricals. GAS 814; N 416; McM 157

154 An emergency department physician evaluates a 28-year-old man who injured his hand in a knife fight. The physician notes that the ring and little fingers cannot be extended at the interphalangeal joints, and the patient cannot spread the fingers of his injured hand. Weakness of which of the following muscles is the major reason for the loss of interphalangeal extension of the medial two fingers? A. Dorsal interosseus muscles B. Extensor digitorum C. Lumbrical muscles D. Palmar interosseus muscles E. Extensor digiti minimi

154 C. The patient likely has damage to the ulnar nerve, which affected both the interossei and medial two lumbricals. The lumbricals extend the interphalangeal joints of the ring and little fingers, while the interossei are responsible for abduction and adduction of the digits. The dorsal interossei are responsible for abduction, while the palmar interossei are responsible for adduction of the digits. The extensor digit minimi is responsible for extension of the little finger only and if damaged will not affect the ring finger. If the extensor digitorum were damaged it would lead to weakness of all four digits, not only the ring and little fingers. GAS 814; N 488; McM 159

155 After a fall on her outstretched arm, a 72-year-old woman presents with elbow pain. Physical examination reveals a palpable defect over her biceps brachii tendon. Elbow flexion causes pain but does not limit active movement. Radiographs do not show fractures or dislocations. She is diagnosed with a biceps brachii tendon rupture. Which of the following muscles most likely allow the patient to continue to flex her elbow? A. Brachialis and brachioradialis B. Flexor carpi ulnaris and flexor carpi radialis C. Flexor digitorum superficialis and flexor digitorum profundus D. Pronator teres and supinator E. Triceps brachii and coracobrachialis

155 A. Flexion of the elbow is achieved by contraction of the biceps brachii, brachialis, and brachioradialis muscles. The brachialis muscle is the major flexor of the elbow joint and together with the brachioradialis will continue to achieve flexion if the biceps brachii is damaged. The flexor carpi ulnaris and radialis produce flexion of the wrist, and the flexor digitorum superficialis and profundus produce flexion of the digits at the metacarpophalangeal and interphalangeal joints, respectively. The pronator teres and supinator are responsible for pronation and supination, respectively. The coracobrachialis does not cross the elbow joint and acts only on the shoulder, while the triceps brachii is the elbow extensor. GAS 755; N 417; McM 150

157 A 36-year-old man presents to the emergency department with pain and tenderness in his right wrist after a fall on his outstretched hand two days ago. On examination there is tenderness on the lateral side of the wrist, just proximal to the base of the first metacarpal. What is the most likely diagnosis? A. Fracture of the first metacarpal B. Fracture of the trapezium C. Tenosynovitis of thumb extensors D. Fracture of the scaphoid E. First carpometacarpal joint arthritis

157 D. The scaphoid is the most frequently fractured carpal bone. Fracture often results from a fall on the palm when the hand is abducted, the fracture occurring across the narrow part ("waist") of the scaphoid. Pain occurs primarily on the lateral side of the wrist, especially during dorsiflexion and abduction of the hand. If the only blood supply to the scaphoid enters the bone distally, avascular necrosis (pathological death of bone resulting from inadequate blood supply) of the proximal fragment of the scaphoid may occur and produce degenerative joint disease of the wrist. Thumb metacarpal fractures are usually caused by an axial blow directed against the partially flexed metacarpal. Tenosynovitis is an infection of the digital synovial sheaths. Symptoms of tenosynovitis include pain, swelling, and difficulty moving the particular joint where the inflammation occurs. Carpometacarpal joint arthritis is a degenerative joint disease affecting the first carpometacarpal joint. GAS 79; N 488; McM 159

158 A 55-year-old man is admitted to hospital after blunt trauma at the junction of his neck and shoulder on the right side. Examination reveals winging of the scapula and partial paralysis of the right side of the diaphragm. Which part of the brachial plexus has been injured? A. Cords B. Divisions C. Ventral rami D. Terminal branches E. Trunks

158 C. The long thoracic nerve arises from the upper three ventral rami to the brachial plexus (C5 to C7) and supplies the serratus anterior, which protracts the scapula. The diaphragm is innervated by the phrenic nerve, which also arises from ventral rami (C3-C5). GAS 727, 741; N 413; McM 140

159 A 55-year-old right-handed woman presents to the clinic with a 1-week history of right elbow pain. The pain started after a long game of competitive tennis. The pain begins in the elbow and at times radiates into the forearm. Splinting of the elbow decreases the intensity of the pain. During physical examination of the elbow mild swelling and tenderness are noted over the lateral epicondyle. Which one of the following wrist movements, if carried out by the patient with a closed fist and against resistance, will most likely exacerbate the pain? A. Radial deviation B. Ulnar deviation C. Flexion D. Extension E. Flexion and ulnar deviation

159 D. The lateral epicondyle is the common extensor origin. Most of the extensor muscles of the forearm originate from this area. Putting those muscles in action will exacerbate pain on the lateral epicondyle, a condition nicknamed "tennis elbow." Radial and lateral deviations have no effect because the movement is at the wrist joint. Flexion exacerbates pain on the medial epicondyle if the patient has "golfer's elbow." GAS 752, 785, 768; N 427; McM 145

160 Following a difficult delivery, a 3-day-old infant girl showed limited movement of the right upper limb, with the arm adducted and internally/medially rotated, the forearm extended at the elbow and pronated, and the wrist slightly flexed. Tearing of fibers in which ventral rami of the brachial plexus best accounts for these symptoms? A. C5 and C6 B. C6 and C7 C. C7 and C8 D. C8 and T1 E. C5 to T1

160 A. Injuries to superior parts of the brachial plexus (C5-C6) usually result from an excessive increase in the angle between the neck and shoulder during a difficult delivery. Injury to the superior trunk of the plexus is apparent by the characteristic position of the limb ("waiter's position"), in which the limb hangs by the side in medial rotation. Injuries to the lower trunk of the brachial plexus (Klumpke paralysis) are much less common. These events injure the inferior trunk of the brachial plexus (C8 and T1) and may avulse the roots of the spinal nerves from the spinal cord. The short muscles of the hand are affected, and a claw hand results (GAS Fig. 7-52A). GAS 738; N 452; McM 159

161 A 48-year-old woman is seen in the orthopedic clinic with symptoms of carpal tunnel syndrome. This could result in weakening of which muscles? A. Dorsal and palmar interossei B. Lumbricals III and IV C. Thenar and lumbricals I and II D. Flexor digitorum superficialis and profundus E. Hypothenar

161 C. Carpal tunnel syndrome is a relatively common condition that causes pain, numbness, and a tingling sensation in the hand and fingers. Carpal tunnel syndrome is caused by compression of the median nerve, which supplies the thenar muscles and the first and second lumbricals. Dorsal and palmar interossei and the hypothenar muscles are supplied by the ulnar nerve. The flexor muscles of the forearm are supplied by the median nerve before it passes through the carpal tunnel. GAS 798; N 449; McM 159

162 A 24-year-old man complains of inability to button his shirt. Examination reveals that he can still grip a sheet of paper between his second and third fingers and there is no sensory deficit in the hand. Which nerve has been affected? A. Deep branch of ulnar B. Anterior interosseous C. Median D. Recurrent branch of median E. Deep branch of radial

162 D. Recurrent branch of the median is the correct answer. This nerve, which is a branch of the median nerve, is given off after the median nerve passes through the carpal tunnel. The nerve supplies the thenar muscles. The opponens pollicis muscle, which is part of the thenar muscle group, is used while buttoning a shirt, an action that requires thumb opposition. The deep branch of the ulnar nerve supplies motor innervations to all the intrinsic muscles of the hand except the lateral two lumbricals and sensation to the medial one and a half fingers on both the palmar and dorsal sides. The patient can still grip a paper between the second and third digits, a function largely performed by the interossei muscles, which are innervated by the deep branch of the ulnar nerve. The deep branch of the radial nerve is motor to the long extensors of the wrist and fingers. GAS 817; N 452; McM 157

163 A 22-year-old man accidentally smashes his hand through a window. He is cut across the entire length of the distal transverse crease on the anterior surface of the wrist. The cut is down to the surface of the flexor retinaculum but not into it. During physical examination which is one of the neuromuscular deficits that will be found? A. Weakened pronation of the forearm B. Inability to abduct the thumb C. Weakened flexion of thumb D. Weakened opposition of the thumb E. Inability to adduct the thumb

163 E. Inability to adduct the thumb is the correct answer because the ulnar nerve travels superficial to the flexor retinaculum and innervates the adductor pollicis muscle, which adducts the thumb. Pronation of the forearm is carried out by muscles innervated by the median nerve, and abduction of the thumb is performed by muscles innervated by the median and radial nerves. Flexion and opposition of the thumb are performed by muscles innervated by the median nerve and would not be injured, as the median nerve travels deep to the flexor retinaculum. GAS 814; N 452; McM 157

164 A 36-year-old woman is admitted to the emergency department after an athletic injury that has caused weakness in both lateral rotation and the initial 15 to 20 degrees of abduction of the arm. Which nerve was most probably injured? A. Lower subscapular B. Axillary C. Radial D. Suprascapular E. Upper subscapular

164 D. The supraspinatus is innervated by the suprascapular nerve (C5, C6) and the nerve continues through the spinoglenoid notch and innervates the infraspinatus. The supraspinatus initiates abduction of the arm up to the first 15 to 20 degrees. The subscapular nerve supplies the subscapularis and teres major muscles, which are medial rotators of the arm. The axillary nerve supplies the deltoid and teres minor muscles and also a patch of skin on the lateral side of the shoulder. The deltoid abducts the arm beyond 20 degrees, and the teres minor muscle, although a lateral rotator, does not abduct the arm. The radial nerve supplies muscles in the posterior compartments of the arm and forearm, which are extensors of the elbow, wrist, and fingers in that order. The upper subscapular nerve supplies the subscapularis, a medial rotator of the arm. GAS 717, 742; N 413; McM 138

165 A 35-year-old male carpenter suffered a deep cut to the tip of his thumb. Initially the redness, swelling, and pain were limited to the injured part of the thumb, but later the entire thumb and thenar eminence became inflamed. Which group of lymph nodes is the first to receive drainage from this injury? A. Posterior axillary B. Subclavian C. Lateral axillary D. Anterior axillary E. Central axillary

165 C. With the involvement of the thenar muscles, lymph drains initially to the epitrochlear nodes and then to the lateral (humeral) nodes. The posterior axillary nodes receive lymph from the upper back and shoulder. The subclavian nodes receive lymph from all the axillary nodes. The anterior axillary nodes (pectoral nodes) receive lymph from most of the breast and the upper side of the anterolateral chest wall. All anterior, lateral, posterior, and medial axillary nodes drain to the central axillary nodes. GAS 748; N 403; McM 364

167 A 62-year-old woman is seen in the outpatient clinic. A photograph of her hand is shown in Figure 6-10. A radiograph reveals a hairline fracture of the hamate at Guyon's canal. Which of the following will also be present during physical examination? A. Numbness and weakness of the little and ring fingers B. Wrist drop C. Atrophy of the thenar muscles D. Positive Tinel's test E. Trouble turning her forearm outward

167 A. The patient has a classic claw hand due to the damage of the deep branch of the ulnar nerve by the fractured hamate at Guyon's canal. This nerve supplies the intrinsic muscles of the hand except the lateral two lumbricals and the thenar muscles. This nerve also supplies cutaneous innervations to the medial one and a half fingers (ring and little fingers) in the palmar and dorsal sides. The lumbricals and interossei insert at the back of the fingers via the dorsal (extensor) hood. This hood extends from the metacarpophalangeal joint to the distal phalanx. Through this mechanism, the muscles flex the metacarpophalangeal joint and extend the interphalangeal joint. With damage to the deep branch of the ulnar nerve, this function is lost. The result is that there will be flexion of the interphalangeal joints and extension of the metacarpophalangeal joint, giving the appearance as shown in the photograph. GAS 816; N 464; McM 159

168 A 25-year-old woman experiences numbness and tingling in her right arm and hand while carrying a piece of luggage. Physical examination showed no motor or sensory deficits in the upper limb. When asked to abduct her upper limb to 90 degrees and to maintain this position while repeatedly closing and opening her hands, the symptoms are reproduced along the medial border of the limb, from the axilla to the hand. Which nerve structure(s) is/are most likely compressed? A. Ulnar nerve at the medial epicondyle B. Radial nerve at the neck of the radius C. Median nerve in the carpal tunnel D. Inferior trunk of the brachial plexus E. Divisions of the brachial plexus

168 D. In thoracic outlet syndrome—sometimes caused by a cervical rib or a cervical band—ventral rami or trunks of the brachial plexus can be compressed by these structures as they travel from the neck to the axilla. In this case the inferior trunk of the brachial plexus is being compressed by a cervical rib. The anterior division of the inferior trunk continues as the medial cord of the brachial plexus. The medial brachial cutaneous nerve (medial cutaneous nerve of the arm) and medial antebrachial cutaneous nerve (medial cutaneous nerve of the forearm) are branches of the medial cord of the plexus, with the ulnar nerve as its terminal branch. Additionally, there is medial cord contribution to the median nerve. Compression of the inferior cord of the brachial plexus therefore presents with numbness and paraesthesia on the medial part of the arm, forearm, and hand. GAS 150; N 415; McM 140

169 A 50-year-old woman reports to the physician that she is no longer able to play her viola as she used to because of "locking" of her index finger. During physical examination a snap is heard during passive extension of the finger and the consequent flexion of it. What is the most likely diagnosis of this condition? A. Tenovaginitis stenosans (trigger finger) B. Dupuytren's contracture C. Mallet finger D. Boutonniere deformity E. Boxer's fracture

169 A. Tenovaginitis stenosans occurs after swelling or nodular growth of the flexor tendon, which interferes with it gliding through the pulley and producing a snap or click on active extension or flexion. Mallet finger presents with permanent flexion of the distal phalanx from the lateral band of the extensor digitorum avulsion. Boutonnière deformity is due to avulsion of the central band of the extensor digitorum tendon, which presents as abnormal flexion of the middle phalanx and hyperextension of the distal phalanx. Boxer's fracture affects the metacarpals of the second and third digits commonly. Dupuytren's contracture is progressive fibrosis of the palmar aponeurosis and fascia leading to progressive shortening and thickening, eventually leading to permanent partial flexion of the metacarpophalangeal and proximal interphalangeal joints. GAS 802; N 448; McM 157

170 A 25-year-old woman fashion model has an unsightly lump on her wrist that is causing her great distress. She also reports a tingling sensation on the lateral three and a half digits of the palmar aspect of her hand. Her doctor uses his pen torch to illuminate the lump and then uses a syringe to drain its contents. What is the most likely diagnosis of this condition? A. Neurofibroma B. Ganglion cyst C. Chondroma D. Osteoma E. Osteophyte

170 B. Ganglion cysts are outpouchings of the joint capsule or tendons and may occur anywhere in the hand or feet. They contain synovial-like fluid and are pliable to touch. They commonly occur on the dorsum of the hand and may be surgically treated if necessary. The others are all solid tumors and cannot be drained. GAS 790, 800-801; N 440; McM 158

172 A 20-year-old man who is a racquetball player reports to the physician's office complaining that he is not able to grip his racquet during practice. During physical examination the physician notes that the patient has atrophy of the thenar eminence, inability to oppose the thumb, and difficulty in flexing the middle interphalangeal joints of the digits. What is the most likely diagnosis of this condition? A. Hypertrophy of the supinator B. Pronator syndrome C. Medial supracondylar fracture D. Tennis elbow E. Golfer's elbow

172 B. Pronator syndrome is due to damage of the median nerve as it passes between the two heads of a hypertrophied pronator teres muscle. It will present with loss of opposition, atrophy of the thenar muscles, and flexion difficulty of the digits and sensory loss of the lateral three and a half digits. Hypertrophy of the supinator muscle will affect the deep branch of the radial nerve that continues distally as the posterior interosseous nerve. A medial supracondylar fracture might affect the ulnar nerve. Tennis elbow affects only the common extensor muscle origin and will not cause flexor or opposition difficulties of the digits and thumb, respectively (GAS Fig. 7-83). GAS 777; N 463; McM 151

173 A 54-year-old man presents to his primary care physician complaining of weakness in his fingers. His attempt to make a ring between his thumb and index finger by bringing the tips together is shown in Figure 6-11. He is able to successfully hold a piece of paper between his thumb and index finger. Pronation and wrist flexion are weakened. Which of the following nerves is most likely affected? A. Ulnar nerve at Guyon's canal B. Median nerve in the carpal tunnel C. Anterior interosseous nerve beneath the ulnar head of pronator teres D. Posterior interosseous nerve beneath the supinator E. Median nerve beneath the bicipital aponeurosis

173 C. The anterior interosseous nerve runs distally and anterior to the interosseous membrane supplying the deep forearm flexors (except the ulnar part of the flexor digitorum profundus muscle, which sends tendons to the fourth and fifth fingers), it passes deep to and supplies the pronator quadratus muscle, hence the weakness in pronation and wrist flexion (GAS Fig. 7-87). GAS 784; N 463; McM 151

184 An 18-year-old man presents to the emergency department with a painful right shoulder after a fall while diving for a soccer ball. A radiograph of the shoulder is shown in Figure 6-12. Examination revealed pain on passive adduction of the right arm across the chest. Which ligamentous structures must have been stretched/torn resulting in this injury? A. Acromioclavicular joint capsule and coracoclavicular ligament B. Acromioclavicular joint capsule and coracoacromial ligament C. Sternoclavicular joint capsule and coracoacromial ligament D. Coracoclavicular ligament and transverse scapular ligament E. Coracoclavicular ligament and coracoacromial ligament

184 A. The acromioclavicular and coracoclavicular ligaments are critical to the stability of the shoulder. In particular, the coracoclavicular ligament provides much of the weight-bearing support for the upper limb on the clavicle. The acromioclavicular joint ligament attaches the acromion (of the scapula) to the clavicle and the coracoclavicular ligament attaches the coracoid process to the clavicle. Interruption of these ligaments would cause dislocation of the acromioclavicular joint as seen in the radiograph. The sternoclavicular joint exists between the manubrium and the proximal end of the clavicle and is unrelated to either the injury or the radiograph. The coracoacromial ligament extends between the acromion and the coracoid process of the scapula. The transverse scapular ligament lies above the suprascapular notch and converts it into a foramen through which the suprascapular nerve runs (GAS Fig. 7-24). GAS 706; N 408; McM 136

73 A 23-year-old woman had a painful injury to her hand in a dry ski-slope competition, in which she fell and caught her thumb in the matting. Radiographic and physical examinations reveal rupture of the ulnar collateral ligament of the metacarpophalangeal joint of the thumb. Lidocaine is injected into the area to relieve the pain, and she is scheduled for a surgical repair. From which of the following clinical problems is she suffering? A. De Quervain's syndrome B. Navicular bone fracture C. Boxer's thumb D. Gamekeeper's thumb E. Bennett's thumb

73 D. Interestingly, "gamekeeper's thumb" was a term coined because this injury was most commonly associated with Scottish gamekeepers who, it is said, killed small animals such as rabbits by breaking their necks between the ground and the gamekeeper's thumb and index finger. The resulting valgus force on the abducted metacarpophalangeal (MCP) joint caused injury to the ulnar collateral ligament. These days this injury is more commonly seen in skiers who land awkwardly with their hand braced on a ski pole, causing the valgus force on the thumb as is seen in this patient. Whereas the term "skier's thumb" is sometimes used, "gamekeeper's thumb" is still in common usage. GAS 795-796; N 441; McM 163

74 A 26-year-old male power lifter visits the outpatient clinic with a painful shoulder. Radiographic examination reveals tendinopathy of the long head of the biceps brachii muscle. Which of the following conditions will most likely be present during physical examination? A. Pain is felt in the anterior shoulder during forced contraction B. Pain is felt in the lateral shoulder during forced contraction C. Pain is felt during abduction and flexion of the shoulder joint D. Pain is felt during extension and adduction of the shoulder joint E. Pain is felt in the lateral shoulder during flexion of the shoulder joint

74 A. The long head of the biceps brachii muscle assists in shoulder flexion and during a tendinopathy would cause pain in the anterior compartment of the shoulder, where it originates at the supraglenoid tubercle. Also, forced contraction would cause a greater tension force on the tendon. GAS 732; N 419; McM 136

76 A male skier had a painful fall against a rocky ledge. Radiographic findings revealed a hairline fracture of the surgical neck of the humerus. The third-year medical student assigned to this patient was asked to determine whether there was injury to the nerve associated with the area of injury. Which of the following tests would be best for checking the status of the nerve? A. Have the patient abduct the limb while holding a 10 lb weight B. Have the patient shrug the shoulders C. Test for presence of skin sensation over the lateral side of the shoulder D. Test for normal sensation over the medial skin of the axilla E. Have the patient push against an immovable object like a wall and assess the position of the scapula

76 C. The axillary nerve passes dorsally around the surgical neck of the humerus (accompanied by the posterior circumflex humeral artery) and can be injured when the humerus is fractured at that location. The axillary nerve provides sensation to the skin over the upper, lateral aspect of the shoulder. Therefore, although the patient might not be able to abduct the arm because of the injury, a simple test of skin sensation can indicate whether there is associated nerve injury of the axillary nerve (CN XI). Shrugging the shoulders can help assess trapezius function, thereby testing the spinal accessory nerve. Intact sensation of the skin on the medial aspect of the axilla and arm is an indication that the radial and intercostobrachial nerves are functional. Pushing against an immovable object tests the serratus anterior muscle and the long thoracic nerve. GAS 718-720; N 465; McM 139

77 A 27-year-old man had lost much of the soft tissue on the dorsum of his left hand in a motorcycle crash. Imaging studies show no other upper limb injuries. Because function of the left extensor carpi radialis longus and brevis tendons was lost, it was decided to replace those tendons with the palmaris longus tendons from both forearms because of their convenient location and relative unimportance. Postoperatively, it is found that sensation is absent in both hands on the lateral palm and palmar surfaces of the first three digits; there is also paralysis of thumb opposition. What is the most likely cause of the sensory deficit and motor loss in both thumbs? A. Bilateral loss of spinal nerve T1 with fractures of first rib bilaterally B. Lower plexus (lower trunk) trauma C. Dupuytren contracture D. Left radial nerve injury in the posterior compartment of the forearm E. The palmaris longus was absent bilaterally; the nerve normally beneath it looked like a tendon and was cut

77 E. The surgeon took the distal segments of the median nerves from both forearms, mistakenly believing them to be palmaris longus tendons. Both of the structures lie in the midline of the ventral surface of the distal forearm and are often of similar appearance in color and diameter. The nerve is located deep to the tendon, when the tendon is present, but when the tendon is absent, the nerve appears to be where the tendon belongs. There is no evidence of rib fractures; even so, a fractured rib would not explain loss of sensation on the lateral portion of the palm. Lower plexus trauma (C8, T1) would result in paralysis of forearm flexor muscles and all intrinsic hand muscles and sensory loss over the medial dorsum of the hand, in addition to palmar sensory loss. Dupuytren's contracture is a flexion contracture of (usually) digits four and five from connective tissue disease in the palm. Radial nerve injury in the posterior forearm would affect metacarpophalangeal joint extension, thumb extension, and so on, not palmar disturbances. GAS 784-785; N 432; McM 150

78 A 15-year-old boy received a shotgun wound to the ventral surface of the upper limb. Three months after the injury the patient exhibits a complete claw hand but can extend his wrist. What is the nature of this patient's injury? A. The ulnar nerve has been severed at the wrist. B. The median nerve has been injured in the carpal tunnel. C. The median and ulnar nerves are damaged at the wrist. D. The median and ulnar nerves have been injured at the elbow region. E. The median, ulnar, and radial nerves have been injured at midhumerus.

78 C. Trauma both to the median and ulnar nerves at the wrist results in total clawing of the fingers. The metacarpophalangeal joints of all digits are extended by the unopposed extensors because the radial nerve is intact. All interossei and lumbricals are paralyzed because the deep branch of the ulnar nerve supplies all of the interossei; lumbricals I and II are paralyzed, for they are innervated by the median nerve; lumbricals III and IV are paralyzed, for they receive supply from the deep ulnar nerve. The interossei and lumbricals are responsible for extension of the interphalangeal joints. When they are paralyzed, the long flexor tendons pull the fingers into a position of flexion, completing the "claw" appearance. If the median nerve were intact, the clawing would be less noticeable in the index and long fingers because the two lumbricals would still be capable of some degree of extension of those interphalangeal joints. If the median nerve alone is injured in the carpal tunnel, there would be loss of thenar opposition but not clawing. If the median and ulnar nerves are both transected at the elbow, the hand appears totally flat because of the loss of long flexors, in addition to intrinsic paralysis. GAS 784, 814-818; N 434; McM 157

82 A 19-year-old man fell from a cliff when he was hiking in the mountains. He broke his fall by grasping a tree branch, but he suffered injury to the C8 to T1 spinal nerve ventral rami. Sensory tests would thereafter confirm the nature of his neurologic injury by the sensory loss in the part of the limb supplied by which of the following? A. Lower lateral brachial cutaneous nerve B. Musculocutaneous nerve C. Intercostobrachial nerve D. Medial antebrachial cutaneous nerve E. Median nerve

82 D. In a lesion of the lower trunk of the brachial plexus, or the C8 and T1 ventral rami, there is sensory loss on the medial forearm and the medial side of hand (dorsal and ventral). The medial cord is an extension of the lower trunk. The medial cord gives origin to the medial antebrachial cutaneous nerve, which supplies the T1 dermatome of the medial side of the antebrachium. The lower lateral brachial cutaneous nerve arises from the radial nerve, C5 and C6. The musculocutaneous nerve arises from the lateral cord, ending in the lateral antebrachial cutaneous nerve, with C5 and C6 dermatome fibers. The intercostobrachial nerve is the lateral cutaneous branch of the T2 ventral primary ramus and supplies skin on the medial side of the arm. The median nerve distributes C6 and C7 sensory fibers to the lateral part of the palm, thumb, index, long finger, and half of the ring finger. GAS 738-745; N 416; McM 138

83 The mastectomy procedure on a 52-year-old woman involved excision of the tumor and removal of lymph nodes, including the pectoral, central axillary, and infraclavicular groups. Six months after her mastectomy, the patient complains to her personal physician of an unsightly deep hollow area inferior to the medial half of the clavicle, indicating a significant area of muscle atrophy and loss. She states that the disfigurement has taken place quite gradually since her mastectomy. Physical examination reveals no obvious motor or sensory deficits. What was the most likely cause of the patient's cosmetic problem? A. Part of the pectoralis major muscle was cut and removed in the mastectomy B. The pectoralis minor muscle was removed entirely in the surgery C. A branch of the lateral pectoral nerve was cut D. The medial pectoral nerve was cut E. The lateral cord of the brachial plexus was injured

83 C. The first branch of the lateral pectoral nerve is typically the only source of motor supply to the clavicular head of the pectoralis major muscle. If it is injured (as in this case of an iatrogenic injury when the infraclavicular nodes were removed), this part of the muscle undergoes atrophy, leaving an infraclavicular cosmetic deficit. The remainder of the lateral pectoral nerve joins the medial pectoral nerve in a neural arch that provides motor supply to the remaining parts of the pectoralis major and the pectoralis minor. Physical examination reveals no obvious motor or sensory deficits. Loss of the medial pectoral nerve would have no effect on the clavicular head of pectoralis major and might not be discernible. Injury to the lateral cord would lead to loss not only of all of the lateral pectoral nerve but also the musculocutaneous nerve, resulting in biceps brachii and brachialis paralysis and lateral antebrachial sensory loss. GAS 724, 742; N 415; McM 141

87 A 45-year-old female motorcyclist, propelled over the handlebars of her bike by an encounter with a rut in the road, lands on the point of one shoulder. She is taken by ambulance to the emergency department. During physical examination, the arm appears swollen, pale, and cool. Any movement of the arm causes severe pain. Radiographic examination reveals a fracture and a large hematoma, leading to diagnosis of Volkmann's ischemic contracture. At which of the following locations has the fracture most likely occurred? A. Surgical neck of humerus B. Radial groove of humerus C. Supracondylar line of humerus D. Olecranon E. Lateral epicondyle

87 C. A fracture of the humerus just proximal to the epicondyles is called a supracondylar fracture. This is the most common cause of a Volkmann ischemic fracture. The sharp bony fragment often lacerates the brachial (or other) artery, with bleeding into the flexor compartment. Diminution of arterial supply to the compartment results in the ischemia. Bleeding into the compartment causes greatly increased pressure, first blocking venous outflow from the compartment, then reducing the arterial flow into the compartment as the pressure rises to arterial levels. The ischemic muscles then undergo unrelieved contracture. A humeral fracture is sometimes placed in a cast from shoulder to wrist, often concealing the ischemia until major tissue loss occurs. Cold, insensate digits, and great pain are warnings of this compartmental syndrome, demanding that the cast be removed and the compartment opened ("released") for pressure reduction and vascular repair. Fracture of the surgical neck endangers the axillary nerve and posterior circumflex humeral artery, although not ischemic contracture. Fracture of the humerus in the spiral groove can injure the radial nerve and profunda brachii artery. Fracture of the olecranon does not result in Volkmann's contracture, although the triceps brachii can displace the distal fractured fragment of the ulna. GAS 766; N 420; McM 149

89 A 67-year-old woman had a bad fall while walking her dog the evening before. She states that she fell on her outstretched hand. Radiographs do not demonstrate any bony fractures. The clinician observes the following signs of neurologic injury: weakness of flexion of her wrist in a medial direction, a loss of sensation on the medial side of the hand, and clawing of the fingers. Where is the most likely place of nerve trauma? A. Behind the medial epicondyle B. Between the pisiform bone and the flexor retinaculum C. Within the carpal tunnel D. At the cubital fossa, between the ulnar and radial heads of origin of flexor digitorum superficialis E. At the radial neck, 1 cm distal to the humerocapitellar joint

89 A. The force of the woman's fall on the outstretched hand was transmitted up through the forearm, in this case resulting in dislocation of the olecranon at the elbow, putting traction on the ulnar nerve as it passes around the medial epicondyle of the humerus. Ulnar trauma at the elbow can cause weakness in medial flexion (adduction) at the wrist, from loss of the flexor carpi ulnaris. Ulnar nerve injury also results in sensory loss in the medial hand and paralysis of the interossei and medial two lumbricals, with clawing especially of digits 4 and 5. Injury of the ulnar nerve at the pisiform bone would not affect the flexor carpi ulnaris, nor would it produce sensory loss on the dorsum of the hand because the dorsal cutaneous branch of the ulnar branches off proximal to the wrist. Carpal tunnel problems affect median nerve function, which is not indicated here. The ulnar nerve passes medial to the cubital fossa between the heads of the flexor carpi ulnaris, not between the heads of the flexor digitorum superficialis. Injuries at the radial neck affect the site of division of the radial nerve, and its paralysis would not result in the clinical problems seen in this patient. GAS 768, 784; N 464; McM 145

93 Several weeks after surgical dissection of her left axilla for the removal of lymph nodes for staging and treatment of her breast cancer, a 32-year-old woman was told by her general physician that she had "winging" of her left scapula when she pushed against resistance during her physical examination. She told the physician that she had also experienced difficulty lately in raising her left arm above her head when she was combing her hair. In a subsequent consult visit with her surgeon, she was told that a nerve was accidentally injured during the diagnostic surgical procedure and that this produced her scapular abnormality and inability to raise her arm normally. What was the origin of this nerve? A. The upper trunk of her brachial plexus B. The posterior division of the middle trunk C. Ventral rami of the brachial plexus D. The posterior cord of the brachial plexus E. The lateral cord of the brachial plexus

93 C. The long thoracic nerve was injured during the axillary dissection, resulting in paralysis of the serratus anterior muscle. The serratus anterior is important in rotation of the scapula in raising the arm above the level of the shoulder. Its loss results in protrusion of the medial border ("winging" of the scapula), which is more obvious when one pushes against resistance. The long thoracic nerve arises from the ventral rami of C5, C6, and C7. The upper trunk (C5, C6) supplies rotator and abductor muscles of the shoulder and elbow flexors. The posterior division of the middle trunk contains C7 fibers for distribution to extensor muscles; likewise, the posterior cord supplies extensors of the arm, forearm, and hand. The lateral cord (C5, C6, and C7) gives origin to the lateral pectoral nerve, the musculocutaneous nerve, and the lateral root of the median nerve. There is no sensory loss in the limb in this patient; injury to any of the other nerve elements listed here would be associated with specific dermatome losses. GAS 726-727; N 413; McM 138

95 A 24-year-old female basketball player is admitted to the emergency department after an injury to her shoulder. Radiographic examination reveals a shoulder dislocation. What is the most commonly injured nerve in shoulder dislocations? A. Axillary B. Radial C. Median D. Ulnar E. Musculocutaneous

95 A. The axillary nerve is a direct branch of the posterior cord and wraps around the surgical neck of the humerus to innervate the teres minor and the deltoid muscles. With this anatomic arrangement, the axillary nerve is tightly "tethered" to the proximal humerus. When the head of the humerus is dislocated, it often puts traction on the axillary nerve. GAS 718-719; N 413; McM 142

96 An 85-year-old man is admitted to the hospital with a painful arm after lifting a case of wine. Physical examination gives evidence of a rupture of the long tendon of the biceps brachii (Fig. 6-6). Which of the following is the most likely location of the rupture? A. Intertubercular groove B. Midportion of the biceps brachii muscle C. Junction with the short head of the biceps brachii muscle D. Proximal end of the combined biceps brachii muscle E. Bony insertion of the muscle

96 A. The tendon of the long head of the biceps brachii muscles runs in the intertubercular groove on the proximal humerus as it changes direction and turns medially to attach to the supraglenoid tubercle of the scapula. This change in direction within an osseous structure predisposes the tendon to wear and tear, particularly in people who overuse the biceps brachii muscle. This type of injury presents with a characteristic sign called the "Popeye sign" after the cartoon character. GAS 731-732; N 417; McM 114

98 A 5-year-old boy is admitted to the emergency department after falling from a tree. The parents are informed by the radiologist that their son's fracture is the most common fracture that occurs in children. Which of the following bones was broken? A. Humerus B. Radius C. Ulna D. Scaphoid E. Clavicle

98 E. During a fall on an outstretched upper limb, the forces are conducted through the hand on up through the bones of the limb in succession. Often these bones do not fracture but rather pass the compressive forces proximally. The appendicular skeleton joins with the axial skeleton at the sternoclavicular joint. The forces are not sufficiently transferred to the sternum, causing the clavicle to absorb the force, resulting in common pediatric fracture of this sigmoidal-shaped bone. GAS 711; N 461; McM 112

99 A 22-year-old woman visits the outpatient clinic with pain in her left upper limb. She has a long history of pain in this limb and difficulty with fine motor tasks of the hand. Physical examination reveals paraesthesia along the medial surface of the forearm and palm and weakness and atrophy of gripping muscles ("long flexors") and the intrinsic muscles of the hand. The radial pulse is diminished when her neck is rotated to the ipsilateral side (positive Adson's test). What is the most likely diagnosis? A. Erb-Duchenne paralysis B. Aneurysm of the brachiocephalic artery, with plexus compression C. Thoracic outlet syndrome D. Carpal tunnel syndrome E. Injury to the medial cord of the brachial plexus

99 C. The patient is suffering from thoracic outlet syndrome, involving neural and vascular elements. This results from any condition that decreases the dimensions of the superior thoracic aperture (the formal name of the thoracic outlet). It could be a result of a cervical rib, accessory muscles, and/or atypical connective tissue bands at the root of the neck. In this case, symptoms involve the arm, forearm, and hand. Paraesthesia along the medial forearm and hand and atrophy of long flexors and intrinsic muscles point to a possible compression or traction problem of the lower trunk (C8, T1) rather than a lesion of either the median or ulnar nerve. The lateral palm has no sensory problem, which tends to rule out median nerve involvement. Changes in the radial artery pulse point to vascular compression. Erb-Duchenne paralysis of the upper trunk would affect proximal limb functions, such as arm rotation, abduction, and so on. This lesion is on the left side, so the brachiocephalic artery could not be involved because it arises from the right side of the aortic arch; moreover, it would not compress the brachial plexus. Carpal tunnel syndrome would not explain the problems of the forearm and medial hand, or the long flexor atrophy. An isolated medial cord lesion would not explain the atrophy of all long flexors and intrinsic muscles and does not explain the radial pulse characteristics. The ischemic pain in the arm is due to vascular compression. GAS 150; N 183; McM 138

D. Osgood-Schlatter disease is also called tibial tuberosity apophysitis and affects the area of the tibial tuberosity. It is not a disease but a problem of overuse, typically in boys of 12 to 14 years or girls 10 to 12 years of age. Very active boys and girls, usually during a growth spurt, are subject to the pain and swelling that occur at the site of attachment of the patellar ligament. The ligament can tear, resulting in a long period of healing following treatment. The medial femoral condyle is the area of attachment of the medial collateral ligament and medial meniscus of the knee joint. The posterior intercondylar eminence is the location of origin of the posterior cruciate ligament. The intercondylar eminence is a bony protuberance on the tibial plateau to which the cruciate ligaments and menisci are attached. The anterolateral tibial tubercle, or Gerdy's tubercle, is the attachment of the iliotibial band or tract; thus it connects the femur and tibia laterally.

A 10-year-old girl is admitted to the emergency department after falling from a tree in which she was playing with her friends. Radiologic and physical examinations reveal Osgood-Schlatter disease (Fig. 5-4). Which of the following bony structures is chiefly affected? A. Medial condyle of tibia B. Posterior intercondylar area C. Intercondylar eminence D. Tibial tuberosity E. Anterolateral tibial tubercle (Gerdy's tubercle)

C. The lymphatic drainage of the foot follows its venous drainage. The small saphenous vein drains the lateral side of the foot and the posterolateral leg. It drains into the popliteal vein in the popliteal fossa. Therefore a lesion on the lateral side of the foot will drain to the popliteal nodes in the popliteal fossa.

A 12-year-old boy is brought to the physician by his father because of redness and swelling of his left foot for 24 hours. Three days earlier he had scraped his foot while wading in a drainage ditch. Examination of the foot shows a purulent abrasion with edema, erythema, and tenderness on the lateral side. Infection will most likely spread from the lateral side of the foot to the regional lymph nodes in which area? A. Lateral surface of the thigh B. Medial malleolus, posteriorly C. Popliteal fossa D. Sole of the foot E. Superficial inguinal area

A. Knowledge of Hilton's Law would lead to this correct answer. This law in a modified form, can be remembered as "a joint is innervated by the same nerves that innervate the muscles that move that joint." A complete explanation of this law can be found in an article by Hebert-Blouin et al., Clinical Anatomy 27:548-555, 2013. The deep fibular (peroneal) nerve is the only nerve listed that innervates muscles that move the ankle joint.

A 15-year-old boy falls and injures his ankle while skateboarding. Examination in the emergency department leads to the conclusion that the ankle is mildly sprained, and it is wrapped with an elastic bandage. The boy still complains of pain in his ankle. Which of the following peripheral nerves is involved in carrying pain sensation from the ankle? A. Deep fibular (peroneal) B. Femoral C. Obturator D. Posterior femoral cutaneous E. Sural

B. The deep fibular (peroneal) nerve is a branch of the common fibular (peroneal) nerve. It is mainly a motor nerve that innervates the anterior compartment of the leg. Its only cutaneous innervation is to the skin of the first web space. The saphenous nerve innervates the medial side of the leg and foot. The cutaneous branch of superficial fibular (peroneal) nerve innervates the anterior part of the lower leg and the dorsum of the foot. The sural nerve innervates the lateral side of the leg and foot.

A 16-year-old boy presents to the emergency department with a fracture of the first and second toes of his right foot. He received an anesthetic injection in the first web space of his foot, to permit easy manipulation and correction. Which nerve was blocked by the anesthesia? A. Saphenous B. Cutaneous branch of deep fibular (peroneal) C. Cutaneous branch of superficial fibular (peroneal) D. Sural E. Common fibular (peroneal)

A. The popliteal lymph nodes are the first to receive lymph from the foot. These nodes will then drain into the deep inguinal nodes and then to the external iliac nodes. The superficial inguinal and internal iliac nodes do not receive lymph from the foot

A 16-year-old boy received a superficial cut on the lateral side of his foot while playing football and is admitted to the emergency department where the wound is sutured. Four days later the patient returns to the hospital with high fever and swollen lymph nodes. Which group of nodes will first receive lymph from the infected wound? A. Popliteal B. Vertical group of superficial inguinal C. Deep inguinal D. Horizontal group of superficial inguinal E. Internal iliac

B. Ligaments act to prevent excessive movement of joints. When a joint is forced into a position, that ligament is stretched and will be tender or rupture if the force is severe enough. Inversion is when the sole of the foot is turned medially and therefore will stretch ligaments that oppose this action. The calcaneofibular ligament is on the lateral side and stretches between the fibula and the calcaneous. It is the only ligament that would be damaged during such an action. The calcaneonavicular and long and short plantar ligaments are located on the plantar surface of the foot and will not be damaged during inversion injuries. The deltoid ligament is located medially and will not be affected.

A 16-year-old teenage girl suffered an inversion sprain of her ankle during dance class. Physical examination in the clinic most likely reveals severe tenderness over which ligament? A. Calcaneonavicular (spring) B. Calcaneofibular C. Long plantar D. Short plantar E. Deltoid

A. The medial meniscus is firmly attached to the medial (tibial) collateral ligament. Damage to the medial collateral ligament often causes concomitant damage to the medial meniscus because of this relationship. The anterior cruciate ligament lies inside the knee joint capsule but outside the synovial cavity. It is taut during extension of the knee and may be torn when the knee is hyperextended. If this were damaged along with the medial meniscus and medial cruciate ligament, an "unhappy triad" (of O'Donoghue, or Donahue, both spellings are correct; also called a "blow knee") injury would result. The lateral meniscus is not attached to the medial collateral ligament but receives muscular attachment to the popliteus muscle. The posterior cruciate ligament also lies outside of the synovial cavity and limits hyperflexion of the knee. The tendon of the semitendinosus forms one third of the pes anserinus, with the tendons of the sartorius and gracilis making up the other two thirds. The pes anserinus (goose foot) is located at the medial border of the tibial tuberosity, and a portion can be used for surgical repair of the anterior cruciate ligament.

A 19-year-old football player was hit on the lateral side of his knee just as he put that foot on the ground. Unable to walk without assistance, he is taken to the hospital. An MRI examination reveals a torn medial collateral ligament. Which structure would most likely also be injured due to its attachment to this ligament? A. Medial meniscus B. Anterior cruciate ligament C. Lateral meniscus D. Posterior cruciate ligament E. Tendon of the semitendinosus

A. With sufficient downward force, the head of the talus can break through the plantar calcaneonavicular (spring) ligament, causing the medial longitudinal arch of the foot to fall, forcing the anterior part of the foot into abduction. The plantar calcaneonavicular ligament is attached between the sustentaculum tali of the calcaneus and the medial surface of the navicular bone, with the head of the talus lying directly upon the inner surface of the ligament. The cuboid bone is located lateral and anterior to the talus bone and would not be fractured. The plantar aponeurosis, a dense, wide band of tissue beneath the fascia of the sole, attaches to the calcaneus and ends distally in longitudinal bands to each of the toes. It stretches very little, even under very heavy loads, and would not rupture in this case. The anterior talofibular ligament is very often injured in "sprained ankle" but would not be directly involved here. The distal tibiofibular joint is a fibrous (and usually nonsynovial) type of joint (called a syndesmosis) between the tibia and fibula, not involved in the displacement of the talus bone.

A 19-year-old patient is admitted to the orthopedic service with a complaint of severe pain in his very swollen and discolored foot. He states that he hurt the foot when jumping from his girlfriend's bedroom window to the concrete driveway below. Plain film radiographic studies reveal that the head of the talus has become displaced inferiorly, thereby causing the medial longitudinal arch of the foot to fall. What would be the most likely, serious problem in such a case? A. Tearing of the plantar calcaneonavicular (spring) ligament B. Fracture of the cuboid bone C. Interruption of the plantar aponeurosis D. Sprain of the anterior talofibular ligament E. Disruption of the distal tibiofibular ligament

E. The calcaneofibular ligament is located inferior and just anterior to the lateral malleolus and connects the lateral malleolus to the calcaneus. The interosseous ligament between the tibia and the fibula is located medially and superior to the lateral malleolus. The tibionavicular ligaments are located on the medial side of the ankle joint, and the point of injury and tenderness is at the lateral side. The anterior tibiofibular ligament is located anterior to the ankle joint, away from the point of injury.

A 19-year-old woman is admitted to the emergency department with complaints of pain and swelling to the right ankle. In a recent volleyball game she jumped to spike the ball then landed on the opponent's shoe with her right foot. She recalls hearing a loud "pop" and felt immediate pain to the ankle. She was unable to bear weight subsequently. On examination, the right ankle was swollen, with maximal tenderness inferior and anterior to the lateral malleolus. Radiographs of the ankle showed no fractures. Which of the following structures were most likely injured? A. Posterior talofibular ligament B. Interosseous ligament between the tibia and fibula C. Tibionavicular ligament D. Anterior tibiofibular ligament E. Calcaneofibular ligament

B. The tensor fasciae latae assists in flexion of the thigh, as well as medial rotation and abduction. Damage to this muscle would adversely affect these motions. The rectus femoris extends the hip. The vastus intermedius extends the knee. The semimembranosus extends the hip and flexes and medially rotates the knee. The sartorius assists in flexion and lateral rotation of the hip, as well as in medial rotation of the knee.

A 20-year-old man visits the family physician complaining of difficulty to flex and medially rotate his thigh while running and climbing. Which of the following muscles is most likely damaged in this individual? A. Rectus femoris B. Tensor fasciae latae C. Vastus intermedius D. Semimembranosus E. Sartorius

B. The calcaneofibular ligament is a round cord that passes posteroinferiorly from the tip of the lateral malleolus to the lateral surface of the calcaneus. A forced inversion of the foot can result in tearing of the calcaneofibular ligament and sometimes the anterior talofibular ligament as well. Both of these ligaments act to stabilize the foot and prevent an inversion injury. The plantar calcaneonavicular ligament supports the head of the talus. The long plantar ligament passes from the planter surface of the calcaneus to the groove on the cuboid and is important in maintaining the longitudinal arch of the foot. The short plantar ligament is located deep (superior) to the long plantar ligament and extends from the calcaneus to the cuboid and is also involved in maintaining the longitudinal arch of the foot. The deltoid (medial ligament of the ankle) attaches proximally to the medial malleolus and fans out to reinforce the joint capsule of the ankle.

A 22-year-old football player is admitted to the hospital with pain and swelling over the lateral aspect of the ankle. The emergency department doctor diagnoses an inversion sprain. Which ligament was most likely injured? A. Plantar calcaneonavicular (spring) B. Calcaneofibular C. Long plantar D. Short plantar E. Deltoid

A. The common fibular (peroneal) nerve passes around the head of the fibula and gives off deep (L4-5) and superficial fibular (peroneal) nerve (L5, S1-2) branches. The two nerves supply the dorsiflexors and evertors of the foot, respectively. In this case, the tibialis anterior and extensor digitorum longus are the only muscles listed that are supplied by either of these nerve branches, and both are innervated by the deep fibular (peroneal) nerve. The fibularis (peroneus) brevis and longus are innervated by the superficial fibular (peroneal) nerve and are evertors of the foot. The tibial nerve supplies each of the other muscles listed.

A 22-year-old male martial arts competitor was examined by the clinician because of pain and serious disability suffered from a kick to the side of his knee. Physical examination revealed a dark bruise just distal to the head of the fibula. Which of the following muscles will most likely be paralyzed? A. Tibialis anterior and extensor digitorum longus B. Tibialis posterior C. Soleus and gastrocnemius D. Plantaris and popliteus E. Flexor digitorum longus and flexor hallucis longus

D. The anterior cruciate ligament is attached to the anterior intercondylar area and the posterior part of the medial surface of the lateral femoral condyle. Posterior displacement of the femur on the tibia is prevented by the ACL. With the knee joint flexed, the ACL prevents the tibia from being pulled anteriorly. The posterior cruciate ligament is stronger, shorter, and broader, less oblique, and prevents anterior displacement of the femur on the tibia. Lateral collateral ligaments are cordlike and are attached proximally to the lateral side of the head of the fibula overlapped by the tendon of biceps femoris. The medial collateral ligament is a flat band and is attached above to the medial condyle of the femur and below to the medial surface of the shaft of the tibia. It is firmly attached to the edge of the medial meniscus and consequently is more prone to be injured. The oblique popliteal ligament is a tendinous expansion derived from the semimembranosus muscle. It strengthens the posterior aspect of the knee joint capsule. The patellar ligament (tendon) connects the lower border of the patella with the smooth convexity on the tuberosity of the tibia. It represents the continuation of the quadriceps tendon.

A 22-year-old male professional football player is admitted to the emergency department with acute right knee pain after sustaining a kick injury to an extended leg. A radiograph and a subsequent MRI revealed that the trauma caused anterior displacement of the tibia with respect to her femur. Which of the following ligaments was most likely injured? A. Fibular (lateral) collateral B. Tibial (medial) collateral C. Patellar D. Anterior cruciate E. Posterior cruciate F. Oblique popliteal

B. The tibial nerve divides into the medial and lateral plantar nerves on the medial side of the ankle. These two nerves provide sensation for the sole of the foot. Sensory supply to the dorsum of the foot is provided mostly by the superficial fibular (peroneal) nerve, with the deep fibular (peroneal) nerve providing sensation for the skin between the first and second toes. Foot drop would be caused by interruption of the common fibular (peroneal) nerve. Sensory loss to the lateral side of the foot results from loss of the sural nerve. Paralysis of the extensor digitorum brevis would be attributed to injury to the terminal motor branch of the deep fibular (peroneal) nerve.

A 22-year-old man is admitted to the emergency department after falling from his bicycle. Radiologic examination reveals a fracture of the tibia above the ankle. MRI and physical examination reveal that the tibial nerve is severed on the posterior aspect of the tibia. Which of the following signs will most likely be present during physical examination? A. Sensory loss of the dorsum of the foot B. Sensory loss on the sole of the foot C. Foot drop D. Paralysis of the extensor digitorum brevis E. Sensory loss of the entire foot

E. The posterior cruciate ligament runs from the posterior aspect of the intercondylar area of the tibia to the medial wall of the intercondylar fossa. It prevents posterior displacement of the tibia relative to the femur. This is usually tested with the posterior drawer test, in which the physician pushes the tibia backward while the knee is flexed in an attempt to displace it posteriorly. This is called the positive posterior drawer sign. The anterior cruciate ligament prevents anterior displacement of the tibia on the femur. The medial and lateral collateral ligaments stabilize the medial and lateral sides of the knee joint, respectively. The medial meniscus is an intracapsular fibrocartilage that improves the articulation of the femur and tibia.

A 22-year-old soccer player collides with one of her teammates. During examination on the field, the posterior drawer test was performed and the tibia moved backward in relation to her femur. Injury to which structure is confirmed by performing this test? A. Anterior cruciate ligament B. Lateral collateral ligament C. Medial collateral ligament D. Medial meniscus E. Posterior cruciate ligament

D. The gluteus maximus inserts into the gluteal tuberosity and the iliotibial tract. Although the gluteus maximus would continue to contract at the regions of insertion, their orientation would be displaced by the fracture. The gluteus medius, gluteus minimus, obturator internus, and piriformis all insert on some aspect of the greater trochanter of the femur

A 22-year-old woman is admitted to the emergency department after another vehicle collided with the passenger side of the convertible in which she was riding. Radiologic examination reveals an avulsion fracture of the greater trochanter. Which of the following muscles would continue to function normally if such an injury was incurred? A. Piriformis B. Obturator internus C. Gluteus medius D. Gluteus maximus E. Gluteus minimus

A. Gluteal injections should be given anterior and superior to a line drawn between the posterior superior iliac spine and the greater trochanter to avoid the sciatic nerve and other important nerves and vessels. Occasionally, one can encounter the lateral cutaneous branch of the iliohypogastric nerve, but this usually causes no serious problem. Certainly, one must stay anterior to a vertical line dropped from the highest point of the ilium. If the injected material is too near the sciatic nerve or other motor nerves, it can infiltrate the connective tissue sheath of the nerve, following the nerve, and result in major insult to the neural elements. The needle can cause trauma to this, or other nerves, likewise. Precautions to avoid the sciatic nerve are especially important in injecting the gluteal area in babies. The reduced dimensions are less "forgiving" in babies.

A 22-year-old woman is admitted with high fever and vaginal discharge. Physical and laboratory examinations reveal gonorrheal infection. A series of intramuscular antibiotic injections are ordered. Into which of the following parts of the gluteal region should the antibiotic be injected to avoid nerve injury? A. Anterior and superior to a line between the posterior superior iliac spine and the greater trochanter B. In the middle of a line between the anterior superior iliac spine and the ischial tuberosity C. Inferolateral to a line between the posterior superior iliac spine and the greater trochanter D. Inferomedial to a line between the posterior superior iliac spine and the greater trochanter E. Halfway between the iliac tuberosity and the greater trochanter

C. Incapacitation and unconsciousness from use of cocaine and other powerful narcotics have led to numerous cases of the "gluteal crush syndrome." Compression of the gluteal region while supine for extended periods of time can lead to gluteal crush injury, in which the nerves and vessels of the gluteal area are compressed. This can result in loss of gluteal muscles and other soft tissues and sciatic nerve compression. The nerve compression can cause paralysis of knee flexors and muscles of the anterior and lateral compartments of the leg, with sensory loss in the posterior thigh and leg and sensory loss in the foot. Tibial nerve loss would not result in loss of dorsiflexion of the foot nor generalized sensory loss. Neither piriformis entrapment nor femoral nerve entrapment is associated with loss of gluteal musculature, nor loss of knee flexion or plantar flexion of the foot, nor do they lead to general sensory loss in the limb.

A 22-year-old woman is found in a comatose condition, having lain for an unknown length of time on the tile floor of the courtyard. She is found in possession of cocaine. The patient is transported to the hospital while EMT personnel receive instructions for treatment of drug overdose. During the physical examination the patient's gluteal region shows signs of ischemia. After regaining consciousness, she exhibits paralysis of knee flexion and dorsal and plantar flexion and sensory loss in the limb. What is the most likely diagnosis? A. Tibial nerve loss B. S1-2 nerve compression C. Gluteal crush injury D. Piriformis entrapment syndrome E. Femoral nerve entrapment

E. One important function of the deltoid ligament is the prevention of excessive extension of the ankle. The ligament is so strong that excessive eversion can cause the medial malleolus to be pulled off (an avulsion fracture) rather than tearing the deltoid ligament.

A 23-year-old male basketball player injured his foot during training and is admitted to the emergency department. An MRI examination reveals a hematoma around the medial malleolus. Upon physical examination the patient shows excessive eversion of his foot. Which of the following ligaments most likely has a tear? A. Plantar calcaneonavicular (spring) B. Calcaneofibular C. Long plantar D. Short plantar E. Deltoid

D. Both the medial and lateral menisci are subject to rotational injuries and may be torn. The medial meniscus is much more liable to injury because it is attached to the fused deep layer of the medial collateral ligament and joint capsule. The lateral meniscus is separated from the fibular collateral ligament and is external to the capsule of the knee joint. Commonly seen in football players' knees, meniscal tears are usually diagnosed by MRI or by arthroscopy. The presenting symptoms of tearing may be pain and swelling, or locking of the knee. Locking of the knee suggests a bucket handle tear, in which a partly detached cartilage wedges between the tibia and femur, inhibiting further movement. A bucket handle tear is often associated with rupture of the anterior cruciate ligament. Sometimes a momentary click can be heard in flexion/extension movements of the knee. Meniscectomy is a successful operation, but currently there is greater emphasis on repairing small tears. Meniscal cysts can form secondary to meniscal tears and some of these can also be treated arthroscopically.

A 23-year-old man is admitted to the emergency department after injuring his knee while playing football. During physical examination there is pain and swelling of the knee, in addition to locking of the knee in full extension. Radiologic examination reveals a bucket handle meniscal tear (Fig. 5-12). Which of the following ligaments is most likely injured? A. Posterior cruciate B. Medial collateral C. Lateral collateral D. Anterior cruciate E. Coronary

A. The ventral ramus of L4 contains both sensory and motor nerve fibers. Injury from a stab wound could result in loss of sensation from the dermatome supplied by this segment. A dermatome is an area of skin supplied by a single spinal nerve; L4 dermatome supplies the medial aspect of the leg and foot. Loss of the Achilles tendon reflex relates primarily to an S1 deficit. The Achilles tendon reflex is elicited by tapping the calcaneus tendon, which results in plantar flexion. The obturator internus and gluteus medius and minimus are responsible for abduction of the thigh and are innervated by nerves L4, L5, and S1 (with L5 usually dominant). Nerves L5, S1, and S2 are responsible for eversion of the foot (S1 dominant)

A 23-year-old man is admitted to the emergency department with a deep, bleeding stab wound of the pelvis. After the bleeding has been arrested, a magnetic resonance imaging (MRI) examination gives evidence that the right ventral primary ramus of L4 has been transected. Which of the following problems will most likely be seen during physical examination? A. Reduction or loss of sensation from the medial aspect of the leg B. Loss of the Achilles tendon reflex C. Weakness of abduction of the thigh at the hip joint D. Inability to evert the foot E. Reduction or loss of sensation from the medial aspect of the leg and loss of Achilles tendon reflex

B. The original axial vessel of the lower limb is retained as the (usually tiny) sciatic branch of the inferior gluteal artery. In some cases this vessel is retained as the primary proximal vessel to the limb, wherein there is hypoplastic development of the femoral artery. Aneurysms of the enlarged sciatic artery in the gluteal region are relatively common, as is rupture of the vessel (with profuse bleeding) if they are exposed in the gluteal area. The profunda femoris or deep femoral branch of the femoral artery usually provides three perforating branches to the posterior compartment, but not a branch such as that described. The descending branch of the medial circumflex femoral anastomoses with the first perforator. The superior gluteal artery anastomoses with the inferior gluteal by a descending branch or branches. The descending branch of the lateral circumflex femoral is the descending genicular artery, which anastomoses with the superior lateral genicular branch of the popliteal artery

A 23-year-old man is admitted to the emergency department with pain and cyanosis of his right lower limb. Doppler ultrasound studies reveal deficiency in development of his femoral artery, which appears to terminate mid thigh. A thrombotic occlusion is seen in an unusual, rather tortuous, large vessel in the posterior compartment of the thigh, arising in the gluteal area and continuous inferiorly with a normal appearing popliteal artery. It is decided that a vascular graft should be placed from the femoral artery to the popliteal artery. What is the identity of the aberrant artery? A. A large, fifth perforating branch of the femoral B. An sciatic branch of the inferior gluteal artery C. Descending branch of the medial circumflex femoral D. Descending branch of the superior gluteal artery E. An enlarged descending lateral circumflex femoral artery

B. The psoas muscle arises from the base of the transverse processes, the sides of the vertebral bodies, and the intervertebral discs, from the twelfth thoracic to the fifth lumbar vertebrae and inserted into the lesser trochanter of the femur. The psoas flexes the thigh at the hip joint on the trunk, or if the thigh is fixed, it flexes the trunk on the thigh, as in sitting up from a lying position. The inflamed appendix is pushed up against the peritoneum from the the contracted psoas. As a result it is in touch with the parietal peritoneum, producing acute pain. In some other cases it may retain the purulence of a psoas abscess, and spinal tuberculosis may present as a cold abscess in the groin. The psoas is enclosed in a fibrous sheath that is derived from the lumbar fascia. The sheath is not part of the lumbar fascia but the lateral edge blends with the anterior layer of that fascia.

A 23-year-old man is taken to the emergency department because of anorexia, nausea, vomiting, and severe abdominal pain in the right lower quadrant. On examination, he has tenderness in the right lower quadrant with rebound tenderness. The physician suspects appendicitis. To confirm this diagnosis, the physician attempts to straighten the patient's flexed thigh. This causes the patient to wince with pain. Which of the following muscles most likely caused this symptom? A. Adductor magnus B. Psoas major C. Biceps femoris D. Obturator internus E. Gluteus maximus

C. Mesenchyme between digital rays undergoes apoptosis for the digits to form. Failure or incomplete apoptosis usually results in fused digits (syndactyly). This may involve the skin and soft tissues alone or may include the bone. Digital rays form from the hand plate. Failure of development of any digital ray results in underdevelopment of a finger or toe. Neural crest cells do not contribute to the formation of the foot. The zone of polarizing activity modulates the patterning of the limb in the anteroposterior diameter. The abnormality described did not involve the phalanges, as shown by radiograph, and thus could not have been caused by faulty chondrification.

A 23-year-old woman delivered a live male infant at 37 weeks' gestation after an uneventful pregnancy. Examination of the infant revealed the right second and third toes were fused. Radiographs of the right foot indicated 14 phalanges in their correct position. After review by the pediatric orthopedic surgeon, corrective surgery for the deformity was scheduled. Which of the following embryologic conditions explains the infant's condition? A. The digital ray for the third toe did not develop B. Excessive neural crest cell migration into the foot C. Incomplete apoptosis of tissue between digital rays D. Lack of signal from the zone of polarizing activity (ZPA) E. Faulty development of chondrification centers

A. The skin of the anterior medial thigh and medial leg and foot is supplied by the femoral nerve. The saphenous nerve is a branch of the femoral and only supplies the medial leg and foot. The obturator supplies the medial and medial posterior aspect of the thigh. The tibial nerve supplies the skin of the posterolateral leg, lateral ankle and foot and sole of the foot. The fibular (peroneal) nerve supplies the skin over the lateral aspect of the leg and dorsal aspect of the foot.

A 23-year-old woman was taken to the emergency department after being involved in a head-on collision with a truck. On physical examination a hematoma was seen in the medial thigh. A CT scan revealed a fracture of the femur with a ruptured femoral artery. She was taken to the operating room for repair of the damaged structures. Two days postoperatively during physical examination the patient has loss of sensation to the anterior medial thigh and medial side of her leg and foot. Branches of which of the following nerves were most likely injured in the repair of the fracture? A. Femoral B. Saphenous C. Obturator D. Tibial E. Fibular (peroneal)

D: The saphenous nerve is a cutaneous branch of the femoral nerve and supplies the skin on the medial leg along the great saphenous vein. Middle clunial is a posterior division of S1 to S3. Lateral femoral cutaneous is from L2 to L3 and the lumbar plexus but is not a femoral nerve branch. Superficial fibular (peroneal) nerve is a branch from the common fibular (peroneal) nerve.

A 24-year-old female motocross racer was involved in a crash that left her right leg pinned under her bike. After the accident, she could no longer extend her right knee. The affected nerve gives rise to which of the following cutaneous nerves? A. Middle clunial nerve B. Lateral femoral cutaneous branch C. Superficial fibular (peroneal) nerve D. Saphenous nerve E. Deep fibular (peroneal)

B: Extensors of the knee are the quadriceps femoris muscle, which attaches to the tibia via the patellar ligament. This muscle group is supplied by the femoral nerve. The sciatic nerve supplies the posterior thigh muscles not the anterior compartment. The obturator supplies the medial compartment and the saphenous nerve does not supply motor innervation to any muscles but is a cutaneous branch of the femoral nerve

A 24-year-old female motocross racer was involved in a crash that left her right leg pinned under her bike. After the accident, she could no longer extend her right knee. Which of the following nerves was most likely affected? A. Sciatic nerve B. Femoral nerve C. Obturator nerve D. Saphenous nerve E. Tibial nerve

B. The gastrocnemius muscle arises from the femur just proximal to the femoral condyles. This strong muscle could displace the distal fragment of the fractured femur posteriorly. In addition, the popliteal artery is the deepest structure in the popliteal fossa (right against the popliteal surface of the distal femur) and is susceptible to laceration in this scenario as the fractured end of the distal femoral fragment is pulled against the popliteal artery. Orthopedic surgeons always look for damage to the popliteal artery in a patient with a supracondylar fracture. The soleus arises from the tibia and would have no effect upon the femur. The semitendinosus arises from the ischial tuberosity and inserts medially on the proximal tibia, via the pes anserinus. The tibialis anterior arises from the tibia and inserts mostly onto the navicular bone.

A 24-year-old man is admitted to the emergency department after a car collision. Radiologic examination reveals a fracture at the junction of the middle and lower thirds of the femur. An MRI examination provides evidence that the popliteal vessels were injured when the distal fragment of the fracture was pulled posteriorly. Which of the following muscles is most likely to displace the distal fracture fragment? A. Soleus B. Gastrocnemius C. Semitendinosus D. Gracilis E. Tibialis anterior

C. A severe injury of the tibial nerve in the popliteal fossa would result in a dorsiflexed and everted foot because of the intact muscles of the extensor (anterior) and evertor (lateral) compartments of the leg. It would result also in some weakening of knee flexion because of loss of the gastrocnemius muscle, which flexes the knee and plantar flexes the foot. The hamstrings also flex the knee, so this function would not be lost. Plantar flexion at the ankle would be paralyzed with the loss of the gastrocnemius and soleus, in addition to the flexors of the toes, and inversion by the tibialis posterior. Foot drop results from loss of the anterior compartment, innervated by the deep fibular (peroneal) nerve.

A 24-year-old woman received a small-caliber bullet wound to the popliteal fossa from a drive-by assailant. The patient was admitted to the emergency department, where the surgeons recognized that the bullet had severed the tibial nerve. Such an injury would most likely result in which of the following? A. Inability to extend the leg at the knee B. Foot drop C. A dorsiflexed and everted foot D. A plantar flexed and inverted foot E. Total inability to flex the leg at the knee joint

A. The femoral vein lies medial to the femoral artery in the femoral sheath. The femoral sheath is broken into three compartments: lateral, intermediate, and medial. The lateral compartment contains the femoral nerve. The medial compartment encloses the femoral canal and consists of lymphatic tissue and a lymph node, plus areolar tissue. The intermediate contains the femoral vein.

A 25-year-old man, an intravenous drug abuser, had been injecting himself with temazepam (a powerful intermediate acting drug in the same group as diazepam (Valium) and heroin for 5 years, leaving much residual scar tissue over points of vascular access. The patient is admitted to the emergency department for a detoxification program requiring an intravenous infusion. The femoral veins in his groin are the only accessible and patent veins for intravenous use. Which of the following landmarks is the most reliable to identify the femoral veins? A. The femoral vein lies medial to the femoral artery. B. The femoral vein lies within the femoral canal. C. The femoral vein lies lateral to the femoral artery. D. The femoral vein lies directly medial to the femoral nerve. E. The femoral vein lies lateral to the femoral nerve.

C. In an inversion injury the most common ligament involvement comes from the anterior talofibular and calcaneofibular ligaments. The medial plantar nerve is medially located within the sole of the foot and might be injured by traction in an eversion injury, not an inversion injury. The posterior talofibular ligament is located posteriorly and is not usually injured in an inversion injury. The deltoid ligament is located medially and would be injured with an eversion injury; it is so strong, however, that eversion is more likely to fracture the medial malleolus rather than tear the deltoid ligament.

A 27-year-old female tennis pro injured her ankle during the quarterfinal match. A physical examination at the outpatient clinic revealed a severe inversion sprain of the ankle. Which of the following structures is most commonly damaged in such injuries? A. Medial plantar nerve B. Tibial nerve C. Anterior talofibular ligament D. Posterior talofibular ligament E. Deltoid ligament

A. The common fibular (peroneal) nerve is a branch of the sciatic nerve. It descends on the lateral side of the popliteal fossa before winding around the head of the fibula. It then divides into superficial and deep nerves that supply the lateral and anterior compartments of the leg respectively. Due to its superficial course, it is easily injured in patients with long leg casts (which run from just below the knee). The nerve supplies the dorsiflexors of the leg, the skin of the first web space (via the deep fibular), the evertors of the foot, and the skin of the lateral side of the leg and dorsum of the foot (via the superficial fibular).

A 29-year-old man is brought to the physician for removal of a cast from his left leg. He had sustained a fracture of the left lower extremity 6 weeks prior which was immobilized in a cast that extended from just below the knee to the foot. At the time of injury, there was severe pain but normal strength in the extremity. When the cast was removed, physical examination showed a pronounced left foot drop with paresthesia and sensory loss over the dorsum of the left foot and lateral leg. Injury to which nerve is the most likely cause? A. Common fibular (peroneal) B. Superficial fibular (peroneal) C. Deep fibular (peroneal) D. Sciatic E. Tibial

B. The tendons of the semitendinosus and semimembranosus provide the superior medial border of the popliteal fossa. The semitendinosus inserts with the pes anserinus on the proximal, medial tibia. The semimembranosus inserts on the tibia posteriorly. The biceps femoris forms the superior lateral border of the fossa, as the tendon passes to insertion on the fibula. The plantaris arises from the femur just above the lateral head of the gastrocnemius, passing distally to insert on the calcaneus via the tendo Achilles. The popliteus arises from the tibia and passes superiorly and laterally to insert on the lateral condyle of the femur, with a connection to the lateral meniscus.

A 27-year-old male triathlon competitor complained that he frequently experienced deep pains in one calf that almost caused him to drop out of a regional track-and-field event. Doppler ultrasound studies indicated, and surgical exposure confirmed, the existence of an accessory portion of the medial head of the gastrocnemius that was constricting the popliteal artery. Above the medial head of the gastrocnemius, the superior medial border of the popliteal fossa could be seen. Which of the following structures forms this border? A. Tendon of biceps femoris B. Tendons of semitendinosus and semimembranosus C. Tendon of plantaris D. Adductor hiatus E. Popliteus

D. The deep fibular (peroneal) nerve is responsible for sensation over the first web space of the foot. Dorsiflexion and inversion of the ankle is produced by the muscles supplied by the deep fibular (peroneal) nerve. The nerves responsible for the knee jerk reflex, knee flexion, eversion, and plantar flexion are all located superior to the location of the tumor and will not be damaged. The nerve located in the space between the tibia and fibula is the deep fibular (peroneal) nerve.

A 27-year-old man has had increasing difficulty walking and complained of an area of numbness on the dorsum of his right foot. Examination reveals a hard mass at the anterolateral aspect of his right leg just below the knee. Imaging studies reveal a large bone tumor between the fibula and tibia that is compressing a nerve, accounting for his neurological symptoms. Which of the following is the most likely description of abnormalities on neurological examination? A. Decreased/absent knee jerk reflex and decreased sensation on the medial aspect of the leg B. Weakness of flexion at the knee and decreased sensation of the plantar aspect of the foot C. Weakness of eversion at the ankle and decreased sensation between the first and second toes D. Weakness of inversion, dorsiflexion at the ankle, and decreased sensation between the first and second toes E. Weakness of plantar flexion at the ankle, weakness of toe flexion, decreased sensation of the plantar aspect of the foot

D. The lesser (short) saphenous vein ascends up the middle of the calf from beneath the lateral malleolus, most commonly terminating at the popliteal fossa by piercing the deep fascia and joining the popliteal vein. The popliteal vein is the most superficial of major structures deep to the deep popliteal fascia. The perforating tributaries of the deep femoral vein drain to the deep femoral vein of the posterior compartment of the thigh, thereafter into the femoral vein. The superior medial genicular vein is a tributary to the popliteal vein.

A 27-year-old woman had suffered a penetrating injury in the popliteal region by an object thrown from a riding lawnmower. She was admitted to the emergency department for removal of the foreign object. After making a midline incision in the skin of the popliteal fossa, the surgical resident observed a vein of moderate size in the superficial tissues. What vein would be expected at this location? A. Popliteal vein B. Perforating tributary to the deep femoral vein C. Great saphenous vein D. Lesser (short) saphenous vein E. Superior medial genicular vein

D. The inferior gluteal nerve supplies the gluteus maximus muscle, which extends and laterally rotates the hip joint; through the iliotibial tract, it also extends the knee joint. The superior gluteal nerve supplies the gluteus medius and minimus and tensor fasciae latae muscles, which work together as medial rotators of lower limb. The nerve to the piriformis supplies the piriformis muscle, which laterally rotates the femur with hip extension and abducts the femur with hip flexion.

A 29-year-old construction worker falls onto some rusty wire mesh and suffers a deep laceration to his right buttock. When the ambulance arrives to transport him to the emergency department and it is noted that he has difficulty stepping up into the ambulance with his right leg. Which nerve has probably been damaged? A. Superior gluteal B. Tibial C. Common fibular (peroneal) D. Inferior gluteal E. Nerve to piriformis

E. The lateral femoral cutaneous nerve leaves the pelvis laterally, about 2 cm medial to the anterior superior iliac spine, passing beneath, or through, the inguinal ligament. As a consequence of its site of exit, any tension upon or compression of the inguinal ligament can affect the nerve. If it is thus affected, the individual may feel burning sensations or pain along the lateral aspect of the thigh, which is the region of distribution of the nerve. Obesity, sudden weight loss, wearing a heavy gun belt, wearing trousers that are too tight (Calvin Klein syndrome), or having someone sitting on another's lap for an extended period of time can lead to meralgia paresthetica, the painful lateral thigh. The femoral nerve emerges from beneath the middle of the inguinal ligament and is not usually affected by similar traction or compression. The obturator nerve leaves the pelvis through the obturator canal and enters the thigh deeply in a protected location. It innervates the adductor muscles and supplies sensation on the medial aspect of the thigh. The fibular (peroneal) division of the sciatic nerve supplies the muscles of the anterior and lateral compartments of the leg and provides sensory fibers for the dorsum and lateral side of the foot. The superior gluteal nerve provides motor supply to the gluteus medius and minimus muscles.

A 29-year-old male police officer is examined in a neighborhood clinic, with a complaint of discomfort in the lateral thigh. The physician observes that the policeman is rather overweight and that he is wearing a heavy leather belt, to which numerous objects are attached, including his empty holster. After a thorough physical examination a diagnosis of meralgia paresthetica is confirmed. Which of the following nerves is most likely involved? A. Superior gluteal B. Femoral C. Obturator D. Fibular (peroneal) division of sciatic E. Lateral femoral cutaneous

A. The obturator nerve is a branch of the lumbar plexus that originates from L2 to L4. It descends medial to the psoas on the posterior abdominal wall into the pelvis where it runs along the lateral wall of the lesser pelvis, above and anterior to the obturator vessels. It enters into the medial thigh via the obturator canal (an opening above the obturator membrane) to supply the obturator externus muscle and the adductors of the thigh. The femoral nerve innervates the anterior compartment of the thigh. The inferior gluteal innervates the gluteus maximus muscle, while the superior gluteal innervates the gluteus minimus and medius. The tibial nerve innervates the posterior compartment of the lower limb.

A 29-year-old woman is involved in a car crash and is taken to the emergency department. Radiographs reveal a fracture of her pelvis. During healing of the pelvic fracture, a nerve becomes entrapped in the bone callus. Musculoskeletal examination reveals an inability to adduct the thigh. Which of the following nerves is most likely affected? A. Obturator B. Femoral C. Inferior gluteal D. Superior gluteal E. Tibial

B. Injury to the superior gluteal nerve results in a characteristic motor loss, with paralysis of the gluteus medius and minimus. In addition to their role in abducting the thigh, the gluteus medius and minimus function to stabilize the pelvis. When the patient is asked to stand on the limb of the injured side, the pelvis descends on the opposite side, indicating a positive Trendelenburg test. The gluteal, or lurching, gait that results from this injury is characterized by the pelvis drooping to the unaffected side when the opposite leg is raised. In stepping forward, the affected individual leans over the injured side when lifting the good limb off the ground. The uninjured limb is then swung forward. The gluteus maximus, supplied by the inferior gluteal nerve, is the main muscle responsible for allowing a person to rise to a standing position (extending the flexed hip). Spinal nerve roots L1 and L2 and the femoral nerve are responsible for hip flexion. Injury to the left superior gluteal nerve would result in sagging of the right side of the pelvis when the affected individual stands on the left limb. The hamstring muscles, mainly responsible for flexing the knees to allow a person to sit down from a standing position, are innervated by the tibial branch of the sciatic nerve.

A 30-year-old man suffered a superior gluteal nerve injury in a motorcycle crash in which his right lower limb was caught beneath the bike. He is stabilized in the emergency department. Later he is examined and he exhibits a waddling gait and a positive Trendelenburg sign. Which of the following would be the most likely physical finding in this patient? A. Difficulty in standing from a sitting position B. The left side of the pelvis droops or sags when he attempts to stand with his weight supported just by the right lower limb C. The right side of the pelvis droops or sags when he attempts to stand with his weight supported just by the left lower limb D. Weakened flexion of the right hip E. Difficulty in sitting from a standing position

B. The suprapatellar bursa is found in the anterior surface of the inferior part of the femur under the quadriceps femoris muscle. A high-riding superior patellar fragment lead to the exposure of this bursa. In traumatic episodes following this condition, blood and fat from the knee can enter into the suprapatellar bursa. The popliteal bursa is located behind the knee, away from the site of injury. The superficial infrapatellar bursa is located below the patella, between the patella ligament and the skin. The deep infrapatellar bursa is located below the site of the injury, between the upper part of the tibia and the patella ligament. The gastrocnemius bursa is at the back of the knee, as well as below the level of the injury.

A 30-year-old man was brought to the emergency department after his involvement in a motor vehicle crash. He complained of pain in the right knee and an inability to bear weight. On examination, there were minor abrasions about his body, in addition to a deep, 5 cm oblique laceration over the anterior right knee, which exposed the patella. He was unable to extend the right knee. Radiographs revealed a displaced transverse fracture of the inferior pole of the patella. The superior fragment of the patella appeared to be "high riding" over the anterior surface of the femur. Which of the following most likely occurred? A. Blood and fat from the injury can enter the popliteus bursa B. Blood and fat from the injury can enter the suprapatellar bursa C. Joint fluid can enter the subcutaneous infrapatellar bursa D. The deep infrapatellar bursa will be affected E. The gastrocnemius bursa will not be affected

E. The obturator membrane is a fibrous sheet that almost completely closes the obturator foramen, leaving a small gap, the obturator canal, for the passage of the obturator nerve and vessels as they leave the pelvis to enter the medial thigh. The femoral canal is the small medial compartment for the lymph vessels. It is about 0.5 in (1.3 cm) long, and its upper opening is called the femoral ring. It has following borders: anteriorly the inguinal ligament; posteriorly the superior ramus of the pubis; medially the lacunar ligament; and laterally the femoral vein. A triangular shaped defect in the external oblique aponeurosis lies immediately above and medial to the pubic tubercle. This is known as the superficial inguinal ring. The deep ring is an oval opening in the transversalis fascia and lies about 0.5 in (1.3 cm) above the inguinal ligament, midway between the anterior superior iliac spine and the pubic symphysis. Fossa ovalis, which refers to an oval opening in the superomedial part of the fascia lata of the thigh, lies 3 to 4 cm inferolateral to the pubic tubercle.

A 30-year-old man who is a bodybuilder presents to the physician's office complaining of pain and tingling sensation radiating down the inside of his thigh that was exacerbated upon thigh movement. A hernia through which opening would most likely cause this presentation? A. Femoral ring B. Superficial inguinal ring C. Deep inguinal ring D. Fossa ovalis E. Obturator canal

E. The muscles of the limbs develop from the myotome component of the somites. The somites are condensations of paraxial mesoderm that form after the formation of the trilaminar disc. The lateral plate mesoderm gives rise to the serous membranes, coverings of organs and the heart. The intermediate mesoderm gives rise to the urogenital system and its accessory glands. The chondrification centers precede the formation of the muscles, as it gives the skeletal framework. Neural crest cells give rise to dorsal root ganglia, leptomeninges, Schwann cells, sympathetic ganglia, and the chromaffin cells of the adrenal medulla.

A 30-year-old woman is admitted to the emergency department with complaints of pain to the anterior left thigh. While participating in a 100-meter race, she felt a sudden onset of pain in the anterior mid thigh area and could only limp to the finish line. Physical examination revealed a swollen, tender right thigh anteriorly. Extension of the knee was limited due to pain. Ultrasonography of the area revealed a defect in the fibers of the quadriceps muscle, confirmed by CT scan of the limb. Which of the following is the embryologic origin of the affected structure? A. Lateral plate mesoderm B. Dorsolateral migration of neural crest cells C. Preceded the development of chondrification centers D. Intermediate mesoderm E. Migration of cells from paraxial mesoderm

E. The paraxial mesoderm develops into somites. Limb muscles develop from the ventral myotome of the somites in response to molecular signals. Embryological derivatives of the lateral plate mesoderm include the circulatory and gut wall, body wall lining, and dermis. Derivative of the neural crest cells does not include the limb muscles. Chondrification is associated with cartilage formation and not muscles. The intermediate mesoderm eventually thins out laterally and becomes the mesoderm, which gives the circulatory and gut walls, plus the lining of the body wall and dermis.

A 30-year-old woman is admitted to the emergency department with complaints of pain to the anterior left thigh. While participating in a 100-meter race, she felt a sudden onset of pain in the anterior mid thigh area and could only limp to the finish line. Physical examination revealed a swollen, tender right thigh anteriorly. Extension of the knee was limited due to pain. Ultrasonography of the area revealed a defect in the fibers of the quadriceps muscle, confirmed by CT scan of the limb. Which of the following is the embryologic origin of the affected structure? A. Lateral plate mesoderm B. Dorsolateral migration of neural crest cells C. Preceded the development of chondrification centers D. Intermediate mesoderm E. Migration of cells from paraxial mesoderm

B. The obturator membrane is a thin membrane that covers the obturator foramen except at its superior part. The obturator nerve exits the pelvis and enters into the medial compartment of the thigh by passing through the obturator canal alongside the obturator vessels. Traumatic injuries to the membrane will most likely lead to obturator nerve damage. The obturator nerve supplies motor innervations to the adductor muscles of the thigh (gracilis, obturator externus, adductor longus, adductor brevis and a portion of the adductor magnus). It also provides sensory innervation to the medial aspect of the thigh. Urinary and fecal incontinence is mediated by autonomic nerves and the pudendal nerve. Both nerves have no relationship with the obturator membrane. The gluteus medius and minimus muscles are the main hip abductors. They also stabilize the hip on the swing-side during motion. These muscles are supplied by the superior gluteal nerve, which leaves the pelvis through the greater sciatic foramen above the piriformis muscle. Flexors of the hip found in the anterior compartment of the thigh are innervated by the femoral nerve, which has no relationship with the obturator membrane. The sciatic nerve supplies the muscles in the posterior compartment of the thigh and also sends cutaneous innervations to the skin of the posterior thigh. It enters the posterior compartment of the thigh from the gluteal region.

A 30-year-old woman was admitted to the emergency department after being involved in a motor vehicle crash. The patient complained of pain to the right hip and knee. During physical examination, there is no deformity of the lower limb, but there is tenderness over the right ischiopubic ramus. Pelvic radiographs revealed an inferiorly displaced fracture of the right superior and inferior pubic rami with dislocation of the right sacroiliac joint and pubic symphysis. The patient was referred to the orthopedics team, which had a high suspicion of rupture of the right obturator membrane. What clinical findings are most likely to be present in this patient? A. Urinary and fecal incontinence and diminished sensation over the perineum B. Weak adduction of the hip and diminished sensation over the upper medial thigh C. Weak abduction of the hip and positive Trendelenburg sign D. Weak flexion of the hip and diminished sensation over the anterior thigh and medial leg E. Weak extension of the hip and diminished sensation over the posterior thigh

D. The sural nerve is formed by contributions from the tibial nerve and a branch from the common fibular (peroneal) nerve. It provides sensation for the lower lateral portion of the calf and continues beneath the lateral malleolus as the lateral cutaneous nerve of the foot. It is often used for nerve grafting procedures as well as biopsied for diagnostic purposes. When it is grafted to the "living end" of a cut motor or sensory nerve, the severed nerve processes within the "living" nerve grow into the sural nerve sheath, using it as a guide to the distal, surgically anastomosed nerve. Thus, axons from a branch of a functional motor nerve can grow to reinnervate paralyzed muscles. In this case, the surgeon would connect portions of the sural nerve to the functional facial nerve, tunnel it to the opposite side of the face, and join it surgically to the branches of the paralyzed nerve, where it would grow through the now empty nerve sheaths (due to Wallerian degeneration) to the muscles. Growth and reinnervation usually occur at a rate of 1 mm/day (or 1 inch/month) so the time estimated before reinnervation is based on the distance the regenerating fibers need to traverse. The tibial nerve supplies muscles and sensation to the calf and plantar surface of the foot. The common fibular (peroneal) nerve innervates the lateral and anterior compartment muscles and sensation to the dorsum of the foot. The saphenous nerve accompanies the great saphenous vein on the medial side of the leg and foot.

A 31-year-old woman presents to the department of surgery with a complaint of facial paralysis (Bell's palsy), which had appeared a year earlier and had resulted in paralysis of muscles of one side of her face. The chief of plastic surgery recommends a nerve graft, taking a cutaneous nerve from the lower limb to replace the defective facial nerve. The surgery is successful. Six months after the procedure, there is restoration of function of previously paralyzed facial muscles. There is an area of skin on the back of the leg laterally and also on the lateral side of the foot that has no sensation. What nerve was used in the grafting procedure? A. Superficial fibular (peroneal) B. Tibial C. Common fibular (peroneal) D. Sural E. Saphenous

B. Pott's fracture is a rather archaic term for a fracture of the fibula at the ankle. The term is often used to indicate a bimalleolar fracture of fibula and tibia, perhaps with a tear in the medial collateral ligament, allowing the foot to be deviated laterally. (The medial malleolus will often break before the deltoid ligament tears.) This fracture is also known as Dupuytren's fracture. The fracture results from abduction and lateral rotation of the foot in extreme eversion. There can also be fracture of the posterior aspect of the distal tibia. The spring ligament, also known as the plantar calcaneonavicular ligament, extends from the calcaneus to the navicular bone and is a part of the medial longitudinal arch. This ligament would not be affected in eversion or inversion of the ankle. The plantar ligament, which is composed of the long and short plantar ligaments, supports the lateral longitudinal arch of the foot and would therefore not be affected by inversion or eversion of the foot. The calcaneofibular ligament runs from the calcaneus to the fibula. It would be injured during inversion of the foot, not in eversion, as is the case in a Pott's fracture

A 32-year-old male basketball player comes down hard on his ankle. He is admitted to the outpatient clinic, and radiologic examination reveals a Pott's fracture. What ligament is most likely injured? A. Calcaneofibular ligament B. Deltoid ligament C. Spring ligament D. Plantar ligament E. Long plantar ligament

B. The lower portion of the vastus medialis inserts upon the medial aspect of the patella and draws it medially, especially in the last quarter of extension—during which it is especially palpable in contraction. This lower portion of the muscle is referred to as the vastus medialis obliquus (VMO). Increasing the strength of this muscle can lessen the lateral dislocation of the patella. The rectus femoris arises from the anterior inferior iliac spine and lip of the acetabulum and draws the patella vertically upward, as does the vastus intermedius.

A 34-year-old woman has a direct blow to the patella by the dashboard of the vehicle during an automobile crash. The woman is admitted to the emergency department and radiologic examination reveals patellofemoral syndrome. This type of syndrome is characterized by lateral dislocation of the patella. Which of the following muscles requires strengthening by physical rehabilitation to prevent future dislocation of the patella? A. Vastus lateralis B. Vastus medialis C. Vastus intermedius D. Rectus femoris E. Patellar ligament

C. If the needle is inserted about 1.5 cm lateral to the maximal femoral pulse, it will intersect the femoral nerve in most cases. (Fluoroscopic or ultrasound guidance is advisable to avoid iatrogenic errors.) The deep inguinal ring is located about 4 cm superolateral to the pubic tubercle and very close to the origin of the inferior epigastric vessels from the external iliac artery and vein. The approximate site of exit of the lateral femoral cutaneous nerve from the abdomen is 1.5 cm medial to the anterior superior iliac spine. Injections 1.5 cm medial to the femoral artery pulse will enter the femoral vein. Midway between the anterior superior iliac spine and the pubic symphysis can vary approximately 1.5 cm either medial or lateral from the femoral artery.

A 32-year-old man is admitted to the emergency department after a car collision. Radiologic examination reveals a distal fracture of the femur. The patient is in severe pain, and a femoral nerve block is administered. What landmark is accurate for localizing the nerve for injection of anesthetics? A. 1.5 cm superolateral to the pubic tubercle B. 1.5 cm medial to the anterior superior iliac spine C. 1.5 cm lateral to the femoral pulse D. 1.5 cm medial to the femoral pulse E. Midway between the anterior superior iliac spine and pubic symphysis

C. The bone to which the injured ligament attaches is the calcaneus. The navicular bone, located medially in the foot, articulates posteriorly with the head of the talus and anteriorly with the cuneiform bones. The cuboid bone of the lateral longitudinal arch articulates posteriorly with the calcaneus. The talus articulates with the tibia and fibula in the ankle joint mortise.

A 32-year-old man is admitted to the emergency department after an injury to his foot while playing football with his college friends. An MRI examination reveals multiple tendinous tears (Fig. 5-8). Which of the following bones is associated with the muscle tears? A. Navicular B. Cuboid C. Calcaneus D. Sustentaculum tali E. Talus

C. The deep fibular (peroneal) nerve is a branch of the common fibular (peroneal) nerve and begins at about the level of the neck of the fibula, between it and the fibularis (peroneus) longus. This nerve supplies the extensors of the foot (extensor digitorum longus, fibularis [peroneus] tertius, extensor hallucis longus, tibialis anterior, extensor digitorum brevis, and extensor hallucis brevis). It innervates the first web space of the foot. Fracture of the head of the fibula can damage this nerve, resulting in a high stepping gait and numbness over the dorsum and first web space of the foot. Muscles in the posterior compartment of the leg are involved in planter flexion. These muscles are innervated by the tibial nerve, which is a continuation of the sciatic nerve; its medial cutaneous branch supplies sensation to the posteromedial side of the leg. Waddling gait and numbness at the anterolateral side of the leg are associated with the superficial fibular and the lateral sural cutaneous nerves.

A 32-year-old man is brought to the emergency department with complaints of pain to the left ankle and knee. The patient recalls that during a football game, his left foot landed in a hole as he was running on an uneven dirt field. The ankle was externally rotated and everted while the knee twisted medially. He was unable to bear weight subsequently. During physical examination, the right ankle is swollen and there is exquisite tenderness over the right medial malleolus and the proximal lateral leg. Radiologic examination of the right lower limb reveals a displaced fracture of the neck of right fibula and a comminuted fracture of the tibial plafond and medial malleolus. Which of the following describes the most likely consequences of this injury? A. Weak "push-off" while walking and numbness over the posteromedial leg B. Weak ankle eversion and numbness over the dorsum of the foot C. High stepping gait and numbness over the dorsum and first web space of the foot D. Waddling gait and inability to feel a pin prick over the anterolateral leg E. Swing-out gait and numbness over the medial leg

C. Forceful external rotation and eversion of the ankle often leads to this type of injury as the bony components are pushed apart forcefully. It is commonly referred to as a Pott fracture (the medial malleolus is pulled forcefully by the strong deltoid ligament as the talus moves laterally, causing a fracture of the lateral malleolus). Inversion, extreme plantar flexion, and forceful dorsiflexion cause a rupture of the ligaments. Direct upward force of the talus is usually due to a fall from great height and will damage the spine and calcaneus.

A 32-year-old man was brought to the emergency with complaints of pain to the left ankle and leg. During a game of football, his left foot landed in a hole as he was running on an uneven dirt field. He was unable subsequently bear weight on his left limb. On examination, the right ankle was swollen, with exquisite tenderness over the right medial malleolus and over the distal third of the lateral right leg. Radiographs of the right lower limb revealed an inferiorly displaced fracture of the right medial malleolus and a spiral fracture of the distal third of the right fibula. Which of the following describes the most likely mechanism of this injury? A. Forceful inversion of the ankle B. Direct upward force from the talus into the tibial plafond C. Forceful external rotation and eversion of the ankle D. Forceful dorsiflexion of the foot E. Extreme plantar flexion of the foot

A. The gluteus maximus is innervated by the inferior gluteal nerve, and this muscle is responsible for extension and lateral rotation of the thigh. It is the primary muscle that extends the flexed hip and is used to rise from a seated position. The gluteus minimus is innervated by the superior gluteal nerve and is responsible for abduction of the thigh. Hamstring muscles are innervated by the tibial portion of the sciatic nerve, and these are responsible for extension of the thigh and flexion of the leg. The iliopsoas muscle is innervated by L1 and L2 and the femoral nerve, and flexes the thigh. The obturator internus is innervated by the nerve to the obturator internus and is a lateral rotator of the thigh.

A 32-year-old patient received a badly placed intramuscular injection to the posterior part of his gluteal region. The needle injured a motor nerve in the area. Later, he had great difficulty rising to a standing position from a seated position. Which muscle was most likely affected by the injury? A. Gluteus maximus B. Gluteus minimus C. Hamstrings D. Iliopsoas E. Obturator internus

B. The lateral femoral circumflex artery is a branch of the femoral artery close to the hip joint. It gives a branch that runs down the lateral aspect of the thigh and joins the genicular anastomosis via the superior lateral genicular artery. The medial circumflex femoral artery does not provide any branches that descend toward the knee. The anterior and posterior tibial arteries are the terminal branches of the popliteal artery and would not receive any blood if the popliteal is damaged. The fibular (peroneal) artery is a branch of the posterior tibial artery.

A 37-year-old unconscious man is rushed to the emergency department after being retrieved from a motor vehicle crash. On physical examination bruising and obvious deformity is seen over his left knee joint. Radiological studies showed a posteriorly dislocated supracondylar fracture with severe compression of the popliteal artery. Which of the following arteries would ensure adequate blood supply to the leg and foot in this patient? A. Medial femoral circumflex B. Lateral femoral circumflex C. Anterior tibial artery D. Posterior tibial artery E. Fibular (peroneal) artery

D. Inflammation of the plantar aponeurosis is referred to as plantar fasciitis. Plantar fasciitis is a common clinical condition that results from tearing or inflammation of the tough band of tissue stretching from the calcaneus to the ball of the foot (the plantar aponeurosis). It happens frequently to people who are on their feet all day, such as mail carriers, or engaged in athletics, especially in running and jumping. The pain of plantar fasciitis is usually most significant in the morning, just after you get up from bed and begin to walk. Rest, orthotics, night splints, and antiinflammatory medications are employed in treatment. A Morton's neuroma is a painful lesion of the neural interconnection of the medial and lateral plantar nerves between the third and fourth toes. An eversion sprain of the ankle can break the medial malleolus or tear the deltoid ligament. An inversion sprain commonly injures the fibulocalcaneal ligament or anterior talofibular ligament.

A 34-year-old male distance runner visits the outpatient clinic with a complaint of pain he has suffered in his foot for the past week. The clinical examination indicates that the patient has an inflammation of the tough band of tissue stretching from the calcaneus to the ball of the foot. Which of the following conditions is most characteristic of these symptoms? A. Morton's neuroma B. Ankle eversion sprain C. Tarsal tunnel syndrome D. Plantar fasciitis E. Inversion sprain of the ankle

B. The deep fibular (peroneal) nerve supplies the dorsiflexors of the foot, including the extensor hallucis longus and extensor digitorum longus. It also supplies the tibialis anterior, an invertor of the foot. This nerve has sensory distribution only to the skin between the first two toes. The common fibular (peroneal) nerve supplies not only the preceding muscles but also the evertors of the foot and provides sensation for most of the dorsum of the foot. The sciatic nerve innervates the muscles of the posterior thigh and all muscles of the leg and foot, in addition to providing sensory supply in those areas. The superficial fibular (peroneal) nerve innervates the evertors of the foot and provides sensation for the dorsum of the foot. The tibial nerve is the nerve for muscles of the posterior compartment of the leg and also of the plantar region and supplies sensation over the medial aspect of the leg posteriorly and the plantar surface of the foot and toes

A 34-year-old male long-distance runner complained to the team physician of swelling and pain of his shin. Skin testing in a physical examination showed normal cutaneous sensation of the leg. Muscular strength tests showed marked weakness of dorsiflexion and impaired inversion of the foot. Which nerve serves the muscles involved in the painful swelling? A. Common fibular (peroneal) B. Deep fibular (peroneal) C. Sciatic D. Superficial fibular (peroneal) E. Tibial

C. When the popliteus contracts, it rotates the distal portion of the femur in a lateral direction. It also draws the lateral meniscus posteriorly, thereby protecting this cartilage as the distal femoral condyle glides and rolls backward, as the knee is flexed. This allows the knee to flex and therefore serves in unlocking the knee. The biceps femoris is a strong flexor of the leg and laterally rotates the knee when it is in a position of flexion. The gastrocnemius is a powerful plantar flexor of the foot. The semimembranosus, similar to the biceps femoris, is a component of the hamstring muscles and is involved in extending the thigh and flexing the leg at the knee joint. The rectus femoris is the strongest quadriceps muscle in extending the leg at the knee.

A 34-year-old male power lifter visits the outpatient clinic because he has difficulty walking. During physical examination it is observed that the patient has a problem unlocking the knee joint to permit flexion of the leg. Which of the following muscles is most likely damaged? A. Biceps femoris B. Gastrocnemius C. Popliteus D. Semimembranosus E. Rectus femoris

D. Plantar fasciitis is a common clinical condition that results from tearing or inflammation of the tough band of tissue stretching from the calcaneus to the ball of the foot (the plantar aponeurosis). It usually happens to people who are on their feet frequently or engaged in athletics, especially running and jumping. Plantar fasciitis is usually most painful in the morning, just after getting up from bed and beginning to walk. Rest, orthotics, night splints, and antiinflammatory medications are employed in treatment. A Pott's fracture is a bimalleolar fracture, specifically a fracture of the distal end of the fibula (lateral malleolus) and medial malleolus, with outward displacement of the foot. Dupuytren's fracture involves fracture of the distal fibula with dislocation of the foot. Each of these fractures occurs due to sudden and forceful eversion of the foot.

A 34-year-old male runner visits the outpatient clinic complaining of pain in his foot for the past week. Physical examination reveals inflammation of the tough band of tissue stretching from the calcaneus to the ball of the foot. Which of the following conditions is characteristic of these symptoms? A. Pott's fracture B. Dupuytren fracture C. Tarsal tunnel D. Plantar fasciitis E. Rupture of spring ligament

A. In a femoral hernia, abdominal contents are forced through the femoral ring, which is just lateral to the lacunar ligament (of Gimbernat) and just medial to the femoral vein. The femoral vein would be found immediately lateral to the femoral hernia. This is correct in most cases because in the majority of people, the femoral vein is found more medial to both the femoral artery and nerve in the femoral triangle. The adductor longus muscle as well as the pectineus muscle would be found deep and medial to the hernia.

A 34-year-old man is lifting heavy weights while doing squats. Unfortunately, while making a maximal effort, he drops the weight and immediately grabs at his upper thigh, writhing in pain. The man is admitted to the emergency department and during physical examination is diagnosed with a femoral hernia. What reference structure would be found immediately lateral to the herniated structures? A. Femoral vein B. Femoral artery C. Pectineus muscle D. Femoral nerve E. Adductor longus muscle

D. Bipartite patella is a normal variant of an unfused superolateral secondary ossification center, which can easily be mistaken for a fracture on a radiograph. The subcutaneous prepatellar bursa can become painfully enlarged with acute or chronic compression, as in crawling about on the knees. Osgood-Schlatter disease is painful involvement of the patellar ligament on the tibial tuberosity, commonly in children 10 to 14 years of age. The medial retinaculum is an expanded portion of the vastus medialis tendon toward the patella.

A 34-year-old man visits the outpatient clinic for an annual checkup. A radiographic examination of his knees is shown in Fig. 5-11. Physical examination reveals no pathology or pain to his knees. The patient has no past history of any knee problems. What is the most likely diagnosis? A. Enlarged prepatellar bursa B. Osgood-Schlatter disease C. Normal intercondylar eminence D. Bipartite patella E. Injury to lateral meniscus

A. The tendon of the obturator internus leaves the pelvic cavity by passing through the lesser sciatic foramen, wrapping around the lesser sciatic notch, changing direction by about 90 degrees. It is joined there by the superior and inferior gemelli and inserts with them on the upper portion of the greater trochanter. The obturator externus arises on the external surface of the pubic bone and obturator membrane and inserts on the greater trochanter. The quadratus femoris arises from the ischial tuberosity and inserts on the intertrochanteric line of the femur. The gluteus medius and minimus insert together on the lateral aspect of the greater trochanter.

A 37-year-old woman had been suffering for months from piriformis entrapment syndrome, which was not relieved by physical therapy. Part of the sciatic nerve passed through the piriformis, and a decision was made for surgical resection of the muscle. When the area of entrapment was identified and cleared, a tendon could be seen emerging through the lesser sciatic foramen, at first hidden by two smaller muscles and several nerves and vessels destined for the region of the perineum. The tendons of which of the following muscles pass through this opening? A. Obturator internus B. Obturator externus C. Quadratus femoris D. Gluteus minimus E. Gluteus medius

A. Morton's neuroma most commonly involves compression (and possible enlargement) of an anastomosing branch that connects the medial and lateral plantar nerves between the third and fourth toes. The pain can be severe. The medial plantar nerve provides sensation for the medial three and a half toes; the lateral plantar nerve supplies the little toe and half of the fourth toe. The neural interconnection can be compressed between the transverse metatarsal ligament and the floor. Women are 10 times more likely than men to be afflicted with this problem, most likely due to wearing shoes that put excessive stress on the forefoot. In about 80% of cases the pain can be eased with different (less confining) shoes or cortisone injections.

A 39-year-old woman is admitted to the emergency department with a painful foot. Radiologic examination reveals a Morton's neuroma. What is the most typical location of this neuroma? A. Between the third and fourth metatarsophalangeal joints B. Between the second and third metatarsophalangeal joints C. Between the first and second metatarsophalangeal joints D. Between the fourth and fifth metatarsophalangeal joints E. In the region of the second, third, and fourth metatarsophalangeal joints

A. Any superficial inflammation in the gluteal region drains into the superficial horizontal group of inguinal nodes. The vertical group drains the lower limbs, whereas deep gluteal injuries drain into the superior and inferior gluteal nodes.

A 39-year-old woman who is a school teacher unwittingly sits on a thumbtack a student placed on her chair. Her left buttock becomes painful and inflamed. Which group of nodes will first receive lymph from the infected wound? A. Superficial horizontal group B. Superficial vertical group C. Superior and inferior gluteal nodes D. External iliac E. Deep inguinal

D. A positive valgus stress test indicates injury to the medial collateral ligament. Injuries to this ligament usually involve the anterior cruciate ligament. The femur is usually pushed posteriorly during stair climbing, an action that is opposed by a normal anterior cruciate ligament. Injury to the anterior cruciate ligament results in posterior displacement of the femur in relation to the tibia with difficulty climbing stairs. Extension of the knee is done mainly by the quadriceps femoris muscle. The posterior thigh muscles provide flexion of the knee. Gravity pushes the femur forward while walking down a flight, which is stabilized by the posterior cruciate ligament, which is not damaged in this case. The patient has no difficulty walking down the stairs; the posterior cruciate ligament stabilizes the knee during this action.

A 40-year-old man was brought to the emergency department after being struck by a car. He complained of pain to the left knee and leg and inability to bear weight on the affected limb. On examination, there was a joint effusion of the left knee, and tenderness over the medial and lateral side of the joint. A valgus stress test was positive while the varus stress test was negative. An MRI of the left knee showed complete disruption of multiple ligament support structures of the knee. What other symptoms are most likely possible in this patient? A. Inability to extend the knee B. Inability to flex the knee C. Instability of the knee when walking down a flight of stairs D. Instability of the knee when walking up a flight of stairs E. Excessive extension of the knee and difficulty walking down stairs

A. The plantar calcaneonavicular (spring) ligament supports the head of the talus and maintains the longitudinal arch of the foot. A fracture of the cuboid bone would not disrupt the longitudinal arch of the foot. Interruption of the plantar aponeurosis is not the best answer because this aponeurosis provides only passive support, unlike the spring ligament. A sprain of the anterior talofibular ligament would result from an inversion injury of the ankle and would not disrupt the longitudinal arch of the foot. A sprain of the deltoid ligament results from eversion of the ankle joint and would not disrupt the longitudinal arch of the foot.

A 41-year-old man is admitted to the emergency department with a swollen and painful foot. Radiologic examination reveals that the head of the talus has become displaced inferiorly, thereby causing the medial longitudinal arch of the foot to fall. What would be the most likely cause in this case? A. Tearing of the plantar calcaneonavicular (spring) ligament B. Fracture of the cuboid bone C. Interruption of the plantar aponeurosis D. Sprain of the anterior talofibular ligament E. Sprain of the deltoid ligament

49 D. The second perforating branch of the profunda femoris (deep femoral) artery commonly provides the nutrient artery to the femur, a vessel that passes through a rather large foramen to enter the proximal part of the shaft. The deep circumflex branch of the external iliac passes around the medial aspect of the iliac crest, also supplying the lower lateral part of the anterior abdominal wall. The acetabular branch of the obturator artery supplies tissues in the hip socket, usually including a branch to the ligament of the head of the femur. The lateral circumflex femoral branch of the deep femoral artery supplies the vastus lateralis muscle. The medial circumflex femoral branch of the deep femoral artery supplies proximal adductor musculature and the region of the hip joint, including the neck and head of the femur

A 42-year-old male sign painter is admitted to the emergency department after falling to the sidewalk from his ladder. Radiologic examination reveals a fracture of the proximal femur. Which of the following arteries supplies the proximal part of the femur? A. Deep circumflex iliac B. Acetabular branch of obturator C. Lateral circumflex femoral D. A branch of profunda femoris E. Medial circumflex femoral

A. The lumbosacral trunk consists of fibers from a portion of the ventral ramus of L4 and all of the ventral ramus of L5 and provides continuity between the lumbar and sacral plexuses. The deep fibular (peroneal) nerve receives supply from segments of L4, L5, and S1. It supplies the extensor hallucis longus and extensor digitorum longus, the main functions of which are extension of the toes and dorsiflexion of the ankle. L5 is responsible for cutaneous innervation of the dorsum of the foot. Injury to L4 would affect foot inversion by the tibialis anterior. Injury to L4 in the lumbosacral trunk would not affect the patellar tendon reflex, for these fibers are delivered by the femoral nerve. Therefore, an injury to the lumbosacral trunk would result in all of the patient's symptoms. Nerve root injury at L5 and S1 would result in loss of sensation of the plantar aspect of the foot and motor loss of plantar flexion, with weakness of hip extension and abduction. The fibularis (peroneus) longus and brevis are supplied by the superficial fibular (peroneal) nerve, which is composed of fibers from segments L5, S1, and S2; these are responsible for eversion of the foot (especially S1). Transection of the fibular (peroneal) division of the sciatic nerve would result in loss of function of all the muscles of the anterior and lateral compartments of the leg. Injury to the sciatic nerve will affect hamstring muscles and all of the muscles below the knee. Injury to the tibial nerve causes loss of plantar flexion and impaired inversion.

A 42-year-old man is admitted to the emergency department after his automobile hit a tree. He is treated for a pelvic fracture and several deep lacerations. Physi- cal examination reveals that dorsiflexion and inversion of the left foot and extension of the big toe are very weak. Sensation from the dorsum of the foot, skin of the sole, and the lateral aspect of the foot has been lost and the patellar reflex is normal. The foot is everted and plantar flexed. Which of the following structures is most likely injured? A. The lumbosacral trunk at the linea terminalis B. L5 and S1 spinal nerves torn at the intervertebral foramen C. Fibular (peroneal) division of the sciatic nerve at the neck of the fibula D. Sciatic nerve injury at the greater sciatic foramen ("doorway to the gluteal region") E. Tibial nerve in the popliteal fossa

B. The superficial inguinal nodes are located near the saphenofemoral junction and drain the superior thigh region. The vertical group receives lymph from the superficial thigh, and the horizontal group receives lymph from the gluteal regions and the anterolateral abdominal wall. The deep inguinal lie deep to the fascia lata and receive lymph from deep lymph vessels (popliteal nodes). The external and internal iliac nodes first receive lymph from pelvic and perineal structures.

A 42-year-old man is bitten on his posterior thigh by a dog. The superficial wound is sutured in the emergency department. Four days later the patient returns to the hospital with high fever and swollen lymph nodes. Which group of nodes first receives lymph from the infected wound? A. External iliac B. Vertical group of superficial inguinal C. Deep inguinal D. Horizontal group of superficial inguinal E. Internal iliac

D. The ligament of the head of the femur conveys a small blood vessel for supply of the head of the femur (primarily in childhood). The ligament is stretched during abduction and lateral rotation of the hip joint and has an important role in stabilizing an infant's hip joint before walking. It has the potential to increase stability of the joint in hip reconstruction in developmental hip dysplasia in the pediatric population. The strength of this ligament is comparable to the anterior cruciate ligament of the knee. The iliofemoral ligament (the inverted "Y-shaped ligament of Bigelow") on the anterior aspect of the hip bone resists hyperextension of the hip joint. The pubofemoral ligament arises from the pubic bone and is located on the inferior side of the hip joint; it resists abduction of the joint. The ischiofemoral ligament is a triangular band of strong fibers that arises from the ischium and winds upward and laterally over the femoral neck, strengthening the capsule posteriorly. The transverse acetabular ligament attaches to the margins of the acetabular notch and provides origin for the ligament of the head of the femur. The transverse acetabular ligament is fibrous, not cartilaginous, but is regarded as part of the acetabular labrum.

A 42-year-old mother of three children visits the outpatient clinic complaining that her youngest son cannot walk yet even though he is 4 years old. Radiologic and physical examinations reveal an unstable hip joint. Which of the following ligaments is responsible for stabilization of the hip joint in childhood? A. Iliofemoral B. Pubofemoral C. Ischiofemoral D. Ligament of the head of the femur E. Transverse acetabular ligament

E. The vastus lateralis muscle is located on the lateral aspect of the thigh. The distal portion of this muscle lies superficial to the proximal part of the lateral aspect of the joint capsule of the knee. When a needle is inserted superiorly and laterally to the patella, it penetrates the vastus lateralis muscle on its course to the internal capsule. The short head of biceps femoris has its origin on the posterior aspect of the femur, merges with the long head of the biceps femoris, and inserts on the head of the fibula. The rectus femoris passes longitudinally on the medial aspect of the femur and inserts on the tibial tuberosity, via the patellar tendon, or quadriceps tendon. A needle inserted laterally to the patella would not penetrate this muscle. The sartorius originates on the anterior superior iliac spine and forms part of the pes anserinus, which inserts on the medial aspect of the proximal part of the tibia. A needle inserted laterally to the patella would not penetrate this muscle.

A 43-year-old man visits the outpatient clinic with a painful, swollen knee joint. The patient's history reveals chronic gonococcal arthritis. A knee aspiration is ordered for bacterial culture of the synovial fluid. A standard suprapatellar approach is used, and the needle passes from the lateral aspect of the thigh into the region immediately proximal to and deep to the patella. Through which of the following muscles would the needle pass? A. Adductor magnus B. Short head of biceps femoris C. Rectus femoris D. Sartorius E. Vastus lateralis

A. Gracilis due to its shape, size, and more importantly the nature of neurovascular supply is used very commonly in reconstructive surgery as a free functioning autograft. Also, the other adductors of the thigh compensate for the absence of the gracilis. For similar reasons the remaining muscles are not good candidates during reconstructive surgery of the upper limb.

A 43-year-old victim of a drunk driving car crash is undergoing reconstructive arm surgery. The surgeon performs an autograft using a weak adductor of the leg located superficially on the medial side of the thigh. Which muscle is most likely being harvested to perform this reconstruction? A. Gracilis B. Sartorius C. Rectus femoris D. Vastus lateralis E. Vastus medialis

A. When the anterior cruciate ligament is torn, the tibia can be slightly displaced anteriorly from the area of the knee joint by pulling firmly with both hands upon the leg, with the patient in a seated position. This is a positive anterior drawer sign.

A 50-year-old man is admitted to the emergency department after a car crash. An MRI examination reveals an injured anterior cruciate ligament. Physical examination reveals a positive drawer sign. Which of the following signs is expected to be present during physical examination? A. The tibia can be slightly displaced anteriorly B. The tibia can be slightly displaced posteriorly C. The fibula can be slightly displaced posteriorly D. The fibula can be slightly displaced anteriorly E. The tibia and fibula can be slightly displaced anteriorly

C. Entrapment compression of all or part of the sciatic nerve by the piriformis can mimic disc herniation, most commonly resembling compression of spinal nerve S1. This results in pain down the posterior aspect of the thigh and leg and the lateral side of the foot. In this case, loss of sensation over the dorsum of the foot and weakness of foot extension, in addition to eversion, indicate that more than S1 is involved. Foot drop would be anticipated with fibular (peroneal) nerve involvement. As noted also in a previous question, compression of the common fibular (peroneal) division of the sciatic nerve by the piriformis gives rise to the clinical condition known as piriformis entrapment. This condition is associated with point pain in the gluteal area, pain in the posterior part of the limb, and possible weakness of muscles in the lateral and anterior compartments of the leg. It can be confused with herniated disc (L5) compression of S1 and sciatica. Paralysis of plantar flexion occurs with a lesion of the tibial division of the sciatic nerve or the tibial nerve. Paralysis of the quadriceps is associated with pathology of the femoral nerve. Clonic contraction of the adductors could result from obturator nerve problems

A 43-year-old woman is examined by a neurologist, to whom she complains of pain in her lower limb of 6 months' duration. She has pain in the gluteal area, thigh, and leg. The neurologist observes reduced sensation over the dorsum and lateral side of the involved foot and some weakness in foot dorsiflexion and eversion. A diagnosis of a piriformis entrapment syndrome is made, with compression of the fibular (peroneal) division of the sciatic nerve. Which of the following conditions did the neurologist also most likely find during her physical examination of the patient? A. Paralysis of plantar flexion B. Instability of the knee, due to paralysis of the quadriceps femoris C. Foot drop D. Spasm or clonic contractures of the adductor musculature of the thigh E. Loss of sensation in the gluteal area, by paralysis of anterior cluneal nerves

A. The gluteal region (buttocks) is a common site for intramuscular injection of drugs, particularly if the volume of the injection is large. To avoid injury to the underlying sciatic nerve, the injection should be given well forward on the upper outer quadrant of the buttock (superolateral quadrant). The patient is showing the Trendelenburg gait pattern (or gluteus medius lurch), which is caused by weakness of the gluteus medius and minimus muscles. These muscles are supplied by the superior gluteal nerve (L4, L5, S1), which emerges from the greater sciatic notch above the upper border of the piriformis and immediately disappears beneath the posterior border of the gluteus medius and runs forward between the gluteus medius and minimus. Intramuscular injection in the upper inner quadrant (superomedially) is most likely to damage this nerve. The sciatic nerve is most likely damaged in the inferomedial quadrant of the buttock.

A 43-year-old woman receives deep intramuscular injections for the past week for treatment of a sexually transmitted disease. She complains to her doctor that she has difficulty walking. During physical examination her right hip drops every time she raises her right foot. Which of the following injection locations will most likely correspond with the clinical presentation of this patient? A. Superomedial quadrant of the buttock B. Superolateral quadrant of the buttock C. Inferomedial quadrant of the buttock D. Inferolateral quadrant of the buttock E. Posterior thigh

B. The obturator nerve innervates the adductor muscles, including the gracilis, pectineus, and obturator externus. The tibial nerve supplies the calf muscles and intrinsic muscles in the plantar portion of the foot. The inferior gluteal nerve innervates the gluteus maximus; the superior gluteal nerve supplies the gluteus medius and minimus and tensor fasciae latae. The femoral nerve provides motor supply to the quadriceps femoris, sartorius, and, in some cases, the pectineus. This gait pattern is characteristic of hypertonia in the lower limb. As a result these areas become flexed to various degrees, giving the appearance of crouching, while tight adductors produce extreme adduction

A 45-year-old intoxicated man was struck by a tour bus while walking in the middle of the street. The man was admitted to the emergency department and during physical examination was diagnosed with "adductor gait," in which an individual crosses one limb in front of the other, due to powerful hip adduction. Which of the following nerves was most likely involved in this condition? A. Tibial B. Obturator C. Inferior gluteal D. Superior gluteal E. Femoral

B. The posterior cruciate ligament tightens in flexion of the knee. It can be damaged by posterior displacement of the tibia upon the femur. With the patient seated, a rupture of the ligament can be demonstrated by the ability to push the tibia posteriorly under the femur. This is called the posterior drawer sign because it's similar to pushing in a desk drawer. The anterior cruciate ligament resists knee hyperextension. The lateral collateral ligament is a thick, cordlike band that passes from the lateral femoral condyle to the head of the fibula. It is located external to the capsule of the knee joint. The lateral meniscus is a nearly circular band of fibrocartilage that is located laterally within the knee joint. It is less frequently injured than the medial meniscus because it is not attached to the joint capsule or other ligaments. The patellar ligament is the heavy, ligamentous band of insertion of the quadriceps muscle to the tibial tuberosity. GAS 584-5

A 45-year-old is admitted to the hospital after his left leg impacted a fence post when he was thrown from a powerful four-wheel all-terrain vehicle. Radiologic examination reveals posterior displacement of the tibia upon the femur. Which of the following structures was most likely injured? A. Anterior cruciate ligament B. Posterior cruciate ligament C. Lateral collateral ligament D. Lateral meniscus ligament E. Patellar ligament

A. All the branches of the posterior femoral cutaneous nerve are cutaneous. It arises from the dorsal divisions of the first and second and the ventral divisions of the second and third sacral nerves and travels through the greater sciatic foramen beneath the piriformis muscle to innervate the shin over the lower parts of the gluteus maximus muscle through the inferior clunial nerves and the posterior surface of the thigh and leg and perineum via its perineal branches. The lateral femoral cutaneous nerve, which innervates the lateral surface of the thigh, is an incorrect choice; the obturator, sciatic, and femoral nerves all have both motor and sensory branches and can be eliminated, since no motor deficits were described.

A 45-year-old man after being diagnosed with a posterior acetabular fracture is taken to the operating room to repair the fracture. During the neurological examination the physician notices loss of sensation to the skin of the inferior half of the buttocks, posterior, and upper medial thigh. The patient had a normal neurovascular examination preoperatively. Which of the following nerves was mostly likely damaged during the operation? A. Posterior femoral cutaneous B. Obturator nerve C. Sciatic D. Femoral E. Lateral femoral cutaneous

E. The deep fibular (peroneal) nerve along with the superficial fibular (peroneal) nerve are branches of the common fibular nerve. The deep fibular (peroneal) nerve innervates muscles of the anterior compartment of the leg dorsiflexors of the foot and the skin between the great toe and second toes, while the superficial fibular nerve innervates the lateral compartment muscles of the leg, which are evertors of the foot and the skin on most of the dorsum of the foot. If the common fibular nerve were damaged all the structures that receive innervation via this nerve will be compromising dorsiflexion and eversion. Damage to the superficial fibular nerve affects the ability to evert the foot but does not result in foot drop, making the deep fibular nerve, which innervates the dorsiflexors of the foot, the best choice as the injury describes foot drop with the ability to evert the foot conserved. The saphenous nerve is a cutaneous nerve, while the tibial nerve innervates posterior compartment muscles.

A 45-year-old man is admitted to the emergency department after a fall and subsequent leg injury. On physical examination the patient has a foot drop but eversion is unaffected. Which nerve is most likely injured? A. Tibial B. Common fibular (peroneal) C. Superficial fibular (peroneal) D. Saphenous E. Deep fibular (peroneal)

C. The Achilles tendon reflex is a function of the triceps surae muscle, composed of the gastrocnemius and soleus muscles that insert on the calcaneus. The innervation is provided primarily by spinal nerve S1. The S1 root leaves the vertebral column at the S1 foramen of the sacrum, but a herniated disc at the L5-S1 intervertebral space puts the S1 root under tension, resulting in pain and possible weakness or paralysis of S1 supplied muscles, especially the plantar flexors. A disc lesion at L3-4 would affect the L4 spinal nerve (affecting foot inversion and extension); a lesion at L4-5 would cause problems with L5 (hip abduction and knee flexion). A disc lesion at S1-2 in the sacrum is improbable, unless there was lumbarization of the S1 vertebra. The gluteal crush syndrome usually occurs when a patient has been lying unconscious and unmoving on a hard surface for an extended period of time.

A 45-year-old man is admitted to the emergency department after experiencing a sharp pain while lifting a box of books. He told the physician that he "felt the pain in my backside, the back of my thigh, my leg, and the side of my foot." During physical examination it is observed that his Achilles tendon jerk is weakened on the affected side. Which is the most likely cause of injury? A. Disc lesion at L3-4 B. Disc lesion at L4-5 C. Disc lesion at L5-S1 D. Disc lesion at S1-2 E. Gluteal crush syndrome of the sciatic nerve or piriformis syndrome

E. The deep fibular (peroneal) nerve is responsible for innervating the muscles of the anterior compartment of the leg, which are responsible for toe extension, foot dorsiflexion, and inversion. Injury to this nerve will result in foot drop and also loss of sensation between the first and second toes. Injury to the tibial nerve affects the posterior compartment muscles of the leg, which are responsible for plantar flexion and toe flexion, as well as the intrinsic muscles of the sole of the foot. The common fibular (peroneal) nerve splits into the superficial and deep fibular (peroneal) nerves, and these supply both the lateral and anterior compartments. The superficial fibular (peroneal) nerve innervates the fibularis (peroneus) longus and brevis muscles, which provide eversion of the foot. If the common fibular (peroneal) nerve were injured, eversion of the foot and plantar flexion would be lost in addition to dorsiflexion and inversion. The saphenous nerve, a continuation of the femoral nerve, is a cutaneous nerve that supplies the medial side of the leg and foot and provides no motor innervation.

A 45-year-old man is treated at the hospital after he fell from his bicycle. Radiologic examination reveals fractures both of the tibia and the fibula. On physical examination the patient has a foot drop, but normal eversion (Fig. 5-1). Which of the following nerves is most likely injured? A. Tibial B. Common fibular (peroneal) C. Superficial fibular (peroneal) D. Saphenous E. Deep fibular (peroneal)

E. The tibial nerve is responsible for innervating the posterior compartment of the leg. These muscles are responsible for knee flexion, plantar flexion, and intrinsic muscle functions of the foot. Compression of this nerve can affect plantar flexion of the foot. Dorsiflexion of the foot would be compromised if the deep fibular (peroneal) nerve were compressed by this Baker's cyst. Flexion of the thigh is a function of muscles supplied by lumbar nerves and the femoral nerve. The deep fibular (peroneal) nerve is also responsible for extension of the digits, whereas the femoral nerve is responsible for extension of the leg.

A 45-year-old man presents at the local emergency clinic with the complaint of a painful knee and difficulty in walking. A computed tomography (CT) scan examination reveals a very large cyst in the popliteal fossa compressing the tibial nerve. Which movement will most likely be affected? A. Dorsiflexion of the foot B. Flexion of the thigh C. Extension of the digits D. Extension of the leg E. Plantar flexion of the foot

C. The lateral plantar nerve innervates the interossei and adductor hallucis. These losses would be obvious when the patient attempts to abduct and adduct the toes. Sensation would be absent over the lateral side of the sole, the fifth and fourth toes, and half of the third toe. The medial plantar nerve provides sensation over the plantar surface of the first and second toes and half of the third toe as well as function of the so-called LAFF muscles: first lumbrical, abductor hallucis, flexor hallucis brevis, and flexor digitorum brevis

A 46-year-old woman stepped on a broken wine bottle on the sidewalk and the sharp glass entered the posterior part of her foot. The patient was admitted to the hospital, and a physical examination concluded that her lateral plantar nerve had been transected (cut through). Which of the following conditions will most likely be confirmed by further physical examination? A. Loss of sensation over the plantar surface of the third toe B. Paralysis of the abductor hallucis C. Paralysis of the interossei and adductor hallucis D. Flexor hallucis brevis paralysis E. Flexor digitorum brevis paralysis

D. Spinal tuberculosis can spread within the sheath of the psoas major to its insertion with the iliacus upon the lesser trochanter, presenting there also with painful symptoms. The iliopsoas muscle is the principal flexor of the hip joint. Abduction of the hips is performed by the gluteus medius and minimus with assistance from short lateral rotator muscles. Extension of the hip is a function of the gluteus maximus, together with the hamstring muscles. Internal rotation is performed by the adductor muscle group.

A 61-year-old female immigrant had been diagnosed with spinal tuberculosis. The woman had developed a fluctuant, red, tender bulge on one flank, with a similar bulge in the groin on the same side. This presentation is likely due to spread of disease process within the fascia of a muscle with which of the following actions at the hip? A. Abduction B. Adduction C. Extension D. Flexion E. Internal rotation

D. The S1 nerve root provides cutaneous innervation to the lateral aspect of the ankle, the lateral sides of the dorsum and sole of foot, and motor innervation to the gastrocnemius muscle, which plantar flexes the foot and contracts during the ankle jerk reflex. It receives its innervation from the S1, S2 nerve roots via the tibial nerve, making D the correct choice. T12 roots do not reach the foot; L2 roots will reach the hip region and thigh; L4 innervates the invertors of the foot and skin over medial leg, ankle, and side of foot; and S3 innervates the sitting area of the buttocks, posterior scrotum or labia, and the small muscles of the foot.

A 48-year-old man comes to the physician because of severe back pain for 2 days. The pain radiates down to the buttock, posterior thigh, and posterolateral leg. He also has numbness on the lateral side of his left foot. On physical examination sensation to pain is decreased over the lateral side of the left foot. Deep tendon reflexes are absent at the left ankle, and there is a weakness of dorsiflexion of the left foot. Compression of which of the following nerve roots is the most likely cause of these findings? A. T12 B. L2 C. L4 D. S1 E. S3

D. Femoral artery puncture is one of the most common vascular procedures. The femoral artery can be localized often by simply feeling for the strongest point of the femoral pulse just inferior to the inguinal ligament. The femoral artery can be accessed with fluoroscopic assistance at the medial edge of the upper portion of the head of the femur. It is easily localized by Doppler ultrasound if the pulse is difficult to detect, such as in an obese patient. It is here that catheters are passed into the femoral artery for catheterization of abdominopelvic and thoracic structures and for antegrade angiography. It is also a site where arterial blood can be obtained for gas analysis. The mid inguinal point, halfway between the anterior superior iliac spine and the pubic symphysis, can be either medial or lateral to the femoral artery and is not a dependable landmark. A needle inserted at the level of the inguinal crease, or inferior to the femoral head, can enter the femoral artery distal to the origin of the deep femoral artery, presenting more risk for accidental vascular injury. Four centimeters lateral to the pubic tubercle overlies the deep inguinal ring, with potential entry to spermatic cord, femoral vein, or artery. The fossa ovalis is the opening in the deep fascia of the thigh for the termination of the great saphenous vein in the femoral vein.

A 48-year-old woman is admitted to the hospital with severe abdominal pain. Several imaging methods reveal that the patient suffers from intestinal ischemia. An abdominopelvic catheterization is ordered for antegrade angiography. A femoral puncture is performed. What is the landmark for femoral artery puncture? A. Halfway between anterior superior iliac spine and pubic symphysis B. 4.5 cm lateral to the pubic tubercle C. Midpoint of the inguinal skin crease D. Medial aspect of femoral head E. Lateral to the fossa ovalis

A. The femoral ring is the abdominal opening of the femoral canal. A femoral hernia passes through the femoral ring into the femoral canal deep and inferior to the inguinal ligament. It can appear as a bulging at the saphenous hiatus (fossa ovalis) of the deep fascia of the thigh, the hiatus through which the saphenous vein passes to the femoral vein. The superficial inguinal ring is the triangular opening in the aponeurosis of the external abdominal oblique and lies lateral to the pubic tubercle. The deep inguinal ring lies in the transversalis fascia lateral to the inferior epigastric vessels. Herniation into either of these two openings is associated with an inguinal hernia. The obturator canal, a bony opening between the superior and inferior ramus of the pubic bone, is the site of an obturator hernia.

A 49-year-old male construction worker is admitted to the emergency department with a painful lump on the proximal medial aspect of his thigh. Radiologic and physical examinations reveal that the patient has a herniation of abdominal viscera beneath the inguinal ligament into the thigh. Through which of the following openings will a hernia of this type initially pass to extend from the abdomen into the thigh? A. Femoral ring B. Superficial inguinal ring C. Deep inguinal ring D. Fossa ovalis E. Obturator canal

B. Contraction of the gastrocnemius on the fractured calcaneus would increase the pain because the gastrocnemius inserts with the soleus upon that bone, via the calcaneal tendon, or tendo Achilles. The flexor digitorum profundus passes the ankle medially to enter the sole of the foot, where it inserts upon the distal phalanges. The tibialis posterior, likewise, passes under the medial malleolus, with complex insertions upon the navicular bone, cuneiform bones, metatarsal bones, and the cuboid bone. The tibialis anterior, a muscle of the anterior leg compartment, inserts upon the navicular bone and, with the tibialis posterior, is a strong invertor of the foot. The fibularis (peroneus) longus is a muscle of the lateral compartment of the leg. It passes under the lateral malleolus, entering the sole of the foot by crossing the lateral surface of the calcaneus, and inserts primarily into the medial cuneiform and base of the first metatarsal bone.

A 49-year-old male worker fell from a ladder, with his weight impacting on the heels of his feet. Radiologic examination reveals comminuted calcaneal fractures. After the injury the contraction of which one of the following muscles would most likely increase the pain in the injured foot? A. Flexor digitorum profundus B. Gastrocnemius C. Tibialis posterior D. Tibialis anterior E. Fibularis (peroneus) longus

A. If the femoral artery is occluded, the descending branch of the lateral circumflex femoral will provide collateral circulation to the thigh. The descending genicular artery is a branch of the femoral and therefore would also be occluded. The medial circumflex femoral artery is a proximal branch of the deep femoral artery and supplies part of the head of the femur. The first perforating branch of the deep femoral artery supplies a small portion of the muscles of the posterior thigh. Finally, the obturator artery supplies a very small artery and vascularizes only the most proximal part of the head of the femur and usually only during the early years of life.

A 49-year-old man is admitted to the emergency department complaining that he has difficulties walking. Physical examination reveals that the patient suffers from peripheral vascular disease. An ultrasound examination reveals an occlusion of his femoral artery at the proximal portion of the adductor canal. Which of the following arteries will most likely provide collateral circulation to the thigh? A. Descending branch of the lateral circumflex femoral B. Descending genicular C. Medial circumflex femoral D. First perforating branch of deep femoral E. Obturator artery

D. The posterior tibial artery passes under the medial malleolus, about halfway between that bony landmark and the heel, or the calcaneus. The medial edge of the plantar aponeurosis can be palpated just medial to the muscular belly of the abductor hallucis. The sural nerve and the short (lesser) saphenous vein pass around the lateral side of the foot, about halfway between the lateral malleolus and the calcaneus. The sartorius passes behind the medial femoral condyle to insert on the proximal, medial aspect of the tibia via the pes anserinus; usually no pulse can be felt clearly there. The popliteal artery passes between the two heads of the gastrocnemius, where the arterial pulse may be felt very deeply, medial to the midline.

A 49-year-old man is admitted to the emergency department with a cold and pale foot. Physical examination reveals that the patient suffers from peripheral vascular disease; duplex ultrasound studies indicate possible occlusion of his popliteal artery, and the pulse of the posterior tibial artery is absent. What is the most common location for palpation of the pulse of the posterior tibial artery? A. Lateral to the muscular belly of the abductor hallucis B. Posteroinferior to the medial femoral condyle C. Groove midway between the lateral malleolus and the calcaneus D. Groove midway between the medial malleolus and the calcaneus E. Medially, between the two heads of the gastrocnemius

B. The femoral triangle is the best place to palpate the femoral pulse. It is bounded by the sartorius muscle laterally, adductor longus medially, and the inguinal ligament superiorly. It contains the femoral vein, artery, and nerve (from medial to lateral, respectively). The adductor canal lies deep between the anterior and medial compartments of the thigh and therefore cannot be palpated. The popliteal fossa is the fossa at the back of the knee and contains the popliteal artery and vein, tibial nerve, and common fibular (peroneal) nerve. The femoral pulse cannot be palpated here. The inguinal canal is in the pelvis and is in communication with the anterior abdominal wall. It contains the spermatic cord in males and round ligament of the uterus in females.

A 49-year-old man is admitted to the emergency department with a cold and pale foot. Physical examination reveals that the patient suffers from peripheral vascular disease; his popliteal artery is occluded and no pulse is felt upon palpation. What is the landmark to feel the pulse of the femoral artery? A. Adductor canal B. Femoral triangle C. Popliteal fossa D. Inguinal canal E. Pubic symphysis

D. The great saphenous vein is commonly used in coronary artery bypass grafts. Because branches of the saphenous nerve cross the vein in the distal part of the leg, the nerve can be damaged if the vein is stripped from the ankle to the knee. Stripping the vein in the opposite direction can protect the nerve and lessen the postoperative discomfort of patients. The saphenous nerve is responsible for cutaneous innervations on the medial surface of the leg and the medial side of the foot. Injury to this nerve will result in a loss of sensation and also can create chronic dysesthesias in the area. The common fibular (peroneal) nerve bifurcates at the neck of the fibula into the superficial and deep fibular (peroneal) nerves, which continue on to innervate the lateral and anterior compartments of the leg, respectively. These nerves are lateral and therefore not associated with the great saphenous vein. The lateral sural nerve is a cutaneous nerve that arises from the junction of branches from the common fibular (peroneal) nerve and tibial nerve and innervates the skin on the posterior aspect of the leg and lateral side of the foot. This nerve is often harvested for nerve grafts elsewhere in the body. The tibial nerve is a terminal branch of the sciatic nerve that continues deep in the posterior compartment of the leg.

A 49-year-old man underwent a coronary bypass graft procedure using the great saphenous vein. Postoperatively, the patient complains of pain and general lack of normal sensation on the medial surface of the leg and foot on the limb from which the graft was harvested. Which nerve was most likely injured during surgery? A. Common fibular (peroneal) B. Superficial fibular (peroneal) C. Lateral sural D. Saphenous E. Tibial

B. The child has the problem of talipes equinovarus, or clubfoot. Clubfoot is a congenital malformation observed in about 1 in 1000 pediatric patients and first appears in the first trimester of pregnancy. This syndrome combines plantar flexion, inversion, and adduction of the foot. The heel is drawn upward by the tendo calcaneus and turned inward; the forefoot is also adducted, or turned inward. The foot usually is smaller than normal. In coxa vara, the angle between the femoral shaft and neck is reduced to less than 120 degrees, often due to excessive activity of the adductor musculature. Hallux valgus is also known as bunion, in which the big toe points laterally. Hallux varus involves a medial deviation of the first metatarsal or big toe, sometimes the result of attempted correction of bunions. It can also result from arthritis or muscular problems.

A 5-month-old baby boy is admitted to the pediatric orthopedic clinic. During physical examination it is noted that the baby has inversion and adduction of the forefoot relative to the hindfoot and plantar flexion. Which of the following terms is diagnostic for the signs observed on physical examination? A. Coxa vara B. Talipes equinovarus C. Hallux valgus D. Hallux varus E. Plantar fasciitis

B. In infants and children up to about 8 years of age, the head of the femur gets its arterial supply by a direct branch of the obturator artery (variably, the medial circumflex femoral). The arterial supply reaches the head of the femur at the fovea capitis by traveling along the ligament of the head of the femur. Probably due to repeated torsion on the ligament, and therefore on the artery, this artery occludes early in life. In turn, this source of supply is replaced by branches of the gluteal and femoral circumflex vessels.

A 5-year-old boy is admitted to the emergency department after a car collision. Radiologic examination reveals a fracture of the head of the femur. An MRI examination reveals a large hematoma. Which of the following arteries is most likely injured? A. Deep circumflex iliac B. Acetabular branch of obturator C. Descending branch of lateral circumflex femoral D. Medial circumflex femoral E. Radicular branches of circumflex artery

A. Dorsalis pedis pulse is palpated at the prominent arch of the top of the foot between the first and second metatarsal bones between the tendon of the extensor hallucis longus and extensor digitorum longus for the second toe.

A 50-year-old diabetic man presents for a routine wellness checkup. During physical examination it is noted that he has paraesthesia in a classic glove and stocking distribution. The physician decides on a complete peripheral vascular system examination, which includes palpating the pulse of the dorsalis pedis. Where can the dorsalis pedis pulse be palpated? A. Between the tendons of extensor hallucis and extensor digitorum longus on the dorsum of the foot B. Superior to flexor hallucis longus just distal to the tarsal tunnel C. Inferolateral to the pubic symphysis and medial to the deep dorsal vein of the penis D. 2 cm anterior to the medial malleolus E. 2 cm posterior to the medial malleolus

C. The lymphatic drainage of the leg is such that superficial lymphatics on the anterolateral side of the foot and leg and all the deep lymphatics in the foot and leg first drain into the popliteal nodes and then to the deep inguinal nodes. This patient has an infected anterolateral mid leg injury, which will first drain into the popliteal nodes. The vertical group of superficial inguinal nodes receives superficial lymphatics from the medial side of the foot, leg, and all the superficial lymph from the thigh. The horizontal group of superficial lymphatics receives lymph from the anterior abdominal wall below the umbilicus, perineum (except the glans penis in men and clitoris in women), and lower third of the anal canal. The popliteal nodes eventually drain to the deep inguinal nodes, but are usually not palpable. The iliac nodes are deep structures and are not be palpable during physical examination.

A 50-year-old woman is admitted to the emergency department complaining of painful swelling to the left leg, fever, and malaise for 2 days. The patient has a history of type 2 diabetes mellitus, and she was bitten on the left leg by an insect a week before presentation. She scratched the pruritic area and applied alcohol to the site when the swelling increased; a purulent fluid began to drain from it 2 days later. During physical examination, the patient was febrile (40° C) with 5 × 5 cm tender, fluctuant swelling over the anterolateral aspect of the middle third of the left leg, which drained copious amounts of purulent fluid. Which of the following findings is most likely to be also present during physical examination of this patient? A. Tender vertical group of superficial inguinal lymph nodes B. Enlarged horizontal group of superficial inguinal lymph nodes C. Enlarged group of deep inguinal lymph nodes D. Enlarged popliteal lymph nodes E. Enlarged iliac nodes

B. The psoas muscle arises from the base of the transverse processes, the sides of the vertebral bodies, and the intervertebral discs, from the twelfth thoracic to the fifth lumbar vertebrae and inserted into the lesser trochanter of the femur. The sheath of the psoas retains the pus of a psoas abscess, and spinal tuberculosis may present as a cold abscess in the groin (in the vicinity of the lesser trochanter). The psoas is enclosed in a fibrous sheath that is derived from the lumbar fascia. The sheath is not part of the lumbar fascia, but the lateral edge blends with the anterior layer of that fascia

A 51-year-old immigrant with tuberculosis is found to have large flocculent masses over the lateral lumbar spine. There is a similar mass located in the ipsilateral groin. Physical examination reveals increased tenderness just medial to the ipsilateral anterior superior iliac spine on palpation. This pattern of involvement most likely suggests an abscess tracking along which of the following muscles? A. Piriformis B. Psoas major C. Adductor longus D. Gluteus maximus E. Obturator internus

E. The talus can be rotated externally when the ankle sustains a trimalleolar fracture, also called a Henderson fracture. The fracture may be caused by eversion and posterior displacement of the talus. This injury involves the fracture of the distal fibula (lateral malleolus); the medial malleolus of the tibia; and the posterior portion, or lip, of the tibial plafond (the distal articular portion of the tibia, sometimes referred to as the posterior malleolus). The posterior part of the plafond is not truly a malleolus but acts this way in this type of twisting fracture of the ankle. The talus can be forced from its normal position in this fracture, adding to the instability of the ankle. The other bones listed are relatively far from the site of the fractures

A 52-year-old woman is admitted to the emergency department after severely injuring her right lower limb when she fell from a trampoline. Radiologic examination reveals a trimalleolar fracture of the ankle involving the lateral malleolus, medial malleolus, and the posterior process of the tibia. Which of the following bones will also most likely be affected? A. Navicular B. Calcaneus C. Cuneiform D. Cuboid E. Talus

B. The right superior gluteal nerve is the correct choice. When a person stands on one leg or walks, the gluteus medius, gluteus minimus, and tensor muscles of the fascia latae act in synergy to stabilize the hip joint by abducting the hip (pelvic tilt). These muscles receive their innervation from the superior gluteal nerve. The abductors of the hip, as they contract to maintain the stability of the hip joint, draw the pelvis forcefully toward the weigh-bearing leg, causing the opposite side of the pelvis to tilt in that same direction. The right superior gluteal nerve innervates its ipsilateral medius, minimus, and tensor muscles of the fascia latae. Loss of these muscles results in a positive Trendelenburg sign with the pelvis dropping on the left side.

A 53-year-old woman reports difficulty in walking. Physical examination showed a positive Trendelenburg sign when she is asked to stand on her right leg. Which nerve has been compromised to produce the positive sign? A. Sciatic B. Right superior gluteal C. Left inferior gluteal D. Left superior gluteal E. Right inferior gluteal

A. The "unhappy triad" (of O'Donoghue) is composed of the medial collateral ligament, medial meniscus, and anterior cruciate ligament. Sudden, forceful thrusts against the lateral side of the knee put tension on the medial collateral ligament, which can then rupture. The medial meniscus is attached to the medial collateral ligament so that it then tears. The anterior cruciate ligament resists hyperextension of the knee; thus, it is the third structure that ruptures in the "unhappy triad" of the knee.

A 55-year-old cowboy is admitted to the emergency department after he was knocked from his feet by a young longhorn steer. MRI examination reveals a large hematoma in the knee joint. Physical examination reveals that the patient suffers from the "unhappy triad" (of O'Donoghue). Which of the following structures are involved in such an injury? A. Medial collateral ligament, medial meniscus, and anterior cruciate ligament B. Lateral collateral ligament, lateral meniscus, and posterior cruciate ligament C. Medial collateral ligament, lateral meniscus, and anterior cruciate ligament D. Lateral collateral ligament, medial meniscus, and anterior cruciate ligament E. Medial collateral ligament, medial meniscus, and posterior cruciate ligament

D. The dorsalis pedis is the continuation of the anterior tibial artery into the foot, as it passes the distal end of the tibia and the ankle joint. The pulse of the dorsalis pedis can be felt between the tendon of the extensor hallucis longus and the tendon of the extensor digitorum longus to the second toe. A strong pulse is a positive indicator of circulation through the limb. The fibular (peroneal) artery is a branch of the posterior tibial artery and passes in the calf between the flexor hallucis longus and tibialis posterior, making it difficult to palpate. The deep plantar artery, the extension of the first dorsal interosseous or lateral plantar arteries, passes deep to the aponeurotic tissues and central muscles of the foot, making palpation unlikely. The dorsal metatarsal branches of the dorsalis pedis pass under cover of the extensor digitorum longus and brevis tendons. Palpable pulses of the first or other dorsal metatarsal arteries can therefore be difficult to detect

A 55-year-old man is admitted to the hospital for an iliofemoral bypass. The operation is performed successfully and the blood flow between the iliac and femoral arteries is restored. During rehabilitation which of the following arteries should be palpated to monitor good circulation of the lower limb? A. Anterior tibial B. Deep fibular (peroneal) C. Deep plantar D. Dorsalis pedis E. Dorsal metatarsal

A. Flat foot (pes planus) is due to flattening of the medial longitudinal arch. Often congenital, it may be associated with minor structural anomalies of the tarsal bones. This condition can be seen in wet footprints in which the medial surface of the sole (normally raised in an arch) is visible. Treatment may include intensive foot exercises or arch supports worn in the shoes. Occasionally, surgery is needed in the form of arthrodesis (fusion of the tarsal bones). Pes cavus is a deformity of the foot characterized by a very high medial arch and hyperextension of the toes. The long plantar ligament is a passive ligament of the longitudinal arch. The long plantar ligament connects the calcaneus and cuboid bones. It can be involved with the plantar aponeurosis in plantar fasciitis. The long plantar ligament converts the cuboid groove into a canal for the tendon of the fibularis (peroneus) longus. The deltoid ligament is a very strong ligament that interconnects the tibia with the navicular, calcaneus, and talus bones. The medial malleolus will usually fracture before this ligament will tear. The plantar calcaneonavicular, or spring, ligament is a key element in the medial longitudinal arch; it supports the head of the talus bone and thereby is subject to vertical forces exerted through the lower limbs. In the present case, the bilateral pes planus appears to be the res

A 55-year-old man visits the outpatient clinic complaining that he cannot walk more than 5 minutes without feeling severe pain in his feet. An image of the feet of this patient is shown in Fig. 5-10. What is the most common cause of this condition? A. Collapse of medial longitudinal arch, with eversion and abduction of the forefoot B. Exaggerated height of the medial longitudinal arch of the foot C. Collapse of long plantar ligament D. Collapse of deltoid ligament E. Collapse of plantar calcaneonavicular ligament

B. An injury to L4 would cause weakness in the patellar reflex and loss of cutaneous innervation to the medial side of the leg. The patellar reflex is used to test L2 to L4 nerve integrity. The motor side of the reflex is primarily derived from spinal nerves L2 and L3, whereas the sensory side of the arc is said to be principally from L4. The L4 spinal nerve supplies the L4 dermatome on the medial side of the leg and foot, by way of the saphenous nerve. It also supplies foot inversion, a function of the tibialis anterior and tibialis posterior muscles; the first is supplied by the deep fibular (peroneal) nerve, and the second supplied by the tibial nerve. Foot dorsiflexion is weakened because of partial denervation of the extensor digitorum longus, but L5 is still contributing to that function. The foot is everted because the S1-supplied (by the superficial fibular nerve) fibularis (peroneus) longus and brevis are unopposed. The Achilles reflex is also primarily supplied by S1. Hip movements are produced primarily by L5- and S1-supplied muscles, as is knee flexion.

A 55-year-old woman is admitted to the emergency department after an automobile crash. Physical examination reveals that the patient's foot is everted and she cannot invert it. A weakness in dorsiflexion and inversion of the foot is noted. Her ipsilateral patellar reflex is reduced in quality, although the Achilles tendon reflex is brisk. Knee extension is almost normal, as are all hip movements and knee flexion. Sensation is greatly reduced on the medial side of the leg. Which of the following nerves is most likely injured? A. Femoral nerve B. L4 spinal nerve C. L4 and L5 spinal nerves D. Common fibular (peroneal) nerve E. Tibial nerve

D. Injury to the dorsalis pedis artery on the dorsum of the foot can also cause trauma to the terminal portion of the deep fibular (peroneal) nerve. In the proximal part of the foot, this could result in loss of sensation between the first and second toes and paralysis of the extensor digitorum brevis and the extensor hallucis brevis muscles. In the distal part of the foot, only the sensory loss might be apparent. Clubfoot is a congenital malformation observed in pediatric patients. This syndrome combines plantar flexion, inversion, and adduction of the foot. Neither extension of the big toe by the extensor hallucis longus nor paralysis of the tibialis anterior (weakness of foot inversion) would occur by this injury because both of these muscles are innervated by the deep fibular (peroneal) nerve much more proximally in the leg

A 55-year-old woman is bitten by a dog in the dorsum of the foot and is admitted to the emergency department. The wound is cleaned thoroughly, during which it is seen that no tendons have been cut, but the dorsalis pedis artery and the accompanying nerve have been injured. Which of the following conditions would be expected during physical examination? A. Clubfoot B. Foot drop C. Inability to extend the big toe D. Numbness between the first and second toes E. Weakness in inversion of the foot

C. The deep fibular (peroneal) nerve passes deep to the extensor retinaculum and supplies the intrinsic muscles on the dorsum of the foot (extensors digitorum and hallucis longus) and the tarsal and tarsometatarsal joints. When it finally emerges as a cutaneous nerve, it is so far distal in the foot that only a small area of skin remains available for innervation: the web of skin between and contiguous sides of the first and second toes. The superficial fibular (peroneal) nerve supplies the skin on the anterolateral aspect of the leg and divides into the medial and intermediate dorsal cutaneous nerves, which continue across the ankle to supply most of the skin on the dorsum of the foot.

A 56-year-old diabetic man complains of repeated injury and ulcers to his right big toe. He also complains that he finds it difficult maintaining his shoes because the tips of the shoes around the toe area easily wear down. He also complains that for a while now, his first two toes "feel funny." He used to enjoy playing soccer on weekends but has found it difficult to be involved. Which of the following nerves is most likely affected? A. Superior gluteal nerve injury B. Inferior gluteal nerve injury C. Deep fibular (peroneal) nerve injury D. Superficial fibular (peroneal) nerve injury E. Common fibular (peroneal) nerve injury

B. The obturator nerve arises from the lumbar plexus and enters the thigh through the obturator canal. This nerve is responsible for innervation of the medial compartment of the thigh (adductor compartment). Injury to this nerve can result in weakened adduction and difficulty walking. The femoral nerve innervates muscles of the anterior compartment of the thigh that are responsible for hip flexion and leg extension. The sciatic nerve branches into the common fibular (peroneal) and tibial nerves. The common fibular (peroneal) nerve branches into the deep and superficial branches of the fibular (peroneal) nerve responsible for innervation of the anterior and lateral compartments of the leg, respectively. The tibial nerve innervates the muscles of the posterior compartment of the thigh and leg, which are responsible for extension of the hip, flexion of the leg, and plantar flexion of the foot.

A 56-year-old man with advanced bladder carcinoma suffers from difficulty while walking. Muscle testing reveals weakened adductors of the right thigh. Which nerve is most likely being compressed by the tumor to result in walking difficulty? A. Femoral B. Obturator C. Common fibular (peroneal) D. Tibial E. Sciatic

E. The lateral plantar artery provides origin to the deep plantar arterial arch. Medially, the vascular arch anastomoses with the distal portion of the dorsalis pedis by way of the deep plantar artery. The anterior tibial artery continues as the dorsalis pedis at the ankle joint. The fibular (peroneal) artery, by way of a perforating branch in some individuals, replaces the dorsal pedis. The arcuate artery, a branch of the dorsalis pedis, provides origin for the dorsal metatarsal arteries to the lateral toes.

A 58-year-old diabetic patient is admitted to the hospital with a painful foot. Physical examination reveals that the patient suffers from peripheral vascular disease. There is no detectable dorsalis pedis arterial pulse, but the posterior tibial pulse is strong. Which of the following arteries will most likely provide adequate collateral supply from the plantar surface to the toes and dorsum of the foot? A. Anterior tibial B. Fibular (peroneal) C. Arcuate D. Medial plantar E. Lateral plantar

A. The adductor hallucis muscle inserts upon the lateral side of the proximal phalanx of the great toe, and also the lateral sesamoid bone, by way of its oblique and transverse heads. It is supplied by the lateral plantar nerve. The abductor hallucis inserts upon the medial side of the proximal phalanx and the medial sesamoid bone of the great toe. The sesamoid bones are within the tendon of the flexor hallucis brevis and assist it in its function at the first metatarsophalangeal joint. The abductor and flexor hallucis brevis are innervated by the medial plantar nerve. The first dorsal interosseous muscle and the first lumbrical both insert on the medial side of the extensor mechanism of the second toe. The quadratus plantae arises from the calcaneus and inserts on the tendon of the flexor digitorum longus muscle. The first lumbrical is supplied by the medial plantar nerve. The quadratus plantae, the lumbricals 2 to 4, and all interossei are innervated by the lateral plantar nerve

A 58-year-old female dancer presented to the orthopedic clinic with a complaint of pain during her work because of bilateral bunions. She was referred to a podiatric surgeon who scheduled her for surgery. The protruding bony and soft tissues of the toe were excised, and a muscle was reflected from the lateral side of the proximal phalanx, together with a sesamoid bone, upon which the muscle also inserted. What muscle was this? A. Adductor hallucis B. Abductor hallucis C. First dorsal interosseous D. First lumbrical E. Quadratus plantae

A. Excessive compression of the prepatellar bursa, as in working on bended knees, can result in pain and swelling of the prepatellar bursa, the so called housemaid's knee. Prepatellar bursitis affects plumbers, carpet layers, and other people who spend a lot of time on their knees. The bursa normally enables the patella to move smoothly under the skin. The constant friction of these occupations irritates this small lubricating sac (bursa) located just in front of the patella, resulting in a deformable tense cushion of fluid. Treatment usually requires simple drainage, but this may need to be repeated and occasionally steroids introduced. Excessive irritation of the infrapatellar bursa in kneeling for frequent and long periods of time (as in prayer) can result in "parson's knee." The posterior cruciate ligament of the knee can be injured in sudden, strong flexion of the knee, with posterior displacement of the tibia upon the femur. The patellar retinacula are strong, tendinous bands of tissue that join the quadriceps tendon to the vastus lateralis and medialis muscles. The lateral meniscus is a cartilaginous structure between the lateral condyles of the femur and tibia.

A 58-year-old female employee of a housecleaning business visits the outpatient clinic with a complaint of a constant burning pain in her knees. Clinical examinations reveal a "housemaid's knee" condition (Fig. 5-9). Which of the following structures is most likely affected? A. Prepatellar bursa B. Infrapatellar bursa C. Posterior cruciate ligament D. Patellar retinacula E. Lateral meniscus

D. The farm instrument has injured the deep fibular (peroneal) branch of the common fibular (peroneal) nerve. It is vulnerable to injury as it arises from the common fibular (peroneal) at the neck of the fibula. The muscles denervated are largely dorsiflexors of the foot; hence, foot drop and a high stepping gait can occur. Sensation on the dorsum of the foot is still present; therefore, the superficial branch is mostly or entirely intact, although sensation between the first and second toes would be absent. Femoral nerve injury would result in loss of knee extension. Loss of the sciatic nerve would result in loss of both the tibial and common fibular (peroneal) nerves. Because plantar flexion is still functional, the tibial nerve has not been cut.

A 58-year-old male farmer was accidentally struck with a scythe (a long, curved cutting blade) by another worker while they were cutting wheat. He was admitted to the county hospital with severe bleeding. During physical examination the doctor noted that the patient had a foot drop; sensation was present over the dorsum of the foot and the skin of the posterior calf. Which of the following nerves was injured? A. Femoral nerve B. Sciatic nerve C. Superficial fibular (peroneal) nerve D. Deep fibular (peroneal) nerve E. Common fibular (peroneal) nerve

C. In piriformis entrapment, the sciatic nerve can be compressed when the piriformis is contracted, leading to painful sensations in the lower limb. These usually involve pain in the gluteal area, posterior thigh, and leg, most frequently resembling a disc lesion at L5-S1, with compression of the S1 spinal nerve. L4 compression would be rather unusual but would involve the quadriceps femoris knee extension, foot inversion, and sensory loss on the medial side of the leg. L5 compression would be indicated by weakness in hip abduction, knee flexion, and sensory loss on dorsal and plantar surfaces of the foot. S1 compression would weaken plantar flexion and foot eversion. Pudendal nerve entrapment would affect the perineal region. The fibular (peroneal) division of the sciatic nerve passes through the piriformis in some individuals, leading to L5, S1-S3 nerve compression.

A 58-year-old man is admitted to the hospital with pain in his lower limb for the past 2 months. Physical examination reveals point tenderness in the region of his greater sciatic foramen, with pain radiating down the posterior aspect of his thigh. An MRI examination reveals that the patient suffers from piriformis entrapment syndrome. He is directed to treatment by a physical therapist for stretching and relaxation of the muscle. Entrapment of which of the following nerves can mimic piriformis entrapment syndrome? A. L4 B. L5 C. S1 D. S2 E. S3

A. The patellar reflex causes extension of the knee and is produced by the quadriceps muscle group which consist of; biceps femoris and vastus lateralis, medialis and intermedius. Quadratus femoris is a lateral rotator of the thigh. The sartorius is a flexor of the hip and knee, and the pectineus is an adductor and flexor of the hip.

A 58-year-old man visited his physician for his annual check-up. Physical examination reveals a hyperreflexia patellar reflex. Which muscle(s) contribute(s) to the tendon that is struck when testing this reflex? A. Quadriceps femoris B. Quadratus femoris C. Sartorius D. Pectineus E. Biceps femoris

A. The saphenous nerve runs with the great saphenous nerve which was being removed from patient. Sensory innervation to the areas of loss described is by the L4 root, which is carried by the saphenous nerve. The obturator nerve innervates the skin on the superior medial thigh. Lateral femoral cutaneous innervates the lateral aspect of the thigh. The tibial nerve supplies cutaneous innervation to the lateral aspect of the leg and if damaged would also produce muscular dysfunction. The femoral nerve is a motor and sensory nerve and is the origin of the saphenous nerve.

A 58-year-old woman presents to the outpatient surgery clinic for removal of varicose veins on the medial aspect of her foot. The operation was successful however, one month later she reports loss of sensation over the medial aspect of her leg and foot. Which of the following nerves was most likely injured during the procedure? A. Saphenous B. Obturator C. Lateral femoral cutaneous D. Tibial E. Femoral

D. Duchenne muscular dystrophy is a condition that causes muscle weakness. It starts in childhood and may be noticed when a child has difficulty standing up, climbing, or running, which requires extension of the hip. This patient has the classic Gower's sign. The gluteus maximus functions primarily between the flexed and standing (straight) positions of the thigh, as when rising from the sitting position, straightening from the bending position, walking uphill and up stairs, and running.

A 6-year-old boy with a family history of muscular disease leading to wheelchair dependency in his maternal uncles presents with difficulty in standing from the seated position. He bends forward, uses his hands to help him push up from the floor, and then straightens his knees to stand. Which of the following muscles is most likely involved by this disease process? A. Tibialis posterior and gastrocnemius B. Quadratus femoris C. Gluteus medius and gluteus minimus D. Gluteus maximus E. Hamstrings F. Iliopsoas

B. The saphenous nerve is the longest and most widely distributed cutaneous branch of the femoral nerve; it is the only branch not from the sciatic nerve to extend beyond the knee. It gives sensory innervations to the medial aspect of the thigh, leg, and the medial planter arch. It accompanies the great saphenous vein over the medial side of the leg. The great saphenous vein is formed by the union of the dorsal vein of the great toe and the dorsal venous arch of the foot. It ascends anterior to the medial malleolus and passes posterior to the medial condyle of the femur and ends when it joins the femoral vein.

A 60-year-old man presents with pain on the medial aspect of his thigh. During physical examination he describes the pain to be constant, nonradiating and he also complains of numbness on the medial aspect of his leg and medial plantar arch. The nerve involved in this patient's numbness is closely associated with a structure with which of the following characteristics? A. Empties into the popliteal vein B. In its ascent in the medial aspect of the leg, it travels posterior to the medial condyle of the femur C. In its ascent in the medial aspect of the leg, it travels anterior to the medial condyle of the femur D. Arches posterior to the medial malleolus E. Is associated with nodes that drain to the horizontal group of inguinal nodes

E. Tarsal tunnel syndrome is a compression neuropathy resulting from the compression of the tibial nerve in the tarsal tunnel. The tarsal tunnel is located between the medial malleolus, the inferomedial surface of the calcaneus, and the flexor retinaculum. The contents are the tibial nerve and its plantar branches, the tendons of the tibialis posterior, flexor digitorum longus, and the flexor hallucis longus muscles together with the posterior tibial vessels. Any inflammation or swelling in the area will compress on these structures, most significantly the tibial nerve. The posterior tibial vein will be most easily compressed but the nerve is most clinically significant. Clinically, this syndrome is diagnosed with the patient's history and physical examination findings including a positive Tinel's sign (lightly tapping over the flexor retinaculum elicits numbness and tingling in the skin over the calcaneus and the sole of the foot).

A 60-year-old retired male marathon runner complains to his primary care physician that during his daily morning jog he experiences bouts of numbness and tingling on the medial aspect of his heel. Upon further examination the doctor discovers the patient has trouble tiptoeing and shows a positive Tinel's sign. Which of the following conditions is most characteristic of these symptoms? A. Plantar fasciitis B. Ankle inversion sprain C. Morton's neuroma D. Lateral ligament E. Tarsal tunnel syndrome

D. The ankle jerk reflex, elicited by tapping the tendo Achilles with the reflex hammer, is mediated by the tibial nerve. The superficial fibular (peroneal) nerve supplies the foot evertor muscles of the lateral compartment of the leg and provides sensory supply for the dorsum of the foot. The deep fibular (peroneal) nerve innervates the foot extensor and invertor muscles in the anterior compartment of the leg and supplies skin between the first and second toes. The common fibular (peroneal) nerve combines the functions of the superficial and deep branches. The medial plantar nerve innervates the abductor and flexor muscles of the big toe, the first lumbrical muscle, and flexor digitorum brevis muscle and provides sensation for the medial plantar surface and three and a half toes. GAS

A 62-year-old man is admitted to the emergency department. Radiologic examination and the available data indicate the likelihood of a transient ischemic attack. During physical examination the ankle jerk reflex is absent. Which of the following nerves is most likely responsible for the reflex arc? A. Common fibular (peroneal) B. Superficial fibular (peroneal) C. Deep fibular (peroneal) D. Tibial E. Superficial and deep fibular (peroneal)

C. The patient's complaint is due to her case of hammertoes. Hammer toe can affect any toe but most commonly the second toe, then the third or fourth toes. It results most commonly from wearing shoes that are too short or shoes with heels that are too high. In hammertoe, the metatarsophalangeal joint is extended, the proximal interphalangeal joint is flexed, and the distal phalanx points downward, looking like a hammer. Hammertoe can occur as a result of a bunion. Calluses, or painful corns, can form on the dorsal surface of the joints. In claw toe, both the proximal and distal interphalangeal joints are strongly flexed, the result of muscle imbalance in the foot. Either hammertoe or claw toe can occur from arthritic changes. Pes cavus is the opposite of flat foot. In this case the patient has a high, flexed plantar arch; it occurs as a result of hereditary motor and sensory neural problems. It is painful because of metatarsal compression.

A 63-year-old woman visits the outpatient orthopedic clinic with the complaint of pain in her foot for more than a year. Radiologic and physical examinations give evidence of constant extension at the metatarsophalangeal joints, hyperflexion at the proximal interphalangeal joints, and extension of distal interphalangeal joints (Fig. 5-6). Which of the following terms is most accurate to describe the signs of physical examination? A. Pes planus B. Pes cavus C. Hammertoes D. Claw toes E. Hallux valgus

E. The posterior tibial artery provides most of the arterial supply for the neck and body of the talus bone. The fibular (peroneal) artery provides a small amount of vascular supply. The medial plantar and lateral plantar branches of the posterior tibial artery are distributed to tissues in the plantar surface of the foot. The dorsalis pedis is the continuation of the anterior tibial artery on the dorsum of the foot.

A 65-year-old man is admitted to the hospital after falling from his roof while cleaning leaves and pine needles from the gutters. Among other injuries suffered in his fall, radiologic examination reveals a fracture of the talus bone in one foot. Much of the blood supply of this bone can be lost in such an injury and can result in osteonecrosis. From what artery does this bone receive its primary vascular supply? A. Medial plantar B. Lateral plantar C. Dorsalis pedis D. Anterior tibial E. Posterior tibial

B. Generally, the angle of inclination between the neck and shaft of the femur in older age decreases to around 120 degrees. However, in pathologic conditions it can either increase or decrease from the predicted value. When the angle of inclination increases, it is referred to as coxa valga. Coxa vara on the other hand is a condition characterized by a decreased angle of inclination. Genu varum and genu valgum are deformities characterized by a decreased Q-angle and increased Q-angle, respectively. The Q-angle refers to the angle between the femur and tibia. Hallux valgus is a condition that presents with a lateral deviation of the large toe.

A 67-year-old woman has been suffering from osteoporosis for the past year. During her annual checkup, radiologic examination reveals an angle of 160 degrees made by the axis of the femoral neck to the axis of the femoral shaft. Which of the following conditions is associated with these examination findings? A. Coxa vara B. Coxa valga C. Genu valgum D. Genu varum E. Hallux valgus

A. The talocrural joint is a synovial hinge joint that connects the distal end of the tibia and fibula with the proximal end of the talus. The articulation between the tibia and the talus bears more weight than other joints. Dorsiflexion (toes pointing upward) and plantar flexion (toes pointing downward) are possible. Dorsiflexion is performed by the tibialis anterior, extensor hallucis longus, extensor digitorum longus, and peroneus tertius. Plantar flexion is performed by the gastrocnemius, soleus, plantaris, peroneus longus, peroneus brevis, tibialis posterior, flexor digitorum longus, and flexor hallucis longus. The movements of inversion and eversion take place at the talocalcaneal joint.

A 69-year-old woman, who fell down the stairs, presents to the emergency department. Radiologic imaging reveals a fracture of the talocrural (tibiotalar) joint. Which movements take place at this joint? A. Plantar flexion and dorsiflexion B. Inversion and eversion C. Plantar flexion, dorsiflexion, inversion, and eversion D. Plantar flexion and inversion E. Dorsiflexion and eversion

B. The medial plantar nerve innervates the abductor hallucis and both flexor hallucis longus and brevis. This nerve also provides motor supply for the flexor digitorum brevis and the first lumbrical. The lateral plantar nerve innervates all other intrinsic muscles in the plantar region of the foot. The sural nerve is sensory to the lateral posterior leg and lateral side of the foot; it arises from a combination of branches of the tibial nerve and common fibular (peroneal) nerve. The deep fibular (peroneal) nerve supplies dorsiflexors, toe extensors, and invertors of the foot.

A 7-year-old girl accidentally stepped on a sharp snail shell while walking to the beach. She was admitted to the hospital, where she received a tetanus shot, and the wound was cleaned thoroughly and sutured. One week later, during a return visit to her physician, it is seen that she has great difficulty in flexing her big toe, even though there is no inflammation present in the sole of the foot. Which nerve was most likely damaged by the piercing of the shell? A. Lateral plantar nerve B. Medial plantar nerve C. Sural nerve D. Superficial fibular (peroneal) nerve E. Deep fibular (peroneal) nerve

D. In claw toe, both the proximal and distal interphalangeal joints are strongly flexed, the result of muscle imbalance in the foot. With muscular imbalance, the extensors of the interphalangeal joints are overpowered by the long flexors. The metatarsophalangeal joint is extended, whereas in hammertoe it can be in a neutral position. Either hammer toe or claw toe can occur from arthritic changes. Hammertoe can affect any toe, but it most commonly affects the second toe, then the third or fourth toes. It results most commonly from wearing shoes that are too short or shoes with heels that are too high. In hammertoe, the metatarsophalangeal joint is extended, the proximal interphalangeal joint is flexed, and the distal phalanx may be dorsiflexed, or it may point downward, looking like a hammer. Hammertoe can occur as a result of a bunion. Calluses, or painful corns, can form on the dorsal surface of the joints. Hallux valgus is more commonly referred to as a bunion. The big toe is angulated toward the little toe and may override the second toe. The base of the first metatarsal bone is directed medially and is subject, painfully, to compression. Pes cavus is the opposite of flat foot; the patient has a high, flexed plantar arch. Pes cavus occurs as a result of hereditary motor and sensory neural problems. It is painful because of metatarsal compression.

A 71-year-old man is admitted to the orthopedic clinic with difficulties walking. The patient has a past history of polio. Physical and radiologic examinations reveal extension at the metatarsophalangeal joints with flexion of both the proximal and distal interphalangeal joints. Which of the following descriptions is most appropriate for this patient's condition? A. Hallux valgus B. Pes planus C. Hammertoes D. Claw toes E. Pes cavus

E. Hallux valgus, or lateral displacement of the great toe, usually presents as pain over the prominent metatarsal head, due to rubbing from shoes, and it can be associated with deformity of the second toe, which then tends to override the great toe. Hallux valgus is commonly known as bunion. In this deformity the big toe points toward the little toe; the base of the first metatarsal points medially, with a swollen bursal sac at the metatarsophalangeal joint. Excess bony growth of the distal protruding part of the metatarsal bone can also occur. Bunions occur only rarely in people who do not routinely wear shoes. Genu varus is also referred to as bowlegs, or bandy legs, in which the knees are bowed outward. The opposite of this is genu valgus, or knock knee. The normal angle between the femoral shaft and femoral neck is between 120 and 135 degrees. In coxa vara the angle between the shaft and neck of the femur is less than 120 degrees. This can result from fractures, other injuries, or congenital softness of the bone of the femoral neck. This defect results in limb shortening and limping. In coxa valga there is an increase in femoral shaft neck angulation, which can lead to hip subluxation or dislocation. Coxa valga results from weakness of the adductor musculature.

A 72-year-old man visits the outpatient clinic with a complaint of severe pain when walking. Physical examination reveals the problems in his feet as shown in Fig. 5-5. What is the most likely diagnosis? A. Coxa varus B. Coxa valgus C. Genu valgus D. Genu vara E. Hallux valgus

E. The ankle jerk reflex involves S1 and S2 levels. L2 to L4 are involved in the patellar reflex. L5 is not a component of a deep tendon reflex.

A 72-year-old woman is admitted to the emergency department after an episode of stroke. During neurologic examination the patient shows no response to the ankle reflex test. Which of the following nerve roots is responsible for this reflex? A. L2 B. L3 C. L4 D. L5 E. S1

B. The lateral circumflex femoral artery arises from the deep femoral (profunda femoris) artery of the thigh and sends a descending branch down the length of the femur to anastomose with the superior medial genicular artery and the superior lateral genicular artery. The medial circumflex femoral artery is responsible for supplying blood to the head and neck of the femur, and it does not anastomose with distal vessels at the knee. The first perforating artery sends an ascending branch that anastomoses with the medial circumflex femoral and the inferior gluteal artery in the buttock. The inferior gluteal artery is a branch of the internal iliac; it has important anastomotic supply to the hip joint. The typically small descending genicular branch of the femoral artery is given off just proximal to the continuation of the femoral artery as the popliteal.

A 72-year-old woman is admitted to the hospital with a painful right foot. A CT scan examination reveals a thrombotic occlusion of the femoral artery in the proximal part of the adductor canal. Which artery will most likely provide blood supply to the leg through the genicular anastomosis? A. Medial circumflex femoral B. Descending branch of the lateral circumflex femoral C. First perforating branch of the deep femoral D. Inferior gluteal E. Descending genicular branch of femoral

C. The iliofemoral ligament ("inverted Y ligament of Bigelow") is the most important ligament reinforcing the joint anteriorly that would resist both hyperextension and lateral rotation at the hip joint. The pubofemoral ligament reinforces the joint inferiorly and limits extension and abduction. The ischiofemoral ligament reinforces the joint posteriorly and limits extension and medial rotation. Negative pressure in the acetabular fossa has nothing to do with resisting hyperextension of the hip joint but does help resist dislocation of the head of the femur. The gluteus maximus muscle extends and laterally rotates the thigh and does not particularly resist hyperextension.

A 72-year-old woman suffered a hip dislocation when she fell down the steps to her garage. Which of the following is most significant in resisting hyperextension of the hip joint? A. Pubofemoral ligament B. Ischiofemoral ligament C. Iliofemoral ligament D. Negative pressure in the acetabular fossa E. Gluteus maximus muscle

E. In infants and children until about the age of 8 years, the head of the femur gets its arterial supply by a direct branch of the obturator artery (variably, the medial circumflex femoral). The arterial supply reaches the head of the femur at the fovea capitis by traveling along the ligament of the head of the femur. This source of supply is replaced later by vessels such as branches of the ascending branch of the medial circumflex femoral that pass into foramina of the neck of the femur within the capsule of the hip joint. Similar branches can arise from the lateral circumflex femoral and gluteal arteries. The deep circumflex iliac artery arises from the external iliac artery and supplies branches to the ilium, the iliacus muscle, and lower portions of the abdominal wall. The acetabular branch of the obturator artery often provides the branch to the head of the femur, an artery that normally regresses early in life, so that it supplies only the immediate area of the fovea capitis. The descending branch of the lateral circumflex femoral supplies the vastus lateralis muscle and participates in anastomoses at the knee. The second perforating branch of the deep femoral artery often supplies the nutrient artery of the shaft of the femur.

A 75-year-old man is admitted to the emergency department with severe pain at his right hip and thigh. An MRI examination reveals avascular necrosis of the femoral head. Which of the following arteries is most likely injured, resulting in avascular necrosis? A. Deep circumflex iliac B. Acetabular branch of obturator C. Descending branch of lateral circumflex femoral D. First perforating branch of profunda femoris E. Ascending branch of medial circumflex femoral

C. An intracapsular femoral neck fracture causes avascular necrosis of the femoral head because the fracture damages the radicular branches of the medial and lateral circumflex arteries that pass beneath the ischiofemoral ligament and pierce the femoral neck. Until an individual reaches about 6 to 10 years of age, blood supply to the head of the femur is provided by a branch of the obturator artery that runs with the ligament of the head of the femur. Thereafter, the artery of the ligament of the head of the femur is insignificant. Intertrochanteric fracture of the femur would not damage the blood supply to the head of the femur but would cause complications because the greater trochanter is an attachment site for several gluteal muscles. During childhood the obturator artery provides the artery of the ligament of the head of the femur. Thrombosis of the obturator artery could result in muscular symptoms, although there are several collateral sources of blood supply in the thigh. Comminuted fracture of the extracapsular femoral neck would not ordinarily imperil the vascular supply.

A 75-year-old man is transported to the emergency department with severe pain of his right hip and thigh. A radiologic examination reveals avascular necrosis of the femoral head (Fig. 5-2). Which of the following conditions most likely occurred to produce avascular necrosis in this patient? A. Dislocation of the hip with tearing of the ligament of the head of the femur B. Intertrochanteric fracture of the femur C. Intracapsular femoral neck fracture D. Thrombosis of the obturator artery E. Comminuted fracture of the extracapsular femoral neck

C. The medial circumflex femoral artery is responsible for supplying blood to the head and neck of the femur by a number of branches that pass under the edge of the ischiofemoral ligament. This artery is most likely at risk for injury in an extracapsular fracture of the femoral neck. The inferior gluteal artery arises from the internal iliac and enters the gluteal region through the greater sciatic foramen, below the piriformis. The first perforating artery sends an ascending branch that anastomoses with the inferior gluteal artery in the buttock. The obturator artery arises from the internal iliac artery and passes through the obturator foramen. It commonly supplies the artery within the ligament of the head of the femur but is not likely to be patent in a person of this age. The superior gluteal artery arises from the internal iliac artery and enters through the greater sciatic foramen above the piriformis.

A 75-year-old woman is admitted to the hospital after falling in her bathroom. Radiologic examination reveals an extracapsular fracture of the femoral neck. Which artery is most likely at risk for injury? A. Inferior gluteal B. First perforating branch of deep femoral C. Medial circumflex femoral D. Obturator E. Superior gluteal

A. A lateral blow to the knee could result in injury to three structures in the knee: anterior cruciate ligament, medial collateral ligament, and the medial meniscus. When all three structures are involved it is collectively called the "unhappy triad." Anterior drawer sign is due to injury to the anterior cruciate ligament and denoted by anterior displacement of the tibia in relation to femur, similar to pulling out a drawer from a desk.

A lateral blow to the knee during a tackle in a football game injures a 24-year-old woman. Field examination reveals an "anterior drawer sign." An MRI demonstrates injury to several structures of the knee, including her medial meniscus. Which structure might also have been injured by the tackle? A. Medial collateral ligament B. Lateral collateral ligament C. Lateral meniscus D. Posterior cruciate ligament E. Tendon of the semitendinosus

A. A lateral blow to the knee often produces a trio of injuries referred to as the "unhappy triad." This involves damage to the anterior cruciate ligament, medial meniscus, and medial collateral ligament. The medial meniscus and medial collateral ligament are often damaged together, as they are tightly attached to each other. The lateral collateral ligament and lateral meniscus would not be damaged because a blow to the lateral knee would not put strain on these structures. Damage to the posterior cruciate ligament would produce a positive "posterior drawer sign" and is typically damaged during a blow to the medial side of the knee. The posterior cruciate ligament is stronger than the anterior and is only typically damaged when a person falls on the tibial tuberosity of a flexed knee. Tendon of semitendinosus is on the medial side of the knee but is not attached closely to the other structures or taut in this injury type.

A lateral blow to the knee during a tackle in a football game injures a 24-year-old woman. Field examination reveals an "anterior drawer sign." An MRI demonstrates injury to several structures of the knee, including her medial meniscus. Which structure might also have been injured by the tackle? A. Medial collateral ligament B. Lateral collateral ligament C. Lateral meniscus D. Posterior cruciate ligament E. Tendon of the semitendinosus

63 A 35-year-old male body builder has enlarged his shoulder muscles to such a degree that the size of the quadrangular space is greatly reduced. Which of the following structures would most likely be compressed in this condition? A. Axillary nerve B. Anterior circumflex humeral artery C. Cephalic vein D. Radial nerve E. Subscapular artery

A. The quadrangular space is bordered medially by the long head of the triceps brachii muscle, laterally by the surgical neck of the humerus, superiorly by the teres minor and subscapularis muscles, and inferiorly by the teres major muscle. Both the axillary nerve and posterior circumflex humeral vessels traverse this space. The other structures listed are not contained within the quadrangular space. The cephalic vein is located in the deltopectoral triangle, and the radial nerve is located in the triangular interval. GAS 718-720, 730; N 413; McM 139

48 A 68-year-old woman is examined by the senior resident in emergency medicine after she fell on a wet bathroom floor in the shopping center. Physical examination reveals a posterior displacement of the left distal wrist and hand. Radiographic examination reveals an oblique fracture of the radius. Which of the following is the most likely fracture involved in this case? A. Colles' fracture B. Scaphoid fracture C. Bennett's fracture D. Volkmann's ischemic contracture E. Boxer's fracture

A. A Colles' fracture is a fracture of the distal end of the radius. The proximal portion of the radius is displaced anteriorly, with the distal bone fragment projecting posteriorly. The displacement of the radius from the wrist often gives the appearance of a dinner fork, thus a Colles' fracture is often referred to as a "dinner fork" deformity. A scaphoid fracture results from a fracture of the scaphoid bone and would thus not cause displacement of the radius. This fracture usually occurs at the narrow aspect ("waist") of the scaphoid bone. Bennett's and boxer's fractures both result from fractures of the metacarpals (first and fifth, respectively). Volkmann's ischemic contracture is a muscular deformity that can follow a supracondylar fracture of the humerus, with arterial laceration into the flexor compartment of the forearm. Ischemia and muscle contracture, with extreme pain, accompany this fracture. GAS 771-774; N 439; McM 124

35 Examination of a 21-year-old female athlete with an injury of the radial nerve in the spiral groove would typically demonstrate which of the following physical signs? A. Weakness of thumb abduction and thumb extension B. Weakness of thumb opposition C. Inability to extend the elbow D. Paralysis of pronation of the hand E. Paralysis of abduction and adduction of the arm

A. Injury to the radial nerve in the spiral groove will paralyze the abductor pollicis longus muscle and both extensors of the thumb. This injury will also lead to wrist drop (inability to extend the wrist). Weakness of grip would also occur, although this is not mentioned in the question. If the wrist is flexed, finger flexion and grip strength are weakened because the long flexor tendons are not under tension. Note how much your strength of grip is increased when your wrist is extended versus when it is flexed. GAS 763, 818; N 427; McM 144

42 The 35-year-old woman has a hard breast nodule about 1 cm in diameter slightly above and lateral to the right areola of her right breast. A specific dye is injected into the tissue around the tumor, and an incision is made to expose the lymphatic vessels draining the area, for the lymphatic vessels to take up the dye, which is visible to the eye. The vessels can then be traced to surgically expose the lymph nodes receiving the lymph from the tumor. Which of the following nodes will most likely first encounter the lymph from the tumor? A. Anterior axillary (pectoral) nodes B. Rotter's interpectoral nodes C. Parasternal nodes along the internal thoracic artery and vein D. Central axillary nodes E. Apical or infraclavicular nodes

A. The anterior axillary (or anterior pectoral) nodes are the first lymph nodes to receive most of the lymph from the breast parenchyma, areola, and nipple. From there, lymph flows through central axillary, apical, and supraclavicular nodes in sequence. The interpectoral Rotter's nodes lie between the pectoral muscles and are, unfortunately, an alternate route in some patients, speeding the rate of metastasis. The parasternal nodes receive lymph from the medial part of the breast and lie along the internal thoracic artery and vein. GAS 748; N 403; McM 179

17 A mother tugs violently on her 4-year-old boy's hand to pull him out of the way of an oncoming car and the child screams in pain. Thereafter, it becomes obvious that the child cannot straighten his forearm at the elbow. When the child is seen in the emergency department, radiographic examination reveals a dislocation of the head of the radius. Which of the following ligaments is most likely directly associated with this injury? A. Anular B. Joint capsular C. Interosseous D. Radial collateral E. Ulnar collateral

A. The anular ligament is a fibrous band that encircles the head of the radius, forming a collar that fuses with the radial collateral ligament and articular capsule of the elbow. The anular ligament functions to prevent displacement of the head of the radius from its socket. In a child of this age the head of the radius is almost the same diameter as the shaft of the bone, so the head is relatively easy to dislocate. The joint capsule functions to allow free rotation of the joint and does not function in its stabilization. The interosseous membrane is a fibrous layer between the radius and ulna helping to hold these two bones together. The radial collateral ligament extends from the lateral epicondyle to the margins of the radial notch of the ulnar and the anular ligament of the radius. The ulnar collateral ligament is triangular ligament and extends from the medial epicondyle to the olecranon of the ulna.

176 A 25-year-old man falls on a slippery trail and injures his elbow and hand. Inspection reveals abrasions over the olecranon, medial epicondyle, and palm of the hand. Physical examination reveals decreased sensation with "pins and needles" (paraesthesia) along the ulnar border of the hand and medial one and a half digits. There is also weakness of finger abduction/ adduction, thumb adduction, and flexion at the DIP of the ring and little fingers. Which structure was most likely injured? A. Ulnar nerve at the medial epicondyle B. Ulnar nerve at Guyon's canal C. Median nerve in the cubital fossa D. Median nerve in the carpal tunnel E. Medial cord of brachial plexus in the axillary inlet

A. The deficits describe ulnar nerve damage close to its entry into the forearm. The ulnar nerve passes behind the medial epicondyle and is relatively unprotected, making this area prone to nerve injury. In the forearm, via its muscular branches, it innervates the flexor carpi ulnaris muscle and the medial half of the flexor digitorum profundus muscle. In the hand the deep branch of the ulnar nerve innervates the hypothenar muscles, adductor pollicis, abductor digiti minimi, flexor digiti minimi brevis, third and fourth lumbricals, opponens digiti minimi, and palmaris brevis muscles. The sensory innervation is to the fifth and medial half of the fourth digit and corresponding part of the hand, which can explain the deficits experienced by the patient. GAS 784; N 463; McM 151

24 A 79-year-old man has numbness in the middle three digits of his right hand and finds it difficult to grasp objects with that hand. He states that he retired 9 years earlier, after working as a carpenter for 50 years. He has atrophy of the thenar eminence (Fig. 6-3). Which of the following conditions is the most likely cause of the problems in his hand? A. Compression of the median nerve in the carpal tunnel B. Formation of the osteophytes that compress the ulnar nerve at the medial epicondyle C. Hypertrophy of the triceps brachii muscle compressing the brachial plexus D. Osteoarthritis of the cervical spine E. Repeated trauma to the ulnar nerve

A. The median nerve supplies sensory innervation to the thumb, index, and middle fingers and also to the lateral half of the ring finger. The median nerve also provides motor innervation to muscles of the thenar eminence. Compression of the median nerve in the carpal tunnel explains these deficits in conjunction with normal functioning of the flexor compartment of the forearm because these muscles are innervated by the median nerve proximal to the carpal tunnel. Also, sensory innervation in the proximal palm will be normal because the palmar branch of the radial nerve usually branches off proximal to the flexor retinaculum. The ulnar nerve is not implicated in these symptoms. It does not provide sensation to digits 1 to 3. Compression of the brachial plexus could not be attributed to pressure from the triceps brachii because this muscle is located distal to the plexus. In addition, brachial plexus symptoms would include other upper limb deficits, rather than the focal symptoms described in this case. Osteoarthritis of the cervical spine would also lead to increasing complexity of symptoms.

56 A 61-year-old man was hit in the midhumeral region of his left arm by a cricket bat. Physical examination reveals an inability to extend the wrist and loss of sensation on a small area of skin on the dorsum of the hand proximal to the first two fingers. What nerve supplies this specific region of the hand? A. Radial B. Posterior interosseous C. Lateral antebrachial cutaneous D. Medial antebrachial cutaneous E. Dorsal cutaneous of ulnar

A. The patient has suffered injury to the radial nerve in the midhumeral region. The nerve that provides sensation to the dorsum of the hand proximal to the thumb and index finger is the superficial branch of the radial nerve. The posterior interosseous nerve supplies a strip of skin on the back of the forearm and wrist extensors. The lateral antebrachial cutaneous nerve is a continuation of the musculocutaneous nerve and supplies the lateral side of the forearm. The medial antebrachial cutaneous is a direct branch of the medial cord and supplies skin of the medial side of the forearm. The dorsal cutaneous branch of the ulnar nerve supplies the medial side of the dorsum of the hand. GAS 761, 792; N 418; McM 144

10 Laboratory studies in the outpatient clinic on a 24-year-old woman included assessment of circulating blood chemistry. Which of the following arteries is most likely at risk during venipuncture at the cubital fossa? A. Brachial B. Common interosseous C. Ulnar D. Anterior interosseous E. Radial

A. The three chief contents of the cubital fossa are the biceps brachii tendon, brachial artery, and median nerve (lateral to medial). The common and anterior interosseous arteries arise distal to the cubital fossa; the ulnar and radial arteries are the result of the bifurcation of the brachial artery distal to the cubital fossa.

181 A 43-year-old woman visits the outpatient clinic with a painful hand. During physical examination, percussion over the flexor retinaculum causes a sharp pain in the lateral three and a half digits. This sign is indicative of which of the following conditions? A. Carpal tunnel syndrome B. De Quervain's tenosynovitis C. Thoracic outlet syndrome D. Mallet finger E. Radial nerve damage

A. Tinel's sign is used to aid in the diagnosis of carpal tunnel syndrome. It is performed by lightly percussing above the carpal tunnel where the median nerve is located. De Quervain's tenosynovitis describes tenosynovitis of the sheath or tunnel that surrounds tendons that control the thumb. It is tested using Finkelstein's test, where the examiner grips the thumb of the individual being tested and ulnar deviates the hand sharply. Thoracic outlet syndrome is tested using Adson's test. Mallet finger describes a finger deformity due to extensor digitorum tendons. Radial nerve damage is tested by evaluating the cutaneous distribution of the radial nerve or by testing the muscles innervated by the radial nerve. GAS 798; N 449; McM 157

E. Plantar flexion is mostly due to the gastrocnemius and soleus muscles, which are supplied by the tibial nerve. The tibial nerve leaves the popliteal fossa by passing deep to the gastrocnemius and soleus muscles and lies posterior to the popliteal artery. Therefore a hematoma of the popliteal artery will also compress the nerve. Dorsiflexion of the foot is due to contraction of the muscles in the anterior compartment of the leg

After being struck from behind by a motor vehicle, a 55-year-old man presents to the hospital with a swelling of his right knee. Imaging reveals a large hematoma of the popliteal artery compressing his tibial nerve. Upon neurologic examination which movement would likely be diminished in strength? A. Dorsiflexion of the foot B. Flexion of the thigh C. Extension of the digits D. Extension of the leg E. Plantar flexion of the foot

D. The piriformis muscle arises from the pelvic surface of the sacrum, passes through the greater sciatic notch, and inserts at the greater trochanter. It is considered the "anatomical key" to gluteal anatomy; the greater sciatic foramen is the "door." The gluteus medius lies posterior to the piriformis. The sciatic nerve emerges from the greater sciatic foramen, normally through the infrapiriform's space. The spine of the ischium separates the greater and lesser sciatic foramina.

After dividing the overlying superficial tissues and gluteal musculature in a 68-year-old female patient, the orthopedic surgeon carefully identified the underlying structures while performing a total hip arthroplasty. The key landmark in the gluteal region, relied upon in surgical explorations of this area, is provided by which of the following structures? A. Gluteus medius B. Obturator internus tendon C. Sciatic nerve D. Piriformis muscle E. Spine of the ischium

C. The deep nodes are located beneath the deep fascia (fascia cribrosa) and lie along the medial side of the femoral vein. The presence of swollen inguinal lymph nodes is an important clinical sign because swelling may indicate an infection in the lower extremities. They then drain superiorly to the external iliac lymph nodes. The superficial nodes lie in the superficial fascia below the inguinal ligament and can be divided into horizontal and vertical groups. External iliac lies along the external iliac vessels; they are arranged in groups of three (anteriorly, medially, and lateral to vessels).

After suffering a deep stab wound to her posterior thigh, a 22-year-old woman presents to the emergency department. The wound is closed but the patient develops a subsequent wound infection. Which group of lymph nodes first receives drainage from this deep wound area, and would most likely be enlarged in this patient? A. External iliac B. Superficial inguinal C. Deep inguinal D. Common iliac E. Internal iliac

A. The patient has bowlegs, or genu varus. The opposite of this is genu valgus, or knock knee. The normal angle between the femoral shaft and femoral neck is between 120 and 135 degrees. In coxa vara the angle between the shaft and neck of the femur is less than 120 degrees. This can result from fractures, other injuries, or congenital softness of the bone of the femoral neck. This defect results in limb shortening and limping. In coxa valga there is an increase in femoral shaft neck angulation, which can lead to hip subluxation or dislocation. Coxa valga results from weakness of the adductor musculature. Hallux valgus is commonly known as bunion. In this deformity the big toe points toward the little toe and may override the second toe; the base of the first metatarsal points medially, with a swollen bursal sac at the metatarsophalangeal joint. Excess bony growth of the distal protruding part of the metatarsal bone can also occur. Bunions occur only rarely in people who do not routinely wear shoes.

An 81-year-old man is admitted to the emergency department with severe pain in his knees. The patient has a long history of osteoarthritis. Radiologic examination reveals degeneration of the joints of his lower limbs. The degeneration is more severe on the medial side of the knees, which causes his knees to be bowed outward when he stands upright. Which of the following terms best describes the condition of his knees? A. Genu varus B. Genu valgus C. Coxa varus D. Coxa valgus E. Hallux valgus

C. The obturator artery provides the artery within the ligament of the head of the femur (in about 60% of cases), the artery that supplies the head of the femur, primarily during childhood, later becoming atretic. In the adult this artery supplies only the area of the fovea of the head of the femur. The ligament of the head of the femur arises from the acetabular notch, thereafter receiving the little artery. In some individuals the medial circumflex femoral gives origin to the artery of the head. In the adult the arterial supply of the neck and head is provided by intracapsular branches of the medial circumflex femoral and lateral circumflex femoral arteries that pierce the neck of the femur, with some supply also from the gluteal arteries. The lateral circumflex femoral artery arises from the deep femoral and supplies the vastus lateralis. The pudendal artery arises from the internal iliac and provides blood supply for the structures of the perineum. Quite often, when an older patient with osteoporosis has a hip fracture, the femoral neck may have fractured, precipitating a fall, rather than the fall resulting in the hip fracture.

An 82-year-old grandmother slipped on the polished floor in her front hall and was transported to the emergency department and admitted for examination with a complaint of great pain in her right lower limb. During physical examination it is observed by the resident that the right lower limb is laterally rotated and noticeably shorter than her left limb. Radiologic examination reveals an intracapsular fracture of the femoral neck. Which of the following arteries supplies the head of the femur in early childhood but no longer in a patient of this age? A. Superior gluteal B. Lateral circumflex femoral C. A branch of the obturator artery D. Inferior gluteal E. Internal pudendal

49 A 34-year-old female skier was taken by ambulance to the hospital after she struck a tree on the ski slope. Imaging gives evidence of a shoulder separation. Which of the following typically occurs in this kind of injury? A. Displacement of the head of the humerus from the glenoid cavity B. Partial or complete tearing of the coracoclavicular ligament C. Partial or complete tearing of the coracoacromial ligament D. Rupture of the transverse scapular ligament E. Disruption of the glenoid labrum

B. In shoulder separation, either or both the acromioclavicular and coracoclavicular ligaments can be partially or completely torn through. The acromioclavicular joint can be interrupted and the distal end of the clavicle may deviate upward in a complete separation, while the upper limb droops away inferiorly, causing a "step off" that can be palpated and sometimes observed. Displacement of the head of the humerus is shoulder dislocation, not separation. The coracoacromial ligament is not torn in separation (but it is sometimes used in the repair of the torn coracoclavicular ligament). Disruption of the glenoid labrum often accompanies shoulder dislocation. GAS 711; N 411; McM 136

20 A 25-year-old male athlete is admitted to the emergency department after a bad landing while performing the pole vault. Radiographic examination of his hand reveals a fractured carpal bone in the floor of the anatomic snuffbox (Fig. 6-2). Which bone has most likely been fractured? A. Triquetral B. Scaphoid C. Capitate D. Hamate E. Trapezoid

B. The anatomic snuffbox is formed by the tendons of the extensor pollicis brevis, the abductor pollicis longus, and the extensor pollicis longus. The floor is formed by the scaphoid bone, and it is here that one can palpate for a possible fractured scaphoid.

16 A 55-year-old man is examined in a neighborhood clinic after receiving blunt trauma to his right axilla in a fall. He has difficulty elevating the right arm above the level of his shoulder. Physical examination shows that the inferior angle of his right scapula protrudes more than the lower part of the left scapula. The right scapula protrudes far more when the patient pushes against the examiner's hand with resistance. Which of the following neural structures has most likely been injured? A. The posterior cord of the brachial plexus B. The long thoracic nerve C. The upper trunk of the brachial plexus D. The site of origin of the middle and lower subscapular nerves E. Spinal nerve ventral rami C7, C8, and T1

B. The condition described in this patient is called "winging" of the scapula. "Winging" of the scapula occurs when the medial border of the scapula lifts off the chest wall when the patient pushes against resistance, such as a vertical wall. The serratus anterior muscle holds the medial border of the scapula against the chest wall and is innervated by the long thoracic nerve. The serratus anterior assists in abduction of the arm above the horizontal plane by rotating the scapula so that the glenoid fossa is directed more superiorly.

28 A 54-year-old female marathon runner presents with pain in her right wrist that resulted when she fell with force on her outstretched hand. Radiographic studies indicate an anterior dislocation of a carpal bone (Fig. 6-4). Which of the following bones is most likely dislocated? A. Capitate B. Lunate C. Scaphoid D. Trapezoid E. Triquetrum

B. The lunate is the most commonly dislocated carpal bone because of its shape and relatively weak ligaments anteriorly. Dislocations of the scaphoid and triquetrum are relatively rare. The trapezoid and capitate bones are located in the distal row of the carpal bones.

8 A 32-year-old woman is admitted to the emergency department after an automobile collision. Radiologic examination reveals multiple fractures of the humerus. Flexion and supination of the forearm are severely weakened. She also has loss of sensation on the lateral surface of the forearm. Which of the following nerves has most likely been injured? A. Radial B. Musculocutaneous C. Median D. Lateral cord of brachial plexus E. Lateral cutaneous nerve of the forearm

B. The musculocutaneous nerve supplies the biceps brachii and brachialis muscles, which are the flexors of the forearm at the elbow. The musculocutaneous nerve continues as the lateral antebrachial cutaneous nerve, which supplies sensation to the lateral side of the forearm (with the forearm in the anatomic position). The biceps brachii muscle is the most powerful supinator muscle. Injury to this nerve would result in weakness of supination and forearm flexion and lateral forearm sensory loss. Injury to the radial nerve would result in weakened extension and a characteristic wrist drop. Injury to the median nerve causes paralysis of flexor digitorum superficialis muscle and other flexors in the forearm and results in a characteristic flattening of the thenar eminence. The lateral cord of the brachial plexus gives origin both to the musculocutaneous and lateral pectoral nerves. There is no indication of pectoral paralysis or weakness. Injury to the lateral cord can result in weakened flexion and supination in the forearm, and weakened adduction and medial rotation of the arm. The lateral cutaneous nerve of the forearm is a branch of the musculocutaneous nerve and does not supply any motor innervation. Injury to the musculocutaneous nerve alone is unusual but can follow penetrating injuries.

179 A 52-year-old man is admitted to the emergency department after a fall. Imaging studies show a fracture at the neck of the radius and a hematoma at the fracture site. Examination reveals weakness of wrist extension, abduction and extension of the thumb, and extension of the metacarpophalangeal and interphalangeal joints of the fingers. However, there was no sensory deficit. Which nerve is most likely affected? A. Anterior interosseous B. Posterior interosseous C. Radial D. Ulnar E. Superficial radial

B. The posterior interosseus nerve innervates the extensors of the wrist, abductor pollicis longus, extensor indicis, digiti minimi, and extensor pollicis longus muscles. The posterior interosseus nerve does not have any cutaneous branches, making it the best answer. The anterior interosseous nerve innervates flexors of the forearm. Although the radial nerve does give rise to the posterior interosseous nerve, there are no sensory deficits mentioned, so the radial nerve proper was not affected. The ulnar nerve also innervates flexors in the hand but since no sensory deficits were noted ulnar nerve injury can be ruled out. The superficial radial nerve is a purely cutaneous nerve. GAS 784, 787; N 466; McM 152

185 A 67-year-old woman with osteoporosis injured her left shoulder/arm in a fall. Examination reveals bruising and dimpling of the upper part of the arm with exquisite tenderness over the affected area. The shoulder radiograph is shown in Figure 6-13. Which nerve is most likely to be injured? A. Radial B. Axillary C. Ulnar D. Median E. Musculocutaneous

B. The radiograph shows a fracture of the humerus at the surgical neck. The bruising and dimpling of the upper arm would result from this injury. The axillary nerve leaves the brachial plexus as a terminal branch of the posterior cord. It passes through the quadrangular space and wraps around the head of the humerus on its way to provide innervation to the teres minor, the deltoid, and the portion of skin over the lower aspect of the deltoid that is known as the "sergeant's patch." The radial nerve travels in the radial groove along the shaft of the humerus and would be injured in a fracture of the shaft of the humerus. The ulnar nerve would be injured in a fracture of the medial epicondyle. The median nerve travels too deep to be injured here and could be compressed at the carpal tunnel or at the cubital fossa. The musculocutaneous nerve is likewise within the tissue and will not be affected by this injury. GAS 704-705; N 418; McM 136

5 While walking to his classroom building, a first year medical student slipped on the wet pavement and fell against the curb, injuring his right arm. Radiographic images showed a midshaft fracture of the humerus. Which pair of structures was most likely injured at the fracture site? A. Median nerve and brachial artery B. Axillary nerve and posterior circumflex humeral artery C. Radial nerve and deep brachial artery D. Suprascapular nerve and artery E. Long thoracic nerve and lateral thoracic artery

C. A midshaft humeral fracture can result in injury to the radial nerve and deep brachial artery because they lie in the spiral groove located in the midshaft. Injury to the median nerve and brachial artery can be caused by a supracondylar fracture that occurs by falling on an outstretched hand and partially flexed elbow. A fracture of the surgical neck of the humerus can injure the axillary nerve and posterior circumflex humeral artery. The suprascapular artery and nerve can be injured in a shoulder dislocation. The long thoracic nerve and lateral thoracic artery may be damaged during a mastectomy procedure.

37 A 60-year-old male butcher accidentally slashed his wrist with his butcher's knife, partially dividing the ulnar nerve. Which of the following actions would most likely be lost as a result of this injury? A. Flexion of the proximal interphalangeal joint of the fifth digit (little finger) B. Extension of the thumb C. Adduction of the fifth digit D. Abduction of the thumb E. Opposition of the thumb

C. Adduction of the fifth digit is produced by contraction of the third palmar interosseous muscle. All of the interossei are innervated by the deep branch of the ulnar nerve. Flexion of the proximal interphalangeal joint is a function of the flexor digitorum superficialis, supplied by the median nerve. Opposition of the thumb is a function of the opponens pollicis, supplied by the recurrent branch of the median nerve. GAS 808; N 452; McM 162

45 A 27-year-old male painter is admitted to the hospital after falling from a ladder. Physical examination reveals that the patient is unable to abduct his arm more than 15 degrees and he cannot rotate the arm laterally. A radiographic examination reveals an oblique fracture of the humerus. He has associated sensory loss over the shoulder area. Which of the following injuries will most likely correspond to the symptoms of the physical examination? A. Fracture of the medial epicondyle B. Fracture of the glenoid fossa C. Fracture of the surgical neck of the humerus D. Fracture of the anatomic neck of the humerus E. Fracture of the middle third of the humerus

C. Fracture of the surgical neck of the humerus often injures the axillary nerve, which innervates the deltoid and teres minor muscles. Abduction of the humerus between 15 degrees and the horizontal is performed by the deltoid muscle. Lateral rotation of the humerus is mainly performed by the deltoid muscle, teres minor, and the infraspinatus. The deltoid and teres minor are both lost in this case. Fracture of the glenoid fossa would lead to drooping of the shoulder. Fracture of the anatomic neck of the humerus will similarly lead to a drooping of the shoulder but would not necessarily affect abduction of the humerus. It is also quite unusual. Fracture of the middle third of the humerus would most likely injure the radial nerve. The ulnar nerve would be potentially compromised in a fracture of the medial epicondyle of the humerus. GAS 705; N 405; McM 140

180 After falling down concrete steps, a 42-year-old woman complains of tingling and numbness along the medial border of her left hand. Neurological examination reveals several abnormalities including Froment's sign. Weakness of which of the following muscles explains the presence of Froment's sign? A. First dorsal interosseous muscle B. Opponens pollicis C. Adductor pollicis D. Flexor pollicis longus E. Flexor pollicis brevis

C. Froment's test is a special test of the wrist to aid in the diagnosis of ulnar nerve palsy. The test evaluates the function of adductor pollicis muscle. The first dorsal interosseous is tested by adducting the index middle and ring fingers against resistance. The opponens pollicis muscle is evaluated using pulp-topulp opposition and the squeeze test. The flexor pollicis longus muscle is a flexor of the thumb and is tested by instructing the patient to flex the tip of the thumb against resistance while the proximal phalanx is held in extension. The flexor pollicis brevis muscle flexes the thumb at the metacarpophalangeal joint and is tested by asking the individual to flex the proximal phalanx of the thumb against resistance. GAS 814-816; N 452; McM 159

44 A 44-year-old woman is diagnosed with radial nerve palsy. When muscle function is examined at the metacarpophalangeal (MCP), proximal interphalangeal (PIP), and distal interphalangeal (DIP) joints, what findings are most likely to be present? A. Inability to abduct the digits at the MCP joint B. Inability to adduct the digits at the MCP joint C. Inability to extend the MCP joints only D. Inability to extend the MCP, PIP, and DIP joints E. Inability to extend the PIP and DIP joints

C. Inability to extend the metacarpophalangeal (MCP) joints. The tendons of the extensor digitorum and extensor digiti minimi muscles, innervated by the radial nerve, are responsible for extension of the MCP and, to a much lesser degree, the proximal (PIP) and distal (DIP) interphalangeal joints. Abduction and adduction of the MCP joints are functions of the interossei, all of which are innervated by the deep ulnar nerve. Extension of the PIP and DIP joints is performed by the lumbricals and interossei. The first two lumbricals are supplied by the median nerve; the other lumbricals and the interossei, by the deep branch of the ulnar nerve. GAS 792, 814-818; N 451; McM 155

9 A 24-year-old female medical student was bitten at the base of her thumb by her dog. The wound became infected and the infection spread into the radial bursa. The tendon(s) of which muscle will most likely be affected? A. Flexor digitorum profundus B. Flexor digitorum superficialis C. Flexor pollicis longus D. Flexor carpi radialis E. Flexor pollicis brevis

C. Tenosynovitis can be due to an infection of the synovial sheaths of the digits. Tenosynovitis in the thumb may spread through the synovial sheath of the flexor pollicis longus tendon, also known as the radial bursa. The tendons of the flexor digitorum superficialis and profundus muscles are enveloped in the common synovial flexor sheath or ulnar bursa. Neither the flexor carpi radialis nor flexor pollicis brevis tendons are contained in synovial flexor sheaths.

12 A 49-year-old woman who had suffered a myocardial infarction must undergo a bypass graft procedure using the internal thoracic artery. Which vessels will most likely continue to supply blood to the anterior part of the upper intercostal spaces? A. Musculophrenic B. Superior epigastric C. Posterior intercostal D. Lateral thoracic E. Thoracodorsal

C. The anterior intercostal arteries are twelve small arteries, two in each of the upper six intercostal spaces at the upper and lower borders. The upper artery lying in each space anastomoses with the posterior intercostal arteries, whereas the lower one usually joins the collateral branch of the posterior intercostal artery. The musculophrenic artery is a terminal branch of the internal thoracic artery (also known as the internal mammary artery), and it supplies the pericardium, diaphragm, and muscles of the abdominal wall. It anastomoses with the deep circumflex iliac artery. The superior epigastric artery is the other terminal branch of the internal thoracic artery, and it supplies the diaphragm, peritoneum, and the anterior abdominal wall and anastomoses with the inferior epigastric artery. The lateral thoracic artery runs along the lateral border of the pectoralis minor muscle and supplies the pectoralis major, pectoralis minor, and serratus anterior. The thoracodorsal artery accompanies the thoracodorsal nerve in supplying the latissimus dorsi muscle and lateral thoracic wall.

30 A 23-year-old female maid was making a bed in a hotel bedroom. As she straightened the sheet by running her right hand over the surface with her fingers extended, she caught the end of the index finger in a fold. She experienced a sudden, severe pain over the base of the terminal phalanx. Several hours later when the pain had diminished, she noted that the end of her right index finger was swollen and she could not completely extend the terminal interphalangeal joint. Which one of the following structures within the digit was most likely injured? A. The proper palmar digital branch of the median nerve B. The vinculum longa C. The insertion of the tendon of the extensor digitorum onto the base of the distal phalanx D. The insertion of the flexor digitorum profundus tendon E. The insertion of the flexor digitorum superficialis tendon

C. The contraction of the extensor mechanism produces extension of the distal interphalangeal joint. When it is torn from the distal phalanx, the digit is pulled into flexion by the flexor digitorum profundus muscle. If a piece of the distal phalanx is attached to the torn tendon it is an avulsion fracture. The proper palmar digital branches of the median nerve supply lumbrical muscles and carry sensation from their respective digits. Vincula longa are slender, bandlike connections from the deep flexor tendons to the phalanx that can carry blood supply to the tendons. The insertions of the flexor digitorum superficialis and profundus are on the flexor surface of the middle and distal phalanges, respectively, and act to flex the interphalangeal joints.

41 A 35-year-old male wrestler is admitted to the emergency department with excruciating pain in his right shoulder and proximal arm. During physical examination, the patient clutches the arm at the elbow with his contralateral hand and is unable to move the injured limb. Radiographic studies show that the patient has a dislocation of the humerus at the glenohumeral joint. Which of the following conditions is the most likely? A. The head of the humerus is displaced anteriorly B. The head of the humerus is displaced posteriorly C. The head of the humerus is displaced inferiorly D. The head of the humerus is displaced superiorly E. The head of the humerus is displaced medially

C. The head of the humerus is displaced inferiorly because in that location it is not supported by rotator cuff muscle tendons or the coracoacromial arch. It is also pulled anteriorly (relative to the tendon of the triceps brachii) beneath the coracoid process by pectoralis and subscapularis muscles. It would not be displaced posteriorly because it is supported by the teres minor and infraspinatus muscle tendons. It would not be displaced superiorly because the coracoacromial ligament and supraspinatus reinforce in that direction. A medial dislocation is blocked by the subscapularis tendon. GAS 712; N 408; McM 136

51 The shoulder of a 44-year-old deer hunter had been penetrated by a bolt released from a crossbow. The bolt had transected the axillary artery just beyond the origin of the subscapular artery. A compress is placed on the wound with deep pressure. After a clamp is placed on the bleeding artery, thought is given to the anatomy of the vessel. What collateral arterial pathways are available to bypass the site of injury? A. Suprascapular with circumflex scapular artery B. Dorsal scapular with thoracodorsal artery C. Posterior circumflex humeral artery with deep brachial artery D. Lateral thoracic with brachial artery E. Supreme thoracic artery with thoracoacromial artery

C. The injury has occurred just beyond the third part of the axillary artery. The only collateral arterial channel between the third part of the axillary artery and the brachial artery is between the posterior circumflex humeral and the ascending branch of the profunda brachii, and this anastomotic path is often inadequate to supply the arterial needs of the limb. The posterior circumflex humeral arises from the third part of the axillary artery. It typically anastomoses with a variably small, ascending branch of the profunda brachii branch of the brachial artery. The suprascapular artery anastomoses with the circumflex scapular deep to the infraspinatus. The dorsal scapular artery (passing beneath the medial border of the scapula) has no anastomosis with thoracodorsal within the scope of the injury. The lateral thoracic artery has no anastomoses with the brachial artery. The supreme thoracic artery (from first part of axillary) has no helpful anastomoses with the thoracoacromial (second part of axillary) (GAS Figs. 7-39 and 7-50). GAS 719-721; N 420; McM 139

186 A 14-year-old boy falls on his outstretched hand. Examination reveals point tenderness above the humeral epicondyles and a pulsatile mass just above the cubital fossa. Neurological examination of the upper limb reveals weakness of pronation, wrist flexion, and grip strength. Only flexion at the distal interphalangeal joints of the ring and little fingers is intact. Thumb flexion and opposition are also impaired. A radiograph reveals a supracondylar fracture of the humerus. Which structures injured by the fracture best account for the findings? A. Axillary nerve and posterior circumflex humeral artery B. Radial nerve and deep brachial artery C. Median nerve and brachial artery D. Superficial radial nerve and radial artery E. Ulnar nerve and ulnar artery

C. The median nerve and brachial artery were injured. Injury to the median nerve is indicated by the weakness of pronation, wrist flexion, and grip strength. The median nerve innervates the muscles that govern or carry out these movements. The brachial artery is near the median nerve and can also be injured. Flexion at the distal interphalangeal joints of the ring and little fingers indicates that the ulnar nerve is intact. The radiograph indicates a supracondylar fracture of the humerus, which is the region in which the median nerve passes. A fracture at the surgical neck would injure the axillary nerve and posterior circumflex humeral artery. A fracture of the shaft of the humerus would injure the radial nerve and deep brachial artery. The superficial radial nerve and artery would be damaged by injury over or in the anatomical snuffbox. GAS 756, 761, 768; N 434; McM 149

7 A 52-year-old female band director suffered problems in her right arm several days after strenuous field exercises for a major athletic tournament. Examination in the orthopedic clinic reveals wrist drop and weakness of grasp but normal extension of the elbow joint. There is no loss of sensation in the affected limb. Which nerve was most likely affected? A. Ulnar B. Anterior interosseous C. Posterior interosseous D. Median E. Superficial radial

C. The radial nerve descends posteriorly between the long and lateral heads of the triceps brachii muscle and passes inferolaterally on the back of the humerus between the medial and lateral heads of the triceps brachii muscle. It eventually enters the anterior compartment and descends to enter the cubital fossa, where it divides into superficial and deep branches. The deep branch of the radial nerve winds laterally around the radius and runs between the two heads of the supinator muscle and continues as the posterior interosseous nerve, innervating extensor muscles of the forearm. Because this injury does not result in loss of sensation over the skin of the upper limb, it is likely that the superficial branch of the radial nerve is not injured. If the radial nerve were injured very proximally, the woman would not be able to extend her elbow. The branches of the radial nerve to the triceps brachii muscle arise proximal to where the nerve runs in the spiral groove. The anterior interosseous nerve arises from the median nerve and supplies the flexor digitorum profundus, flexor pollicis longus, and pronator quadratus muscles, none of which seem to be injured in this example. Injury to the median nerve causes a characteristic flattening (atrophy) of the thenar eminence.

25 A 13-year-old boy is brought to the emergency department after losing control during a motorbike race in which he was hit by several of the other racers. Physical examination reveals several cuts and bruises. He is unable to extend the left wrist, fingers, and thumb, although he can extend the elbow. Sensation is lost in the lateral half of the dorsum of the left hand. Which of the following nerves has most likely been injured to result in these signs, and in what part of the arm is the injury located? A. Median nerve, anterior wrist B. Median nerve, arm C. Radial nerve, midhumerus D. Ulnar nerve, midlateral forearm E. Ulnar nerve, midpalmar region

C. The radial nerve innervates the extensor compartments of the arm and the forearm. It supplies the triceps brachii proximal to the spiral groove, so elbow extension is intact here. It also provides sensory innervation to much of the posterior arm and forearm as well as the dorsal thumb, index, and middle fingers up to the level of the fingernails. Symptoms are described only in the distal limb due to the midhumeral location of the lesion. The median nerve innervates flexors of the forearm and thenar muscles and provides sensory innervation to the lateral palmar hand. The ulnar nerve supplies only the flexor carpi ulnaris and the medial half of the flexor digitorum profundus in the forearm. Additionally, its sensory distribution is to both the palmar and dorsal aspects of the medial hand. It does not supply extensor muscles.

38 A 23-year-old male medical student fell asleep in his chair with Netter's Atlas wedged into his axilla. When he awoke in the morning, he was unable to extend his wrist or fingers. Movements of the ipsilateral shoulder joint appear to be normal. Which of the following nerves was most likely compressed, producing the symptoms described? A. Lateral cord of the brachial plexus B. Medial cord of the brachial plexus C. Radial nerve D. Median nerve E. Lateral and medial pectoral nerves

C. The radial nerve is the most likely nerve compressed to cause these symptoms. This type of nerve palsy is often called "Saturday night palsy." One reason for this nickname is that people would supposedly fall asleep after being intoxicated on a Saturday night with their arm over the back of a chair or bench, thereby compressing the nerve in the spiral groove. The radial nerve innervates all of the extensors of the elbow, wrist, and fingers. It innervates the triceps brachii muscle but the motor branch typically comes off proximal to the site of compression, so the patient can still extend the elbow. Paralysis of the lateral cord of the brachial plexus would result in loss of the musculocutaneous nerve and the pectoral nerves, which do not mediate extension of the forearm or hand. The medial cord of the brachial plexus branches into the median nerve and ulnar nerve. Neither of these nerves innervates muscles that control extension. The median nerve innervates flexors of the forearm and the thenar muscles. The lateral and median pectoral nerves do not extend into the arm and innervate the pectoralis major and minor muscles. GAS 761-763; N 415; McM 139

57 A 45-year-old woman is admitted to the hospital with neck pain. An MRI examination reveals a herniated disc in the cervical region. Physical examination reveals weakness in wrist extension and paraesthesia on the back of her arm and forearm. Which of the following spinal nerves is most likely injured? A. C5 B. C6 C. C7 D. C8 E. T1

C. The seventh cervical nerve makes a major contribution to the radial nerve, and this nerve is the prime mover in wrist extension. The dermatome of C7 is in the region described. GAS 745, 787, 790; N 416; McM 153

61 A 45-year-old woman is admitted to the hospital with neck pain. A computed tomography (CT) scan reveals a tumor on the left side of her oral cavity. The tumor and related tissues are removed and a radical neck surgical procedure is performed. Two months postoperatively the patient's left shoulder droops quite noticeably. Physical examination reveals distinct weakness in turning her head to the right and impairment of abduction of her left upper limb to the level of the shoulder. Which of the following structures was most likely injured during the radical neck surgery? A. Suprascapular nerve B. Long thoracic nerve C. Spinal accessory nerve D. The junction of spinal nerves C5 and C6 of the brachial plexus E. Radial nerve

C. The spinal accessory nerve (CN XI) arises from the ventral rootlets of C1 to C4 that ascend through the foramen magnum to then exit the cranial cavity through the jugular foramen. It innervates the sternocleidomastoid and trapezius muscles, which function in head rotation and raising of the shoulders. The suprascapular nerve receives fibers from C5-6 (occasionally from C4 if the plexus is "prefixed") and innervates the supraspinatus muscle, which is responsible for the first 15 degrees of arm abduction. Erb's point of the brachial plexus is at the union of C5-6 spinal nerves. The long thoracic nerve arises from plexus routes C5, 6, and 7, and supplies the serratus anterior. GAS 714; N 33; McM 132

36 A 58-year-old convenience store operator had received a superficial bullet wound to the soft tissues on the medial side of the elbow in an attempted robbery. A major nerve was repaired at the site where it passed behind the medial epicondyle. Bleeding was stopped from an artery that accompanied the nerve in its path toward the epicondyle. Vascular repair was performed on this small artery because of its important role in supplying blood to the nerve. Which of the following arteries was most likely repaired? A. The profunda brachii artery B. The radial collateral artery C. The superior ulnar collateral artery D. The inferior ulnar collateral artery E. The anterior ulnar recurrent artery

C. The superior ulnar collateral branch of the brachial artery accompanies the ulnar nerve in its path posterior to the medial epicondyle and is important in the blood supply of the nerve. The profunda brachii artery passes down the arm with the radial nerve. The radial collateral artery arises from the profunda brachii artery and anastomoses with the radial recurrent branch of the radial artery proximal to the elbow laterally. The inferior ulnar collateral artery arises from the brachial artery and accompanies the median nerve into the forearm. The anterior ulnar recurrent artery arises from the ulnar artery and anastomoses with the inferior ulnar collateral artery anterior to the elbow. GAS 756; N 435; McM 149

3 A 48-year-old female court stenographer is admitted to the orthopedic clinic with symptoms of carpal tunnel syndrome, with which she has suffered with for almost a year. Which muscles most typically become weakened in this condition? A. Dorsal interossei B. Lumbricals III and IV C. Thenar D. Palmar interossei E. Hypothenar

C. The thenar muscles (and lumbricals I and II) are innervated by the median nerve, which runs through the carpal tunnel. The carpal tunnel is formed anteriorly by the flexor retinaculum and posteriorly by the carpal bones. Carpal tunnel syndrome is caused by a compression of the median nerve, due to reduced space in the carpal tunnel. The carpal tunnel contains the tendons of flexor pollicis longus, flexor digitorum profundus, and flexor digitorum superficialis muscles and their synovial sheaths. The dorsal interossei, lumbricals III and IV , palmar interossei, and hypothenar muscles are all innervated by the ulnar nerve. GAS 798, 808; N 452; McM 159

178 A 54-year-old woman is admitted to the emergency department after a serious motor vehicle accident. Physical examination shows soft tissue edema and bruising around the neck. A radiograph of the humeroscapular region reveals a fracture of the midhumerus. Which of the following areas will most likely have impaired or absent sensation? A. Lateral aspect of the forearm B. Medial aspect of the arm C. Medial aspect of the arm and forearm D. Posterior aspect of the forearm E. Lateral and posterior aspect of the forearm

D. In the midshaft region of the humerus the radial nerve runs in the radial groove; fracture of the humerus at this point will likely impinge directly on the radial nerve, producing a sensory deficit along the posterior aspect of the forearm. The lateral aspect of the forearm is innervated by the lateral antebrachial cutaneous nerve of the forearm, which comes from the musculocutaneous nerve. These nerves may not be affected by a midshaft fracture of the humerus because they are well separated from the bone by muscle. The medial aspect of the arm and forearm is supplied by the intercostobrachial nerve and the medial antebrachial cutaneous nerve that takes its origin from the medial cord of the brachial plexus where it runs superficially, making it extremely difficult to injure both nerves during a midshaft fracture of the humerus. The lateral and posterior aspect of the forearm is an unlikely choice because the displaced bone not only has to impinge on the radial nerve but must also affect the very superficially located lateral antebrachial cutaneous nerve as well. GAS 763; N 465; McM 144

43 During a fight in a tavern, a 45-year-old male construction worker received a shallow stab wound from a broken beer bottle at a point near the middle of the left posterior triangle of his neck. Upon physical examination, it is observed that the left shoulder is drooping lower than the right shoulder, and the superior angle of the scapula juts out slightly. Strength in turning the head to the right or left appears to be symmetric. Which of the following nerves is most likely injured? A. Suprascapular nerve in the supraspinous fossa B. The terminal segment of the dorsal scapular nerve C. The upper trunk of the brachial plexus D. The spinal accessory nerve in the posterior cervical triangle E. The thoracodorsal nerve in the axilla

D. The left spinal accessory nerve (CN XI) has been injured distal to the sternocleidomastoid muscle, resulting in paralysis of the trapezius, allowing the shoulder to droop and the superior angle to push out posteriorly. The sternocleidomastoid muscles are intact, as demonstrated by symmetry in strength in turning the head to the right and left. There is no indication of paralysis of the lateral rotators of the shoulder or elbow flexors (suprascapular nerve or upper trunk). Thoracodorsal nerve injury would result in paralysis of the latissimus dorsi muscle, an extensor, and medial rotator of the humerus. GAS 714; N 413; McM 132

26 A 17-year-old boy has weakness of elbow flexion and supination of the left hand after sustaining a knife wound in that arm in a street fight. Examination in the emergency department indicates that a nerve has been severed. Which of the following conditions will also most likely be seen during physical examination? A. Inability to adduct and abduct his fingers B. Inability to flex his fingers C. Inability to flex his thumb D. Sensory loss over the lateral surface of his forearm E. Sensory loss over the medial surface of his forearm

D. The musculocutaneous nerve innervates the brachialis and biceps brachii muscles, which are the main flexors at the elbow. The biceps brachii inserts on the radius and is an important supinator. Because the musculocutaneous nerve is damaged in this case, it leads to loss of sensory perception to the lateral forearm, which is supplied by the distal continuation of the musculocutaneous nerve (known as the lateral antebrachial cutaneous nerve). The name "musculocutaneuous" indicates it is "muscular" in the arm and "cutaneous" in the forearm. Adduction and abduction of the fingers are mediated by the ulnar nerve and would not be affected in this instance. The flexor pollicis brevis muscle flexes the thumb and is mainly innervated by the recurrent branch of the median nerve. Flexion of the fingers is performed by the long flexors of the fingers and lumbrical muscles, innervated by the median and ulnar nerves. Sensory innervation of the medial forearm is provided by the medial antebrachial cutaneous nerve, usually a direct branch of the medial cord of the brachial plexus.

59 The right shoulder of a 78-year-old woman had become increasingly painful over the past year. Abduction of the right arm caused her to wince from the discomfort. Palpation of the deltoid muscle by the physician produced exquisite pain. Imaging studies reveal intermuscular inflammation extending over the head of the humerus. Which structure was inflamed? A. Subscapular bursa B. Infraspinatus muscle C. Glenohumeral joint cavity D. Subacromial bursa E. Teres minor muscle

D. The patient is suffering from subacromial or subdeltoid bursitis. (If the pain on palpation is less when the arm has been elevated to the horizontal, the bursitis may be thought of as being more subacromial, that is, associated more with the supraspinatus tendon perhaps, for such a bursa may be drawn back under the acromion when the limb is abducted.) The subscapular bursa, beneath the subscapularis muscle, would not present as superficial pain. It can communicate with the glenohumeral joint cavity. Inflammation or arthritic changes within the glenohumeral joint present as more generalized shoulder pain than that present here. The teres minor muscle and tendon are located inferior to the point of marked discomfort. GAS 708, 713; N 424; McM 136

15 A 24-year-old man is admitted with a wound to the palm of his hand. He cannot touch the pad of his index finger with his thumb but can grip a sheet of paper between all fingers and has no loss of sensation on the skin of his hand. Which of the following nerves has most likely been injured? A. Deep branch of ulnar B. Anterior interosseous C. Median D. Recurrent branch of median E. Deep branch of radial

D. The recurrent branch of the median nerve is motor to the muscles of the thenar eminence, which is an elevation caused by the abductor pollicis brevis, flexor pollicis brevis, and opponens pollicis muscles. If the opponens pollicis is paralyzed, one cannot oppose the pad of the thumb to the pads of the other digits because this is the only muscle that can oppose the thumb by moving the first metacarpal on the trapezium. The recurrent branch does not have a cutaneous distribution. Holding a piece of paper between the fingers is a simple test of adduction of the fingers. These movements are controlled by the deep branch of the ulnar nerve, which is not injured in this patient.

47 A 29-year-old woman is examined in the emergency department after falling from her balcony. Radiographic examination reveals that she has suffered a broken clavicle, with associated internal bleeding. Which of the following vessels is most likely to be injured in clavicular fractures? A. Subclavian artery B. Cephalic vein C. Lateral thoracic artery D. Subclavian vein E. Internal thoracic artery

D. The subclavian vein traverses between the clavicle and first rib and is the most superficial structure to be damaged following a fracture of the clavicle. The subclavian artery runs posterior to the subclavian vein, and though it is in the appropriate location, it would likely not be damaged because of its deep anatomic position. The cephalic vein is a tributary to the axillary vein after ascending on the lateral side of the arm. Its location within the body is too superficial and lateral to the site of injury. The lateral thoracic artery is a branch from the axillary artery that runs lateral to the pectoralis minor. It courses inferior and medial from its point of origin from the axillary artery, and it does not maintain a position near the clavicle during its descent. The internal thoracic artery arises from the first part of the subclavian artery before descending deep to the costal cartilages. Its point of origin from the subclavian artery is lateral to clavicular injury. Furthermore, its course behind the costal cartilages is quite medial to the clavicular fracture. GAS 694, 736-737; N 415; McM 129

E. Because the hamstrings cross two joints and are very crucial during all phases of running, but especially during the late swing through midstance phase of running, are easily injured. Their normal action includes hip extension and knee flexion. The do not rotate the hip.

During a 100-meter sprint a 25-year-old male Olympic athlete suddenly pulls up in discomfort and is seen to be clutching the back of his left thigh in agony. Upon further examination the athlete describes the pain as a "tearing" sensation and is unable to flex his knee. Based on these symptoms which of the following actions are affected due to this injury? A. Flexion of the hip and extension of the knee B. Extension of the hip and dorsiflexion C. Medial rotation of the hip D. Lateral rotation of the hip E. Hip extension and knee flexion

D. The fibularis (peroneal) brevis muscle originates from the lateral lower two thirds of the shaft of the fibula and inserts on the tubercle at the base of the fifth metatarsal. Any injury to this area will affect this muscle. Patients will present with a weakness in the eversion of the foot. Fibularis (peroneus) longus, extensor hallucis longus and tibialis anterior all insert on the medial side of the foot and will not be affected in this patient. The gastrocnemius inserts via the Achilles tendon to the posterior surface of the calcaneus.

During a battle, a 19-year-old soldier is shot in the lateral aspect of the right foot by a bullet that ricocheted off a building. The soldier is taken to a field hospital. A radiograph of the foot reveals that the base of the fifth metatarsal was completely obliterated. Which of the following muscles is most likely affected by this injury? A. Tibialis anterior B. Fibularis (peroneus) longus C. Gastrocnemius D. Fibularis (peroneus) brevis E. Extensor hallucis longus

B. The gluteal region can be divided into quadrants by two lines positioned using palpable bony landmarks. One line runs inferiorly from the highest point of the iliac crest. The second line runs horizontally midway between the iliac crests and the ischial tuberosity. This divides the gluteal region into four quadrants. The sciatic nerve runs through the lower medial quadrant and must be avoided during intragluteal injections.

During an interview, a 30-year-old man who is a psychiatric patient suddenly becomes aggressive. In order to calm him down, the patient is given an intramuscular injection in the upper lateral quadrant of the buttock. The injection is given at this specific location to prevent damage to which of the following nerves? A. Lateral femoral cutaneous B. Sciatic C. Superior gluteal D. Obturator E. Inferior gluteal

B. The medial branch of the deep fibular (peroneal) nerve accompanies the dorsalis pedis artery and innervates the skin between the contiguous sides of the first and second toes. The saphenous nerve is responsible for cutaneous innervation of the anteromedial aspect of the leg and foot. The superficial fibular (peroneal) nerve innervates most of the dorsum of the foot, with the exception of the area where sensation was lost (medial branch of deep fibular nerve). The common fibular (peroneal) nerve gives off a cutaneous branch, the sural nerve, which innervates the lateral aspect of the leg and lateral side of the foot.

During the preparation of an evening meal a female medical student dropped a sharp, slender kitchen knife. The blade pierced the first web space of her foot, resulting in numbness along adjacent sides of the first and second toes. Which nerve was most likely injured? A. Saphenous B. Deep fibular (peroneal) C. Superficial fibular (peroneal) D. Sural E. Common fibular (peroneal)

60 A 55-year-old male metallurgist had been diagnosed with carpal tunnel syndrome. To begin the operation, an anesthetic injection into his axillary sheath was given instead of general anesthesia. From which of the following structures does the axillary sheath take origin? A. Superficial fascia of the neck B. Superficial cervical investing fascia C. Buccopharyngeal fascia D. Clavipectoral fascia E. Prevertebral fascia

E. The axillary sheath is a fascial continuation of the prevertebral layer of the deep cervical fascia extending into the axilla. It encloses the nerves of the neurovascular bundle of the upper limb. Superficial fascia is loose connective tissue between the dermis and the deep investing fascia and contains fat, cutaneous vessels, nerves, lymphatics, and glands. The buccopharyngeal fascia covers the buccinator muscles and the pharynx mingles with the pretracheal fascia. The clavipectoral muscle invests the clavicle and pectoralis minor muscle. The axillary fascia is continuous with the pectoral and latissimus dorsi fascia and forms the hollow of the armpit. GAS 721, 731; N 412; McM 361

27 Following several days of 12-hour daily rehearsals of the symphony orchestra for a performance of a Wagnerian opera, the 52-year-old male conductor experienced such excruciating pain in the posterior aspect of his right forearm that he could no longer direct the musicians. When the maestro's forearm was palpated 2 cm distal and posteromedial to the lateral epicondyle, the resulting excruciating pain caused him to grimace. Injections of steroids and rest were recommended to ease the pain. Which of the following injuries is most likely? A. Compression of the ulnar nerve by the flexor carpi ulnaris B. Compression of the median nerve by the pronator teres C. Compression of the median nerve by the flexor digitorum superficialis D. Compression of the superficial radial nerve by the brachioradialis E. Compression of the deep radial nerve by the supinator

E. The deep branch of the radial nerve courses between the two heads of the supinator muscle and is located just medial and distal to the lateral epicondyle. After the nerve emerges from the supinator it is called the posterior interosseous nerve. It can be irritated by hypertrophy of the supinator, which compresses the nerve, causing pain and weakness. The ulnar nerve courses laterally behind the medial epicondyle and continues anterior to the flexor carpi ulnaris muscle. The median nerve passes into the forearm flexor compartment; the superficial radial nerve courses down the lateral aspect of the posterior forearm and would not cause pain due to pressure applied to the posterior forearm.

33 As she fell from the uneven parallel bars, a 17-year-old female gymnast grasped the lower bar briefly with one hand but then fell painfully to the floor. An MRI examination reveals an injury to the medial cord of the brachial plexus. Which of the following spinal nerve levels would most likely be affected? A. C5, C6 B. C6, C7 C. C7, C8 D. C7, C8, T1 E. C8, T1

E. The medial cord has been injured by traction on the lower trunk of the brachial plexus. The medial cord is the continuation of the inferior (lower) trunk of the brachial plexus, which is formed by C8 and T1. C5 and C6 are typically associated with the superior (upper) trunk level and thus the lateral cord. C7 forms the middle trunk. An injury to the posterior cord would usually involve the C7 spinal nerve. This is a typical Klumpke paralysis. GAS 738-747; N 416; McM 139

52 A 17-year-old boy suffered the most common of fractures of the carpal bones when he fell on his outstretched hand. Which bone would this be? A. Trapezium B. Lunate C. Pisiform D. Hamate E. Scaphoid

E. The scaphoid (or the older term, navicular) bone is the most commonly fractured carpal bone. GAS 797; N 439; McM 167

55 An orthopedic surgeon exposed a muscle in the supraspinous fossa so that she could move it laterally while repairing an injured rotator cuff. As she reflected the muscle from its bed, an artery was exposed crossing the ligament that bridges the notch in the superior border of the scapula. What artery was this? A. Subscapular B. Transverse cervical C. Dorsal scapular D. Posterior circumflex humeral E. Suprascapular

E. The suprascapular artery passes over, and the suprascapular nerve passes under, the superior transverse scapular ligament. This ligament bridges the suprascapular notch in the upper border of the scapula, converting the notch to foramen. The artery and nerve then pass deep to the supraspinatus muscle, thereafter supplying it and then passing through the spinoglenoid notch to supply the infraspinatus. The subscapular artery is a branch of the third part of the axillary artery; it divides into circumflex scapular and thoracodorsal branches. The transverse cervical artery courses anterior to this site. The dorsal scapular artery and nerve pass deep to the medial border of the scapula. The posterior circumflex humeral branch of the axillary artery passes through the quadrangular space with the axillary nerve. GAS 719-720; N 414; McM 133

E. The posterior cruciate ligament is responsible for preventing the forward sliding of the femur on the tibia. The anterior cruciate ligament prevents posterior displacement of the femur on the tibia. The lateral collateral ligament limits extension and adduction of the leg. The medial meniscus acts as a shock absorber and cushions the articular surfaces of the knee joint.

Following an injury suffered in a soccer match, a 32-year-old woman is examined in a seated position in the orthopedic clinic. Holding the right tibia with both hands, the clinician can press the tibia backward under the distal part of her femur. The left tibia cannot be displaced in this way. Which structure was most likely damaged in the right knee? A. Anterior cruciate ligament B. Lateral collateral ligament C. Medial collateral ligament D. Medial meniscus E. Posterior cruciate ligament

B. The posterior femoral cutaneous nerve arises from nerves S1 to S3. It provides inferior cluneal branches to the lower portion of the gluteal region and a perineal branch to the perineum and supplies sensation to the posterior thigh to the level of the popliteal fossa. Superior gluteal innervation arises from dorsal rami of L1 to L3. Meralgia paresthetica is the occurrence of pain or burning sensations on the lateral thigh, from compression of the lateral femoral cutaneous nerve. The sural nerve, sensory to the lower calf and lateral foot, arises from contributions from the tibial nerve and common fibular (peroneal) nerve. The posterior femoral cutaneous is a sensory nerve and does not innervate muscles.

Following the insertion of a prosthetic hip joint in a 72-year-old man, it was observed that the patient had greatly diminished sensation in the region of distribution of the posterior femoral cutaneous nerve. Which of the following is characteristic of this nerve? A. Cutaneous supply of the superior aspect of the gluteal region B. Arises from sacral spinal nerve levels S1, S2, S3 C. Motor innervation of the obturator internus and gemelli muscles D. Injury results in meralgia paresthetica E. Provides origin of the sural nerve

B. The iliopsoas forms the lateral portion of the troughlike floor of the femoral triangle. The pectineus forms the medial portion of this floor. The adductor longus provides a medial border for the femoral triangle and meets the sartorius, the lateral border of the triangle, at the apex. The rectus femoris is a superficial contributor to the quadriceps femoris, lateral to the femoral triangle.

In preparing to isolate the proximal portion of the femoral artery, the vascular surgeon gently separated it from surrounding tissues. Posterior to the femoral sheath, what muscle forms the lateral portion of the floor of the femoral triangle? A. Adductor longus B. Iliopsoas C. Sartorius D. Pectineus E. Rectus femoris

D. The tendinous distal portion of the adductor magnus inserts on the adductor tubercle on the upper border of the medial condyle of the femur. The femoral artery passes through the adductor hiatus proximal to this tendinous band, continuing as the popliteal artery. The semimembranosus inserts on the proximal, posterior portion of the tibia. The gracilis inserts with the pes anserinus on the proximal, medial aspect of the tibia. The popliteus inserts on the distal lateral portion of the femur, just above the origin of the lateral head of gastrocnemius. The vastus medialis inserts with other quadriceps muscle components on the patella and then on to the tibial tuberosity.

In the radiographs of the knee of a male 28-year-old basketball player, who had apparently suffered a tear in a medial ligament of the knee, the tubercle on the superior aspect of the medial femoral condyle could be seen more clearly than in most individuals. What muscle attaches to this tubercle? A. Semimembranosus B. Gracilis C. Popliteus D. Adductor magnus E. Vastus medialis

E. The popliteal artery is the continuation of the femoral artery after it passes through the hiatus of the adductor magnus. The popliteal artery divides into the anterior and posterior tibial arteries. The anterior tibial artery passes between the tibia and fibula proximally in the posterior compartment of the leg, whereas the posterior tibial artery continues in the posterior compartment of the leg, to its division into medial and lateral plantar arteries. The posterior tibial artery provides origin for the fibular (peroneal) artery, which supplies the lateral compartment of the leg. The deep femoral artery provides origin for the three or four perforating branches that supply the posterior compartment of the thigh. GAS 600, 617; N 509;

Lower limb angiography of an 82-year-old woman reveals a possible cause for her limb pain during her workout routines in the health spa. The artery that was occluded is one that should have been demonstrable passing between the proximal part of the space between the tibia and fibula. Which of the following arteries is most likely affected? A. Deep femoral B. Popliteal C. Posterior tibial D. Fibular (peroneal) E. Anterior tibial

B. The apex of the femoral triangle occurs at the junction of the adductor longus and sartorius muscles. The subsartorial (Hunter's) canal begins at this location. Immediately deep to this anatomic point lie the femoral artery, femoral vein, deep femoral artery, and deep femoral vein, often overlying one another in that sequence. This has historically been a site of injuries caused by slipping while handling a very sharp butcher's knife. For this reason, injuries at this location are referred to as the "butcher's block" injury. Fatal loss of blood can occur in just a few minutes if pressure, or a tourniquet, is not applied immediately. The common iliac artery becomes the femoral artery at the inguinal ligament. The saphenous vein joins the femoral vein at the saphenous hiatus, or fossa ovalis. The medial circumflex femoral usually arises from the deep femoral artery about 3 to 5 inches inferior to the inguinal ligament, near the origin of the deep femoral artery from the common femoral. Serious blood loss can occur with injury to any of these vessels, although injury to them is not often fatal.

The news reported that the 58-year-old ambassador received a slashing wound to the medial thigh and died from exsanguination in less than 2 minutes. What was the most likely nature of his injury? A. The femoral artery was cut at the inguinal ligament B. A vessel or vessels were injured at the apex of the femoral triangle C. The femoral vein was transected at its junction with the saphenous vein D. The medial circumflex femoral was severed at its origin E. The deep femoral artery was divided at its origin

A. The patellar ligament is a very heavy ligament that connects the patella to the tibial tuberosity; it provides the insertion of the quadriceps femoris tendon upon the tibia. The patella can be thought of as a bone (a sesamoid bone) that develops within the tendon of the quadriceps femoris muscle. When the reflex hammer strikes the patellar ligament, it stretches the ligament slightly for a brief time, resulting in reflex contraction of the quadriceps femoris muscles. This reflex arc is elicited by the femoral nerve (L4 sensory input component and L2, L3 motor output). The quadriceps femoris includes the rectus femoris and the vastus lateralis, intermedius, and medialis. The patella is the largest sesamoid bone in the body. A sesamoid bone is a bone that develops within a tendon. The quadratus femoris muscle of the gluteal area arises from the ischial tuberosity and inserts on the femur proximally. The sartorius arises from the anterior superior iliac spine and inserts on the proximal, medial aspect of the tibia as one of the three tendinous components of the pes anserinus (goose foot). The biceps femoris of the posterior thigh has a long head that arises from the ischial tuberosity and a short head that arises from the femur; they insert on the head of the fibula.

The patellar reflex appears to be markedly reduced in a 33-year-old diabetic female patient, due to deficient vascular supply of the nerves of her lower limb. The tendon of which of the following muscles is stretched during the patellar reflex? A. Quadriceps femoris B. Quadratus femoris C. Sartorius D. Pectineus E. Biceps femoris

B. The common fibular (peroneal) nerve winds around the neck of the fibula before dividing into superficial and deep branches that go on to innervate the lateral and anterior compartments of the leg, respectively. These compartments are responsible for dorsiflexion and eversion of the foot, and injury to these nerves would result in deficits in these movements. The tibial nerve lies superficially in the popliteal fossa. This nerve innervates the posterior compartment of the leg, so compression in this area would result in a loss of plantar flexion and weakness of inversion. The lateral compartment of the leg is innervated by the superficial fibular (peroneal) nerve and is mainly involved in eversion of the foot. The cutaneous branches of the superficial fibular (peroneal) nerve emerge through the deep fascia in the anterolateral aspect of the leg and supply the dorsum of the foot. The anterior compartment of the leg is innervated by the deep fibular (peroneal) nerve and is mainly involved in dorsiflexion of the foot. The medial malleolus is an inferiorly directed projection from the medial side of the distal end of the tibia. The tibial nerve runs near the groove behind the medial malleolus, and compression at this location would result in loss of toe flexion, adduction, abduction, and abduction of the great toe

Upon removal of a knee-high leg cast, a 15-year-old boy complains of numbness of the dorsum of his right foot and inability to dorsiflex and evert his foot. Which is the most probable site of the nerve compression that resulted in these symptoms? A. Popliteal fossa B. Neck of the fibula C. Lateral compartment of the leg D. Anterior compartment of the leg E. Medial malleolus

C. Herniation of the intervertebral disc at L4-5 results typically in compression of the L5 spinal nerve. The L4 spinal nerve exits at the L4-5 intervertebral foramen, but the L5 spinal nerve is put under tension as it passes the herniation to reach the L5-1 foramen. Piriformis entrapment of the fibular (peroneal) division of the sciatic nerve is relatively common, but the dermatome affected here appears to be confined to the L5 distribution to the skin of the foot and also includes the superior gluteal nerve, which supplies the large hip abductors. S1 would involve loss of sensation on the lateral side of the foot and potential weakness in hip extension and plantar flexion. A posterior dislocation of the hip would be unlikely in this injury but, even so, would not result in these deficits.

When he attempted to lift one side of his new electric automobile from the ground to demonstrate his strength, a 51-year-old man felt a sharp pain in his back and quickly dropped the vehicle. Upon examination, it is observed that the patient has deficits in sensation on the dorsum and sole of his foot and marked weakness in abduction and lateral rotation of the lower limb. What was the nature of his injury? A. Piriformis syndrome, with entrapment of the sciatic nerve B. Disc lesion at L3-4 C. Disc lesion at L4-5 D. Disc lesion at L5-S1 E. Posterior hip dislocation


Ensembles d'études connexes

Computer Science Chapters 1 and 2

View Set

Microeconomics Chapters 10, 12-14

View Set

UNIT 3- Cell Division and Cell Energy

View Set

PUCH60 NEUROLOGIC ASSESSMENT PART 1

View Set